davidson's mcqs

August 5, 2017 | Author: Sekhons Akademy | Category: Pulse, Heart, Leukemia, Clinical Medicine, Rtt
Share Embed Donate


Short Description

Download davidson's mcqs...

Description

MCQs VIA WEB 2005 Medicine MCQS VIA Web Copyright © 2005 Elsevier Limited. All rights reserved. Fleshandbones is a registered trademark of Harcourt, Inc. in the United States and other jurisdictions, used under license These mcqs were donloaded By Ahmed Hakim [email protected] TEST Module4 Question 1. The pulse: (a) In pulsus paradoxus the rate slows during inspiration. (False) (b) Pulsus alternans indicates a poorly functioning left ventricle. (True) (c) A tachycardia of 150 beats per minute in a resting patient usually implies an underlying cardiac arrhythmia. (True) (d) A collapsing pulse may be noticed in thyrotoxicosis. (True) (e) Corrigan's sign supports a diagnosis of aortic stenosis. (False) Question 2. Heart murmurs: (a) A low rumbling diastolic murmur with presystolic accentuation may be heard in mitral stenosis accompanied by atrial fibrillation. (False) (b) Causes of a pansystolic murmur include mitral regurgitation and ventricular septal defect. (True) (c) A systolic murmur heard over the whole praecordium associated with a thrill usually indicates aortic stenosis. (True) (d) Left heart murmurs are best heard during expiration. (True) (e) An early blowing diastolic murmur at the left sternal edge indicates aortic incompetence. (True) Question 3. Pulsus paradoxus: (a) The volume of the pulse increases in inspiration. (False) (b) Can be confirmed by detecting >10 mmHg difference in systolic pressure during the breathing cycle. (True) (c) Is a sign of severe asthma. (True) (d) Is called paradoxus because it is the opposite of what normally happens to the pulse. (False) (e) Can occur in cardiac tamponade. (True) Question 4. The jugulovenous pressure: (a) Is raised if it is 2 cm from the sternal angle with the patient seated at 45°. (False) (b) Tall 'a' waves may be seen in pulmonary hypertension. (True) (c) Irregular cannon waves indicate complete heart block. (True) (d) Regular cannon waves may indicate a nodal rhythm. (True) (e) Giant 'v' waves and a pulsatile liver indicate tricuspid stenosis. (False) Question 5. The physical signs of an uncomplicated large pneumothorax include: (a) The trachea deviated to the opposite side. (False) (b) A clicking sound synchronous with the heart beat. (True) (c) Symmetrical expansion of the chest. (False) (d) Increased breath sounds over the pneumothorax. (False) (e) Increased percussion note over the pneumothorax. (True) Question 6. The following would help distinguish between a kidney and a spleen in the left upper quadrant: (a) Dull to percussion over the mass. (False) (b) A well-localized notched lower margin. (False) (c) Moves with respiration. (False) (d) A ballottable mass. (True) (e) A family history of renal failure. (True) Question 7. Nystagmus: (a) Vertical nystagmus usually indicates a lesion of the medulla oblongata. (False) (b) Horizontal nystagmus is usually ipsilateral to an irritative lesion of the labyrinth. (False) (c) Ataxic nystagmus indicates a lesion of the medial longitudinal bundle. (True) (d) May be absent in a lesion of the cerebellar vermis (the central part). (True)

By A. H.

MCQs VIA WEB 2005 (e) Pendular nystagmus may indicate partial blindness. (True) Question 8. The following would suggest an upper rather than a lower motor neuron lesion: (a) Fasciculation. (False) (b) Increased tone. (True) (c) An absent plantar reflex. (False) (d) Clonus. (True) (e) Relatively little wasting. (True) Question 9. Hand signs: (a) Clubbing may be caused by uncomplicated chronic bronchitis. (False) (b) Koilonychia usually indicates liver disease. (False) (c) Osler's nodes and Heberden's nodes both occur in osteoarthritis. (False) (d) Splinter haemorrhages are due to embolic rather than immunological phenomena. (False) (e) Psoriatic arthritis affects most joints in the hand but usually spares the distal interphalangeal (DIP) joints. (False) (True) Question 10. The face: (a) A malar flush may indicate mitral valve disease or hypothyroidism. (True) (b) A butterfly rash in the face is seen in dermatomyositis. (False) (c) Bell's palsy can cause ptosis due to paralysis of orbicularis oculi. (False) (d) Herpes labialis may be associated with pneumococcal pneumonia. (True) (e) An expressionless face and drooling could indicate Parkinson's disease. (True) Question 11. The electrocardiogram: (a) The PR interval is measured from the peak of the P wave to the start of the QRS complex. (False) (b) Right axis deviation is indicated by a QRS axis of -35°. (False) (c) Q waves in S-II, S-III and aVf indicate a transmural inferior myocardial infarction. (True) (d) Left bundle branch block is suggested by broadening of the QRS complex to 0.10 seconds (two and a half little squares), and positive RSR' waves in V4-V6. (False) (e) P mitrale is suggested by a P wave taller than 2.5 mm. (False) Question 12. In the full blood count: (a) A haemoglobin of 10.0 g/dL would be considered normal in a premenopausal woman. (False) (b) Polycythaemia rubra vera is usually indicated by elevation not only of the haemoglobin but also of the white cell count and platelets. (True) (c) A low platelet count could indicate a flare-up of systemic lupus erythematosus (SLE). (True) (d) High platelets can be seen in gastrointestinal bleeding. (True) (e) A raised mean corpuscular volume is usual in significant alcohol excess. (True) Question 13. Heart failure: (a) The clinical features of left heart failure include: tachycardia, basal crepitations, pulsus alternans and a raised JVP. (False) (b) Congestion of the pulmonary veins alone does not result in orthopnoea. (False) (c) Chronic congestive heart failure leads to secondary hyperaldosteronism. (True) (d) Causes of heart failure include ischaemic heart disease, hypertension, and thiamine deficiency. (True) (e) Clinical features of right heart failure include a raised JVP, ankle oedema, and hepatomegaly. (True) Question 14. Stroke: (a) Cerebral haemorrhage accounts for more than 40% of acute strokes. (False) (b) In supratentorial strokes with homonymous hemianopsia, patients cannot see on the hemiplegic side. (True) (c) Vertigo, vomiting, dysphagia, and Horner's syndrome indicate occlusion of the vertebrobasilar circulation. (True) (d) Pinpoint pupils and bilateral upgoing plantars could signal a brainstem stroke. (True) (e) Carotid endarterectomy should be considered for patients with more than 70% stenosis because this is more effective than medical treatment. (True) Question 15. Respiratory failure:

By A. H.

MCQs VIA WEB 2005 (a) Type I failure results in a partial pressure of oxygen (pO2) 6.5 kPa. (False) (b) In respiratory failure associated with chronic bronchitis, the level of carbon dioxide (CO2) determines the respiratory rate. (False) (c) Respiratory failure as defined in (a) would be an indication for ventilation in pure asthma. (True) (d) Doxapram is a respiratory stimulant used in respiratory failure associated with chronic obstructive pulmonary disease. (True) (e) The main aim in type II failure is to keep the pO2 >7.0 kPa without worsening of the acidosis or pCO2. (True) Question 16. Cushing's syndrome: (a) May give rise to hypertension, diabetes, and truncal obesity. (True) (b) Is usually diagnosed by estimation of the urinary free cortisol followed by an overnight dexamethasone suppression test. (True) (c) Could be associated with pigmentation. (True) (d) The most common cause is probably iatrogenic. (True) (e) Nelson's syndrome is a complication of bilateral adrenalectomy for pituitary-dependent Cushing's disease. (True)

Question 17. Leukaemia: (a) The common presenting triad is infection, bleeding, and fatigue. (True) (b) Acute myeloid leukaemia (AML) may result spontaneously or follow on from CML, polycythaemia rubra vera or myelosclerosis. (True) (c) The usual development of chronic lymphocytic leukaemia is a transformation to acute lymphoblastic leukaemia. (False) (d) A platelet count of 40 × 109/L would not normally give rise to spontaneous bleeding. (True) (e) Bone marrow transplantation is a recognized treatment for AML. (True) Question 18. Hypertension: (a) An average diastolic blood pressure of >90 mmHg over prolonged observation is an indication for drug treatment in uncomplicated hypertension. (False) (b) Thiazide diuretics are the least effective antihypertensive drugs. (False) (c) Thiazide diuretics work on the loop of Henle in the kidney. (False) (d) Resistant hypertension is defined as a failure to control the blood pressure adequately with a good three-drug regimen. (True) (e) Thiazide diuretics are contraindicated in gout and diabetes. (True) Question 19. Oral corticosteroids: (a) Are an effective treatment for SLE. (True) (b) In the long term may cause cataracts. (True) (c) Should be avoided in sarcoidosis because they induce pulmonary oedema. (False) (d) May be stopped abruptly after 2 weeks of 40 mg prednisolone daily in patients who are not exposed to repeated courses. (True) (e) May reveal that 15% of patients labelled as having chronic bronchitis, in fact have reversible airways disease. (True) Question 20. Paracetamol overdose: (a) Ipecacuana followed by oral methionine is effective for most patients who are just over the treatment line. (False) (b) Can cause renal failure. (True) (c) Intravenous N-acetylcysteine frequently causes anaphylaxis. (False) (d) The serum paracetamol level is of most value between 1 and 4 hours after ingestion. (False) (e) In co-proxamol (distalgesic) overdose, sudden death is likely to be due to hypoglycaemia caused by paracetamol. (False) Question 21. Treatment of myocardial infarction: (a) Aspirin and streptokinase are more effective than either alone after myocardial infarction. (True) (b) Thrombolysis improves short-term complications but not mortality after myocardial infarction. (False)

By A. H.

MCQs VIA WEB 2005 (c) Tissue plasminogen activator and anistreplase are more effective than streptokinase but not used because they are far more expensive. (False) (d) ACE inhibitors improve outcome after myocardial infarction for patients with ventricular dysfunction. (True) (e) HMGCo-A reductase inhibitor therapy is contraindicated for patients after myocardial infarction. (False) Question 22. For self-poisoning: (a) Gastric lavage is recommended for most drugs up to 12 hours after ingestion. (False) (b) Naloxone is the specific antidote for benzodiazepine overdose. (False) (c) Patients with tricyclic antidepressant overdose need cardiac monitoring for up to 48 hours. (True) (d) All patients should be assessed by a qualified psychiatrist. (False) (e) Pinpoint pupils could indicate opiate overdose. (True) Question 23. Digoxin: (a) Is the treatment of choice for ventricular extrasystoles. (False) (b) May cause xanthopsia. (True) (c) Is excreted by the kidneys. (True) (d) Adverse effects are reduced by hypokalaemia. (False) (e) Must not be coadministered with an ACE inhibitor. (False) Question 24. Dementia may result from: (a) Parkinson's disease. (True) (b) Huntington's chorea. (True) (c) Hypothyroidism. (True) (d) Acquired immune deficiency syndrome (AIDS). (True) (e) A cerebral tumour. (True) Question 25. Oxygen: (a) Should be administered with a high inspired concentration (>50%) in the treatment of type II respiratory failure. (False) (b) Should not be used at high concentration in patients with pulmonary embolism because respiration may be severely impaired when the hypoxic drive is reduced. (False) (c) Continuous long-term (domiciliary) oxygen improves survival in patients with respiratory failure caused by chronic bronchitis and emphysema. (True) (d) Is needed when respiratory failure is diagnosed by finding a pO2 of less than 11 kPa in an arterial blood sample. (False) (e) Comprises 21% of atmospheric air. (True) Module 5 (Cardiology) Question 2. The differential diagnosis for chest pain includes: (a) Myocardial infarction. (True) (b) Oesophagitis. (True) (c) Pulmonary embolus. (True) (d) Cholecystitis. (True) (e) Aortic dissection. (True) Question 3. The following are causes of acute life-threatening dyspnoea: (a) Myocardial infarction. (True) (b) Pulmonary embolus. (True) (c) Pneumothorax. (True) (d) Ventricular or supraventricular tachyarrhythmia. (True) (e) Bacterial endocarditis. (True) Question 4. The following are clinical signs found in infective endocarditis: (a) Clubbing. (True) (b) Haematuria. (True) (c) Pyrexia. (True) (d) Rashes. (True)

By A. H.

MCQs VIA WEB 2005 (e) Focal neurological defect. (True) Question 5. The following are risk factors for ischaemic heart disease: (a) Hypertension. (True) (b) Moderate alcohol intake. (False) (c) Female sex. (False) (d) Hypercholesterolaemia. (True) (e) Increasing age. (True) Question 6. The following are classical features of cardiac syncope: (a) Gradual onset. (False) (b) Warning symptoms. (False) (c) Rapid recovery. (True) (d) Residual neurological deficit. (False) (e) Precipitated by sudden turning of the head. (False) Question 7. The following are causes of a pansystolic murmur: (a) Mitral regurgitation. (True) (b) Aortic regurgitation. (False) (c) Tricuspid regurgitation. (True) (d) Atrial septal defect. (False) (e) Aortic stenosis. (False) Question 8. The following conditions require antibiotic prophylaxis before dental procedures: (a) Prosthetic aortic valve. (True) (b) Ventricular septal defect. (True) (c) Floppy mitral valve with coexistent mitral regurgitation. (True) (d) Enlarged left ventricle. (False) (e) A history of infective endocarditis in the past. (True) Question 9. The following should be considered as possible signs of a positive exercise test: (a) ST segment depression. (True) (b) Exercise-induced hypotension. (True) (c) Exercise-induced ventricular tachycardia. (True) (d) Lack of adequate tachycardic response to exercise. (True) (e) Leg pain at peak exercise. (False)

Question 10. The following are indications for anticoagulating a patient who has atrial fibrillation with warfarin: (a) Age under 60 years. (False) (b) Associated mitral stenosis. (True) (c) Atrial fibrillation of more than 24 hours' duration. (True) (d) A history of cerebral thromboembolism. (True) (e) Associated left ventricular failure. (True) Question 11. The following are true of ventricular tachycardia: (a) It is a life-threatening condition. (True) (b) It may be caused by myocardial ischaemia. (True) (c) It may be caused by hypokalaemia. (True) (d) Amiodarone may be used to prevent recurrent episodes of ventricular tachycardia. (True) (e) Acute ongoing ventricular tachycardia should be treated initially with drugs. (False) Question 12. The following are signs of coarctation of the aorta: (a) Radiofemoral delay in the pulses. (True) (b) Rib notching. (True) (c) Bruits heard over the scapula. (True) (d) Ankle oedema. (False)

By A. H.

MCQs VIA WEB 2005 (e) Atrial fibrillation. (False) Question 13. Functions of the recovery position include: (a) To prevent the tongue from obstructing the airway. (True) (b) To prevent neck injury. (False) (c) To minimize the risk of aspiration of gastric contents. (True) (d) To maintain a straight airway. (True) (e) To enable cardiopulmonary resuscitation to be carried out. (False)

Question 14. Complications of prosthetic heart valves are as follows: (a) Thromboembolic events. (True) (b) Dehiscence of the valve ring. (True) (c) Increased risk of infective endocarditis. (True) (d) Failure of the valve 5 years after placement. (False) (e) Need for anticoagulation in patients who have porcine valves. (False) Question 15. The following statements are true of thiazide diuretics: (a) They act at the level of the distal convoluted tubule. (True) (b) They may cause gout. (True) (c) Diabetic control may deteriorate. (True) (d) Hypokalaemia may occur. (True) (e) They cause ototoxicity. (False) Question 16. The following are classified as high-output states: (a) Hypertension . (False) (b) Sepsis. (True) (c) Hypothyroidism. (False) (d) Pregnancy. (True) (e) Arteriovenous malformations. (True) Question 18. The following statements are true of the apex beat: (a) It is the lowest and most lateral point at which the cardiac impulse can be felt. (True) (b) It is displaced downwards and laterally if the left ventricle is enlarged. (True) (c) It is thrusting in mitral stenosis. (False) (d) It is thrusting in aortic regurgitation. (True) (e) It is heaving in aortic stenosis. (True) Question 17. Cardiac causes of clubbing are as follows: (a) Uncomplicated atrial septal defect. (False) (b) Chronic infective endocarditis. (True) (c) Atrial fibrillation. (False) (d) Acute endocarditis. (False) (e) Empyema. (False) Question 19. The following leads represent the inferior myocardium: (a) I, AVL, and V6. (False) (b) V2, V3, and V4. (False) (c) AVR and V1. (False) (d) V1-V6. (False) (e) II, III, and AVF. (True) Question 20. The following are possible causes of electromechanical dissociation: (a) Pulmonary embolus. (True) (b) Tension pneumothorax. (True) (c) Hypertension. (False) (d) Dehydration. (True)

By A. H.

MCQs VIA WEB 2005 (e) Hypocalcaemia. (True) Question 21. The following are characteristic of pericarditis: (a) The chest pain is dull in nature. (False) (b) There may be an associated pericardial effusion. (True) (c) The pericardial rub may come and go. (True) (d) The ECG usually shows regional ST elevation. (False) (e) The ST elevation is concave. (True)

Question 22. Secondary hypertension may be due to the following: (a) Renal artery stenosis. (True) (b) Renal cell carcinoma. (False) (c) Cushing's syndrome. (True) (d) Pregnancy. (True) (e) Oral contraceptive pill. (True) Question 23. ECG changes due to myocardial infarction may include the following: (a) ST elevation. (True) (b) Sinus tachycardia. (True) (c) Ventricular tachycardia. (True) (d) Complete heart block. (True) (e) Q waves. (True) Question 24. The following drugs are used in the treatment of hypertension: (a) Atenolol. (True) (b) Doxazocin. (True) (c) Enalapril. (True) (d) Bendrofluazide. (True) (e) Nicorandil. (False) Question 25. Complications of myocardial infarction include: (a) Cardiac failure. (True) (b) Mitral regurgitation. (True) (c) Cerebrovascular event. (True) (d) Myocardial rupture. (True) (e) Gastrointestinal bleed. (False) Module 6 (Neurology) Question 1. Concerning neuroanatomy: (a) The corticospinal tract decussates in the pons. (False) (b) The oculomotor nerve runs in close proximity to the posterior communicating artery. (True) (c) The superior colliculus is found in the midbrain. (True) (d) The trochlear (fouth cranial) nerve supplies the lateral rectus muscle. (False) (e) The spinal cord ends at the level of the lower border of L3 in the adult. (False) Question 2. Subdural haematomas can cause: (a) Dementia. (True) (b) Pupillary change. (True) (c) Bradycardia. (True) (d) Changing level of consciousness. (True) (e) Blood-stained cerebrospinal fluid (CSF). (False) Question 3. In a young woman with a spastic paraparesis, the following suggest a diagnosis of multiple sclerosis: (a) Delayed visual evoked potentials. (True) (b) Fasciculations. (False)

By A. H.

MCQs VIA WEB 2005 (c) Raised CSF protein. (False) (d) Oligoclonal bands in the CSF. (True) (e) Periventricular white matter lesions on magnetic resonance imaging (MRI) of the brain. (True) Question 4. Unilateral facial weakness is a recognized feature of: (a) Herpes zoster infection. (True) (b) Motor neuron disease. (False) (c) Acoustic neuroma. (True) (d) Cholesteatoma. (True) (e) Syringomyelia. (False) Question 5. The following are true about headaches: (a) The headache of raised intracranial pressure is worst at the end of the day. (False) (b) A normal CT scan rules out subarachnoid haemorrhage. (False) (c) Amaurosis fugax may be caused by temporal arteritis. (True) (d) Neurological signs on examination rules out migraine as a diagnosis. (False) (e) Cluster headaches are more common in men than in women. (True) Question 6. The following drugs can produce parkinsonism: (a) Chlorpromazine. (True) (b) Benzhexol. (False) (c) Bromocriptine. (False) (d) Metoclopramide. (True) (e) Haloperidol. (True) Question 7. Concerning movement disorders: (a) Huntington's chorea presents with progressive dementia and chorea in middle age. (True) (b) Myoclonus is a feature of subacute sclerosing panencephalitis. (True) (c) Infarction of the subthalamic nucleus causes ipsilateral hemiballism. (False) (d) Chorea is commonly found in Cruetzfeldt-Jakob disease. (False) (e) Alcohol reduces benign essential tremor. (True) Question 8. Concerning papilloedema: (a) There is loss of venous pulsation on funduscopy. (True) (b) There may be enlargement of the blind spot. (True) (c) Intracranial pressure may be normal. (True) (d) Hypocalcaemia is a recognized cause. (True) (e) It is a recognized feature in Guillain-Barré syndrome. (True) Question 9. Ptosis may be a feature of: (a) Myotonic dystrophy. (True) (b) Horner's syndrome. (True) (c) Abducens nerve (sixth nerve ) palsy. (False) (d) Oculomotor nerve (third nerve) palsy. (True) (e) Myasthenia gravis. (True) Question 10. Concerning the Brown-Séquard syndrome: (a) There is ipsilateral corticospinal loss below the lesion. (True) (b) There is ipsilateral loss of joint-position sense below the lesion. (True) (c) There is ipsilateral loss of two-point discrimination below the level of the lesion. (True) (d) There is ipsilateral loss of pain and temperature below the level of the lesion. (False) (e) A central disc lesion at L3 would cause a Brown-Séquard syndrome in the legs. (False) Question 11. Concerning the brachial plexus: (a) In brachial neuritis, severe pain around the shoulder precedes rapid wasting. (True) (b) Klumpke's paralysis causes proximal arm weakness. (False) (c) Erb's palsy is caused by a lesion to C5/C6-derived regions of the brachial plexus. (True)

By A. H.

MCQs VIA WEB 2005 (d) A brachial plexus lesion and an ipsilateral Horner's syndrome may indicate a Pancoast tumour. (True) (e) Vaccination may precipitate brachial neuritis. (True) Question 12. Causes of a polyneuropathy include: (a) Diabetes. (True) (b) Guillain-Barré syndrome. (True) (c) Renal failure. (True) (d) Amyloid. (True) (e) Multiple sclerosis. (False) Question 13. A lesion to the common peroneal nerve at the fibular head causes: (a) Weakness of eversion of the foot. (True) (b) Decreased sensation over the dorsum of the foot. (True) (c) Weakness of plantar flexion. (False) (d) If long term, wasting of tibialis anterior. (True) (e) Brisk ankle jerk. (False) Question 14. Brainstem death may be confirmed by: (a) Extensor response of the limbs to painful stimuli. (False) (b) Absent corneal reflexes. (True) (c) Absent tendon reflexes. (False) (d) A flat EEG. (False) (e) Absent 'doll's eye' reflexes. (True) Question 15. A homonymous hemianopia may arise from a lesion of: (a) The optic tract. (True) (b) The occipital cortex. (True) (c) The optic chiasm. (False) (d) The optic nerve. (False) (e) The optic radiation. (True) Question 16. Dysarthria may result from a lesion of: (a) The cerebellum. (True) (b) Broca's area. (False) (c) The hypoglossal nerve. (True) (d) The basal ganglia. (True) (e) The accessory nerve. (False) Question 17. The following are clinical features of cerebellar dysfunction (a) Postural tremor. (False) (b) Hypotonia. (True) (c) Dysphasia. (False) (d) Titubation. (True) (e) Impaired rapid altering movements. (True) Question 18. The following clinical features may help differentiate between a syncopal attack and a seizure: (a) Upright posture at the onset. (True) (b) Convulsive movements of the limbs. (False) (c) A bitten tongue. (True) (d) Urinary incontinence. (True) (e) Prolonged malaise after the attack. (False) Question 19. The following are features of a subarachnoid haemorrhage: (a) Fever. (True) (b) Thunderclap headache. (True) (c) Photophobia. (True) (d) Positive Kernig's sign. (True)

By A. H.

MCQs VIA WEB 2005 (e) Neck stiffness. (True) Question 20. A physiological tremor is: (a) Present at rest. (False) (b) Worsened by anxiety. (True) (c) Improved by alcohol. (False) (d) Improved by beta-blockers. (True) (e) Familial. (False) Question 21. A lesion of the medulla on one side may give rise to : (a) An ipsilateral hemiparesis. (False) (b) A contralateral hemiparesis. (True) (c) Ipsilateral weakness of the palate. (False) (d) Contralateral weakness of the tongue. (True) (e) Contralateral third nerve palsy. (False) Question 22. The following may be seen in a patient with a lesion of the third nerve or nucleus: (a) A fixed dilated pupil. (True) (b) Ptosis. (True) (c) Diplopia in all positions of gaze. (True) (d) A history of diabetes mellitus. (True) (e) A contralateral hemiplegia. (True) Question 23. In a patient with a sensory ataxia: (a) Vibration may be impaired. (True) (b) The gait is characterized by 'scissoring' posture of the legs. (False) (c) Romberg's test may be positive. (True) (d) A history of alcohol abuse may be implicated in the aetiology. (True) (e) Clonus may be elicited on examination of the legs. (False) Question 24. A patient with herpes zoster infection of the geniculate ganglion may present with: (a) An upper motor neuron facial weakness. (False) (b) Diplopia. (False) (c) Hyperacusis. (True) (d) Altered perception of taste. (True) (e) Pain from the auditory meatus. (True) Question 25. A dissociated sensory loss may be seen in: (a) Syringomyelia. (True) (b) Anterior spinal artery occlusion. (False) (c) A radiculopathy. (False) (d) Occlusion of a middle cerebral artery. (False) (e) Compression of the spinal cord by a prolapsed intervertebral disc. (False) Module 7 (Gastroeneterology) Question 1. The following statements are true: (a) Tylosis is associated with achalasia. (False) (b) On barium swallow, a 'bird's beak' appearance is suggestive of squamous carcinoma. (False) (c) Pneumatic dilatation is the treatment of choice for achalasia. (True) (d) Reduced lower oesophageal sphincter pressure is a common feature of gastro-oesophageal reflux disease . (True) (e) Oesophageal pH is usually less than 4. (False) Question 2. The following is true of Barrett's oesophagus: (a) Columnar epithelium is replaced by squamous epithelium. (False) (b) It appears in an antegrade (top to bottom) direction. (False) (c) It is a premalignant condition. (True) (d) Severe dysplasia is an ominous sign. (True)

By A. H.

MCQs VIA WEB 2005 (e) It is an indication for surveillance endoscopy. (True)

Question 3. Helicobacter pylori: (a) Causes ulceration in the duodenum. (True) (b) Causes Barrett's metaplasia in the oesophagus. (False) (c) Is associated with hypergastrinaemia. (True) (d) Is often resistant to certain antibiotics. (True) (e) Can convert urea to ammonia and carbon dioxide. (True) Question 4. Gastric hypomotility (gastroparesis): (a) Is commonly associated with diabetes mellitus. (True) (b) Is a risk factor for gastro-oesophageal reflux disease. (True) (c) Is a feature of generalized scleroderma (systemic sclerosis). (True) (d) Occasionally responds to erythromycin. (True) (e) Is often secondary to duodenal ulcer disease. (False) Question 5. The following are features of coeliac disease: (a) Hypocalcaemia. (True) (b) Hypercalcaemia. (False) (c) Normocytic anaemia. (False) (d) Hypoalbuminaemia. (True) (e) Positive antiparietal cell antibodies. (False) Question 6. The following is true of Crohn's disease: (a) The rectum is always affected. (False) (b) Commonly affects the terminal ileum. (True) (c) More commonly occurs in smokers. (True) (d) Can result in vitamin B12 deficiency with a negative Schilling test. (True) (e) Commonly presents with bloody diarrhoea. (False) Question 7. The following is true of giardiasis: (a) Diarrhoea abates with avoidance of dairy produce. (False) (b) Diarrhoea abates with avoidance of gluten. (False) (c) Diarrhoea requires treatment with metronidazole. (True) (d) Diarrhoea is usually accompanied by vomiting. (False) (e) Diarrhoea commonly results in vitamin B12 deficiency. (False) Question 8. The following is true of inflammatory bowel disease: (a) Increased liver enzymes in the serum usually indicate the complication of carcinoma. (False) (b) Small bowel barium enema is the best radiological investigation for ulcerative colitis. (False) (c) It is occasionally complicated by carcinoma of the caecum. (True) (d) It is commonly associated with thyroiditis. (False) (e) It is sometimes complicated by iritis. (True) Question 9. The following is true of viral hepatitis: (a) Hepatitis C commonly presents with jaundice. (False) (b) Hepatitis E is fatal particularly in pregnant women. (True) (c) Hepatitis BeAg is a marker of viral replication. (True) (d) Hepatitis A is a risk factor for hepatoma. (False) (e) Hepatitis D occurs only in association with hepatitis C. (False) Question 10. The following drugs cause jaundice: (a) Methotrexate. (False) (b) Flucloxacillin. (True) (c) Metronidazole. (False) (d) Isoniazid. (True)

By A. H.

MCQs VIA WEB 2005 (e) Phenobarbitone. (False)

Question 11. Haemochromatosis: (a) Is a genetic defect resulting in copper overload in the liver. (False) (b) Is a risk factor for the development of hepatoma. (True) (c) Has an equal sex incidence but presents earlier in males than females. (True) (d) Is treated by avoiding meat products. (False) (e) Can cause hypogonadism in the absence of cirrhosis. (True) Question 12. Colonic carcinoma: (a) Most commonly occurs in the right side of the colon. (False) (b) May present with iron deficiency anaemia in the absence of any gastrointestinal symptoms. (True) (c) Commonly arises in colonic polyps. (True) (d) Carries a 5-year survival of less than 10%. (False) (e) Is the cause of carcinoid syndrome. (False) Question 13. The following is true of colon polyps and colon cancer: (a) The larger the polyp, the greater the risk of carcinoma. (True) (b) Malignant polyps can be successfully treated by colonoscopy and polypectomy alone. (True) (c) Hyperplastic polyps have a higher malignant potential than villous polyps. (False) (d) Polyps are most common in the ascending colon. (False) (e) Colonic polyps are often recurrent. (True) Question 14. The following gastrointestinal diseases are associated with the renal conditions listed: (a) Crohn's disease and renal amyloidosis. (True) (b) Hepatitis B and glomerulonephritis. (True) (c) Gastric ulcer and nephrotic syndrome. (False) (d) Pancreatic neuroendocrine tumours and polycystic kidney disease. (False) (e) Liver cysts and glomerulosclerosis. (False) Question 15. The following is true of villous atrophy in the small intestine: (a) If due to coeliac disease, it should recover completely on a gluten-free diet. (True) (b) It can be caused by tuberculosis. (True) (c) It can be associated with Giardi lamblia. (True) (d) It can be associated with Tropheryma whippelei. (True) (e) When associated with bacteria, it may cause a rise in serum folate. (True Question 16. The following skin conditions are associated with the named GI diseases: (a) Dermatitis herpetiformis with coeliac disease. (True) (b) Pruritus with primary biliary cirrhosis. (True) (c) Pyoderma gangrenosum with gastric carcinoma. (False) (d) Bullous pemphigoid with pancreatitis. (False) (e) Erythema nodosum with Crohn's disease. (True) Question 17. The following statements are true in relation to vomiting: (a) Vomiting occurring 12 hours after a suspicious meal is indicative of Salmonella poisoning. (False) (b) Vomiting in association with headache is a feature of migraine. (True) (c) Vomiting associated with weight loss can be indicative of malignant disease. (True) (d) Vomiting usually precedes the pain of biliary colic. (False) (e) Vomiting can be a feature of myocardial infarction. (True) Question 18. Scleroderma can produce the gastrointestinal complications listed: (a) Diarrhoea due to bacterial overgrowth. (True) (b) Constipation due to gut hypomotility. (False) (c) Diarrhoea which is unresponsive to a gluten-free diet. (True) (d) Gastric ulcer due to chronic gastritis. (False)

By A. H.

MCQs VIA WEB 2005 (e) Dysphagia due to abnormal peristalsis in the oesophagus. (True)

Question 19. Chronic pancreatitis: (a) Is a cause of diabetes mellitus. (True) (b) Can result from alcohol ingestion in moderate amounts. (True) (c) May be hereditary in a minority of cases. (True) (d) Can be diagnosed by a raised serum amylase. (False) (e) Is a cause of pancreas divisum. (False) Question 20. The following is true of rectal bleeding: (a) In the absence of haemorrhoids, it is usually due to malignant disease. (False) (b) It occurs more commonly in Crohn's disease than in ulcerative colitis. (False) (c) If it occurs in a patient with ulcerative colitis, it usually indicates that carcinoma has developed. (False) (d) When it is due to diverticular disease, colectomy may be indicated to control it. (True) (e) It may be caused by ingestion of aspirin. (True) Question 21. The following are risk factors for gastric carcinoma: (a) Pernicious anaemia. (True) (b) Coeliac disease. (False) (c) Partial gastrectomy. (True) (d) Helicobacter pylori infection. (True) (e) Ménétrière's disease. (True) Question 22. The following statements are true: (a) Solitary rectal ulcers are commonly associated with Crohn's disease. (False) (b) Crypt abscesses are typical of coeliac disease. (False) (c) Fistula formation can be a feature of Whipple's disease. (False) (d) Anal fissure predisposes to faecal incontinence. (False) (e) Right iliac fossa pain is common with diverticular disease. (False) Question 23. The following are true of hepatitis: (a) Hepatitis B is spread via the faecal-oral route. (False) (b) A vaccine is available for hepatitis C. (False) (c) Incubation time for hepatitis A is approximately 2-3 weeks. (True) (d) Hepatitis B is an RNA virus. (False) (e) Interferon treatment is required for hepatitis E infection. (False) Question 24. The following is a risk factor for the Budd-Chiari syndrome: (a) Oral contraceptive pill. (True) (b) Malignancy. (True) (c) Ascites. (False) (d) Polycythaemia rubra vera. (True) (e) Constrictive pericarditis. (False) Question 25. The following are true regarding prognostic factors for acute pancreatitis: (a) A low pAO2 indicates a poor prognosis. (True) (b) A high serum GGT has a poor prognosis. (False) (c) Age of over 55 years usually has a good prognosis. (False) (d) A low serum albumin indicates a poor prognosis. (True) (e) Abnormal clotting time has a poor prognosis. (True) Module 9 (Gastroenterology) Question 1. The following is true of oesophageal pain: (a) It can occur in the absence of heartburn. (True) (b) It can mimic the pain of a myocardial infarction. (True) (c) It can be relieved by glyceryl trinitrate. (True)

By A. H.

MCQs VIA WEB 2005 (d) It is usually precipitated by exercise. (False) (e) It can be caused by candidiasis. (True) Question 2. The following is true of postgastrectomy syndromes: (a) The anaemia can be corrected with ascorbic acid supplements. (True) (b) The risk of gastric cancer in the long term is increased. (True) (c) Sweating and palpitations can be due to hypoglycaemia. (True) (d) Biliary gastritis in the gastric remnant is common. (True) (e) Diarrhoea is commonly due to bacterial overgrowth. (True) Question 3. The following is true of neoplastic disease in the stomach: (a) Maltoma can occasionally respond to antibiotic treatment in combination with a proton pump inhibitor. (True) (b) Ménétrière's disease is due to metaplasia of the gastric mucosa. (True) (c) Leiomyoma has a characteristic appearance at endoscopy. (True) (d) Gastric carcinoma produces abdominal pain that is often worse after eating. (True) (e) The most common gastric carcinoma is of squamous cell origin. (False) Question 4. The following statements are true: (a) Iron absorption is reduced in hypochlorhydric states. (True) (b) Vitamin D absorption is often deficient in the presence of gastritis. (False) (c) Vitamin B12 supplements are often necessary following gastrectomy. (True) (d) Anaemia associated with chronic atrophic gastritis may respond to ascorbic acid supplements. (True) (e) Intestinal metaplasia in the stomach is a risk factor for gastric carcinoma. (True) Question 5. The following clinical features are associated with coeliac disease: (a) Anaemia. (True) (b) Weight loss. (True) (c) Vomiting. (False) (d) Diarrhoea. (True) (e) Jaundice. (False) Question 6. The following is true of Crohn's disease: (a) C-reactive protein mimics inflammatory activity. (True) (b) Normal albumin indicates remission. (False) (c) Large bowel barium enema is the most definitive radiological test. (False) (d) A small bowel biopsy can be helpful in making the diagnosis. (True) (e) A low blood urea is common. (True) Question 7. The following is true of ulcerative colitis: (a) It commonly presents with pain in the right iliac fossa. (False) (b) It can be associated with ankylosing spondylitis. (True) (c) It is a risk factor for toxic dilatation of the colon. (True) (d) The occurrence of abdominal tenderness is an ominous sign. (True) (e) It often causes ischiorectal abscesses. (False) Question 8. Acholuric jaundice without pain: (a) Is a common presentation of pancreatic carcinoma. (False) (b) Is a feature of Gilbert's disease. (True) (c) Can occur in hereditary spherocytosis. (True) (d) Is associated with pale-coloured stools. (False) (e) Is associated with pruritus. (False) Question 9. The following is true of risk factors for the development of hepatocellular carcinoma: (a) Females are at greater risk than males. (False) (b) Excess iron is a recognized risk factor. (True) (c) Aflatoxin is a risk factor. (True) (d) Hepatitis A is a risk factor. (False)

By A. H.

MCQs VIA WEB 2005 (e) Risk factors generally only operate in the presence of cirrhosis. (True)

Question 10. Alcoholic hepatitis: (a) Recovers rapidly on cessation of drinking. (False) (b) Is a risk factor for hepatorenal syndrome. (True) (c) Ascites is a feature. (True) (d) Coagulopathy is corrected with administration of vitamin K. (False) (e) Encephalopathy occurs only if infection is present. (False) Question 11. Primary sclerosing cholangitis: (a) Occurs predominantly in middle-aged females. (False) (b) Is a major risk factor for cholangiocarcinoma. (True) (c) Occurs in 50% patients with ulcerative colitis. (False) (d) Has been treated with ursodeoxycholic acid. (True) (e) May require insertion of an endoprosthesis for its treatment. (True) Question 12. The following GI conditions are associated with microcytic hypochromic anaemia: (a) Acute duodenal ulceration. (False) (b) Ankylostoma duodenale. (True) (c) Terminal ileitis due to Crohn's disease. (True) (d) Partial gastrectomy. (True) (e) Carcinoma of caecum. (True) Question 13. These gastronomic terms are associated with the following gastroenterological conditions: (a) 'Rice water' diarrhoea with cholera. (True) (b) 'Anchovy sauce' discharge with amoebic dysentry. (False) (c) 'Redcurrent jelly' and intussusception. (True) (d) 'Apple core' lesion and diverticulitis. (False) (e) 'Coffee grounds' and oesophageal varices. (False) Question 14. The following is true of breath tests used for investigation of the gastrointestinal tract: (a) The 14C urea breath test detects Helicobacter pylori infection. (True) (b) The 14C glycocholic acid breath test is used to detect bacterial overgrowth in the colon. (False) (c) A hydrogen breath test following ingestion of lactulose is used to detect bacterial overgrowth in the small intestine. (True) (d) A lactose breath test is used to detect disaccharidase deficiency. (True) (e) A 14C bile salt test can be used to identify bile duct obstruction. (False) Question 15. The following autoantibodies are associated with the diseases listed: (a) Antiendomyseal antibodies are associated with coeliac disease. (True) (b) Anti-LKM antibodies are associated with Goodpasture syndrome. (False) (c) Antimitochondrial antibodies are associated with primary biliary cirrhosis. (True) (d) Antiparietal cell antibodies are associated with Wilson's disease. (False) (e) Antismooth muscle antibodies are associated with autoimmune chronic active hepatitis. (True) Question 16. The following statements are true of colitis: (a) Granulomas are present in collagenous colitis. (False) (b) Rectal sparing is characteristic of Crohn's colitis. (True) (c) Caseating granulomas in the terminal ileum are diagnostic of Crohn's disease. (False) (d) Colitis in a smoker is more likely to be Crohn's than ulcerative colitis. (True) (e) Pain is a characteristic feature of CMV colitis. (True) Question 17. The following statements are true of ascites: (a) A high protein content in ascites is usual in alcoholic liver disease. (False) (b) Ascites resistant to diuretics is characteristic of hepatic vein thrombosis. (True) (c) Ascites is sometimes associated with a pleural effusion. (True)

By A. H.

MCQs VIA WEB 2005 (d) Ascites is a risk factor for bacterial peritonitis. (True) (e) Ascites due to constrictive pericarditis prevents pulsus paradoxus. (False) Question 18. The following statements are true of non-steroidal anti-inflammatory drugs: (a) They can be given as suppositories to avoid gastrointestinal complications. (False) (b) They may have a role in the prevention of colon cancer. (True) (c) They can produce gastric erosions in elderly people causing occult blood loss. (True) (d) They cause gastric erosions by stimulating gastric acid secretion. (False) (e) They may exacerbate long-standing ulcerative colitis. (True) Question 19. The following is true of pancreatic tumours: (a) Jaundice occurs only when carcinoma is present in the tail of pancreas. (False) (b) Presence of diabetes mellitus indicates that the tumour is of neuroendocrine origin. (False) (c) They are generally unresponsive to chemotherapy. (True) (d) They characteristically produce back pain when local invasion is present. (True) (e) They occur with increased frequency in patients with ulcerative colitis. (False) Question 20. The following is true of haematemesis: (a) When it occurs in a patient with alcoholic liver disease, it is always due to oesophageal varices. (False) (b) A visible vessel seen at gastroscopy is a risk factor for further bleeding. (True) (c) When it occurs in patients over 70 years of age who may have arthritis, usually indicates malignancy. (False) (d) When it occurs after repeated retching, it is suggestive of an oesophageal tear. (True) (e) When it is caused by duodenal ulcer, a partial gastrectomy is usually necessary. (False) Question 21. The following drugs can be used for treatment of GORD: (a) Metronidazole. (False) (b) Amoxycillin. (False) (c) Erythromycin. (False) (d) Metoclopramide. (True) (e) Omeprazole. (True) Question 22. Which of the following is dependent on bile salts for its absorption: (a) Vitamin A. (True) (b) Vitamin B. (False) (c) Vitamin C. (False) (d) Vitamin D. (True) (e) Vitamin K. (True) Question 23. The following are indications for liver biopsy: (a) Unexplained abnormal liver enzymes. (True) (b) Pyrexia of unknown origin with normal liver enzymes. (True) (c) Cirrhosis suspected on an ultrasound scan. (True) (d) Raised alkaline phosphatase in teenagers with acholuric jaundice. (False) (e) Abnormal liver enzymes in a patient with epilepsy on phenytoin. (False) Question 24. The following precipitate portasystemic encephalopathy: (a) Infection. (True) (b) Diarrhoea. (False) (c) Gastrointestinal bleeding. (True) (d) Use of opioid drugs. (True) (e) Certain antibiotics. (False) Question 25. The following drugs cause cholestatic jaundice: (a) Rifampicin. (False) (b) Isoniazid. (False) (c) Erythromycin. (True) (d) Halothane. (False)

By A. H.

MCQs VIA WEB 2005 (e) Paracetamol. (False)

Module 10 (Neurology) Question 1. The following cranial nerves carry parasympathetic fibres: (a) Oculomotor. (True) (b) Trigeminal. (False) (c) Facial. (True) (d) Hypoglossal. (False) (e) Vagus. (True) Question 2. Myasthenia gravis: (a) Is caused by antibodies to the acetylcholine receptor in the majority of cases. (True) (b) Causes muscle wasting. (False) (c) May show diurnal variation in symptoms. (True) (d) Is associated with an improvement in strength after exertion. (False) (e) May present with ophthalmoplegia. (True) Question 3. The causes of a mixed upper and lower motor neuron picture include: (a) Guillain-Barré syndrome. (False) (b) Multiple sclerosis. (False) (c) Syringomyelia. (True) (d) Motor neuron disease. (True) (e) Taboparesis. (True) Question 4. Bilateral lower motor neuron facial weakness may occur in: (a) Sarcoidosis. (True) (b) Guillain-Barré syndrome. (True) (c) Lyme disease. (True) (d) Lymphoma. (True) (e) Parasagittal meningioma. (False) Question 5. In idiopathic Parkinson's disease: (a) There is degeneration primarily of the cells of the globus pallidus. (False) (b) The classical features include tremor, bradykinesia, and spasticity. (False) (c) There is an associated vertical gaze palsy. (False) (d) Anticholinergic drugs are most effective in relieving tremor. (True) (e) Treatment is aimed at reducing dopamine levels. (False) Question 6. The following features suggest that increased tone is due to rigidity: (a) Tone is increased equally in flexors and extensors. (True) (b) Extensor plantar responses. (False) (c) Associated pill-rolling tremor. (True) (d) Clasp-knife reflex. (False) (e) Tone increases with synkinesis. (True) Question 7. Causes of a small pupil include: (a) Horner's syndrome. (True) (b) Holmes-Adie syndrome. (False) (c) Tabes dorsalis. (True) (d) Optic neuritis. (False) (e) Pilocarpine eye-drops. (True) Question 8. Concerning optic neuritis: (a) Visual loss is usually painless. (False) (b) White-matter abnormalities on MR imaging increase the likelihood of developing multiple sclerosis in the future. (True)

By A. H.

MCQs VIA WEB 2005 (c) After recovery, some impairment of red-green colour vision may remain. (True) (d) Over 90% of patients with a history of optic neuritis go on to develop multiple sclerosis. (False) (e) It causes a delay in visual evoked potentials. (True) Question 9. The following may cause a third nerve palsy: (a) Aneurysm of the posterior communicating artery. (True) (b) Diabetes. (True) (c) Motor neuron disease. (False) (d) Herniation of the uncus of the temporal lobe. (True) (e) Pancoast tumour. (False) Question 10. The following typically occur within the first 24 hours of complete cervical cord transection: (a) Upgoing plantar responses. (False) (b) Fall in blood pressure. (True) (c) Loss of bladder control. (True) (d) Brisk reflexes. (False) (e) Gastric dilatation. (True) Question 11. In motor neuron disease: (a) Fasciculations are required to make the diagnosis. (False) (b) There may be atrophy of the Betz cells in the motor cortex. (True) (c) Electromyography shows chronic partial denervation. (True) (d) There should be no signs of sensory loss. (True) (e) Familial cases account for 50%. (False) Question 12. Causes of a mononeuropathy include: (a) Diabetes. (True) (b) Hereditary motor sensory neuropathy. (False) (c) Polyarteritis nodosa. (True) (d) Guillain-Barré syndrome. (False) (e) Lead poisoning. (True) Question 13. Charcot joints: (a) May affect the feet in diabetes. (True) (b) Are often painful. (False) (c) May be caused by neurosyphilis. (True) (d) May affect the shoulders in syringomyelia. (True) (e) Are usually hot and swollen. (False) Question 14. Hyposmia may arise secondary to: (a) A head injury. (True) (b) Migraine. (False) (c) Seizures. (False) (d) Antibiotic therapy. (True) (e) A frontal meningioma. (True) Question 15. The following are causes of acute transient visual impairment: (a) Retinitis pigmentosa. (False) (b) Amaurosis fugax. (True) (c) Papilloedema. (True) (d) Migrainous aura. (True) (e) Glaucoma. (False) Question 16. The following may be features of frontal lobe dysfunction: (a) Depression. (True) (b) Social disinhibition. (True) (c) Apraxia of gait. (True)

By A. H.

MCQs VIA WEB 2005 (d) A receptive dysphasia. (False) (e) A grasp reflex. (True) Question 17. The following may give rise to a pseudobulbar palsy (a) Poliomyelitis. (False) (b) Syringobulbia. (False) (c) Huntington's chorea. (False) (d) Occlusion of the anterior cerebral artery. (False) (e) Multiple sclerosis. (True) Question 18. Facial sensory loss may occur with a lesion of: (a) The cerebellopontine angle. (True) (b) The facial nerve. (False) (c) The Gausserian ganglion. (True) (d) The Geniculate ganglion. (False) (e) The cavernous sinus. (True) Question 19. Sensorineural deafness may occur secondary to: (a) Loud noise. (True) (b) Gentamicin therapy. (True) (c) Ménière's disease. (True) (d) An acoustic neuroma. (True) (e) Otosclerosis. (False) Question 20. Choreic movements are: (a) Slow and writhing. (False) (b) Shock-like assymetrical and irregular. (False) (c) Brief, jerky and irregular. (True) (d) A sign of restlessness. (False) (e) Rhythmical and oscillatory. (False) Question 21. Features of an upper motor neuron lesion are: (a) Brisk abdominal and cremasteric reflexes. (False) (b) Wasted muscles. (False) (c) Weakness of individual muscles. (False) (d) Hypotonia. (False) (e) Fatiguable muscle strength. (False) Question 22. A small pupil may be seen in: (a) A lesion in the midbrain. (False) (b) Elderly patients. (True) (c) Horner's syndrome. (True) (d) Terminally ill patients taking morphine for analgesia. (True) (e) A pontine lesion. (True) Question 23. Nystagmus may be seen in: (a) A patient with an internuclear ophthalmoplegia. (True) (b) A lesion of the pons. (True) (c) A patient who is blind. (True) (d) A patient with cerebellar dysfunction. (True) (e) A lesion of the foramen magnum. (True) Question 24. Clinical features of a unilateral lesion of the cerebellopontine angle may be: (a) Conductive deafness on the same side. (False) (b) An ipsilateral hemiparesis. (False) (c) Ipsilateral weakness of the lower face. (False) (d) A pseudobulbar dysarthria. (False)

By A. H.

MCQs VIA WEB 2005 (e) Vertigo as a prominent early symptom. (False)

Question 25. The fibres of the dorsal column pathway: (a) Carry information about temperature perception. (False) (b) Decussate in the midbrain. (False) (c) Are affected in the deficiency of vitamin B12. (True) (d) When damaged may result in a positive Romberg's test. (True) (e) Are spared following occlusion of the anterior spinal artery. (True) Available from Master Medicine Module 1 (trial1) Question 1. The ECG: · The T wave corresponds to atrial contraction (False) · If the S wave is greater than the R wave in lead I, there is right axis deviation (True) · If the S wave is greater than the R wave in lead II, there is left axis deviation (True) · ST segment depression may be a sign of cardiac ischaemia (True) Explanation: It may also be a digoxin effect. · A tall R wave in V1 may be a sign of right ventricular hypertrophy (True) Explanation: It may also be a digoxin effect. Question 2. Endocarditis: · It is important to take blood cultures over at least 24 hour period to make the diagnosis (False) · Transthoracic echocardiography is a sensitive means of making or confirming the diagnosis (False) · Most patients with Staphylococcus aureus bacteraemia have endocarditis (False) · Viral endocarditis leads to valvular abnormality (False) · In patients with a new stroke, endocarditis can be ruled out if the patient is afebrile (False) Question 3. Treatment of endocarditis: · Intravenous antibiotics for 6 weeks are necessary to cure viridans type streptococcal endocarditis (False) · Staphylococcal endocarditis on the tricuspid valve in a drug addict is treated with flucloxacillin and valve replacement (False) · Large vegetations are an indication for surgery (True) Explanation: Flucloxacillin (with gentamicin or rifampicin) is the medical treatment of choice but valve replacement is not appropriate. Insertion of a prosthetic heart valve into a drug addict is very likely to lead to prosthetic valve endocarditis subsequently because of their continuing habit. · Combination antibiotic therapy is almost always appropriate for endocarditis (True) Explanation: For two reasons; first, the selected combinations are usually additive or synergistic. Second, to prevent the development of resistance. · If gentamicin is used for treatment, it should not be used for more than 2 weeks (False) Question 4. Hypertension: · Treatment is of no proven benefit in patients over the age of 70 years (False) · The symptoms of phaeochromocytoma include headache, sweating and palpitations (True) Explanation: There is well-proven benefit, particularly in the prevention of stroke. · Oral treatment producing a fall in diastolic blood pressure of 20 mmHg over 24 hours might be regarded as successful treatment of accelerated hypertension (True) Explanation: There is well-proven benefit, particularly in the prevention of stroke. · ACE inhibitors are the drugs of choice for hypertension in pregnancy (False)

By A. H.

MCQs VIA WEB 2005 ·

Addison's disease should be considered a possible cause in a hypertensive patient with hirsutism (False)

Question 5. Cardiac dysrhythmias: · Digoxin toxicity may cause supraventricular tachycardia (True) Explanation: Typically, paroxysmal atrial tachycardia. · A patient with a completely irregular pulse of 180 beats/min is likely to be in atrial fibrillation (True) Explanation: Typically, paroxysmal atrial tachycardia. · Complete heart block may be asymptomatic (True) Explanation: Particularly congenital complete heart block. · Digoxin is effective in preventing paroxysms of atrial fibrillation (False) · A QRS width less than 3 small squares on the ECG indicates that a tachycardia is supraventricular (True) Explanation: Digoxin slows the ventricular rate during paroxysms of atrial fibrillation but does not prevent them; sotalol or amiodarone may prevent them. Question 6. Chronic bronchial sepsis: · Is an uncommon feature of cystic fibrosis (False) · Typically is caused by unusual, difficult-to-grow bacteria (False) · May lead to haemoptysis (True) Explanation: Haemoptysis is also seen with dry bronchiectasis, chronic bronchial sepsis and with aspergillomas. · Can usually be cured with oral antibiotics (False) · May lead to pulmonary fibrosis (True) Explanation: It produces a fibrotic reaction. Question 7. In the small intestine: · If there is bile salt deficiency, micellar formation is reduced (True) Explanation: Bile salts are essential for micelle formation. · Long-chain triglycerides are transported from the gut in the lymph as chylomicrons (False) · There is no lymphatic tissue (False) · The entire mucosa is turned over every 2-3 weeks (False) · Is the site of most nutrient absorption (True) Question 8. Colorectal cancer: · May arise from a metaplastic polyp (False) · Most often occurs in the rectum and sigmoid (True) Explanation: Metaplastic polyps have no malignant potential. · There are further polyps in most cases (False) · Involvement of local lymph nodes does not affect prognosis (False) · Obstruction is more common in right compared with left-sided lesions (False) Question 9. Angiodysplasia of the colon: · Is more common in the caecum and ascending colon (True) Explanation: It usually occurs in the right side of the colon. · Is associated with a macrocytic anaemia (False) · Is best shown by barium enema (False) · Usually requires surgery (False) · Is a congenital lesion (False) Question 10. Concerning HIV infection and AIDS: · Pneumocystis pneumonia is common in Africa (False) · Tuberculosis in AIDS presents like that in non-AIDS patients (False) · Oral candidiasis is a late feature of AIDS (False) · Toxoplasmosis is usually a cerebral disease (True)

By A. H.

MCQs VIA WEB 2005 Explanation: Brain and heart. The CT/MR scan usually shows multiple ring-enhancing lesions, which are almost diagnostic of toxoplasmosis in AIDS. CNS lymphomas are usually single. Cardiac toxoplasmosis is usually diagnosed at postmortem. · Cytomegalovirus retinitis can be treated with aciclovir (False) Module 1 (trial2) Question 1. In secondary diabetes: · A patient can be assumed not to be ketosis-prone (False) · A patient is more than 85% likely to have clinical pancreatic exocrine deficiency (False) · Classical diabetic complications do not occur (False) · Thiazide diuretics and beta-blockers can both impair insulin secretion (True) Explanation: Secondary diabetes causes all the same complications as idiopathic diabetes. · Most patients with acromegaly are diabetic (False) Question 2. In hypoglycaemia: · Insulin-dependent patients may recover from hypoglycaemic coma without treatment (True) Explanation: The anti-insulin hormones can bring the patient round and the insulin which caused the coma can 'wear off'. · Sweating and shaking are always late symptoms of insulin-induced hypoglycaemia (False) · Insulin-dependent patients may lose their warning symptoms of hypoglycaemia after many years of diabetes (True) Explanation: About 50% of patients who have had type 1 DM for 20 years or more develop 'hypoglycaemia unawareness'. · Metformin is responsible for as many cases of hypoglycaemia as sulfonylureas (False) · The symptoms characteristically come on over hours rather than minutes (False) Question 3. In insulin treatment: · Pen injectors are reserved for the small minority who take four or more injections per day (False) · Only patients who cannot be controlled with once-daily insulin should have two or more injections (False) · Insulin should be started without delay in a thin hyperglycaemic patient with ketonuria (True) Explanation: These are signs of type 1 DM. · Insulin may sometimes be needed during short periods of illness in patients with type 2 DM (True) Explanation: These are signs of type 1 DM. · All patients on insulin should be discouraged from changing their doses without first checking with the doctor or nurse (False) Question 4. Diabetic pregnancy: · Insulin-dependent women should be advised not to contemplate pregnancy (False) · Diabetes increases the risk of neural tube defects (True) Explanation: Neural tube defects are two to three times more common. · Poor glycaemic control at conception increases the risk of congenital malformations (True) Explanation: Hyperglycaemia is teratogenic in early pregnancy; major congenital malformations are two to three times more common. · There is a less than 10% chance that an episode of ketoacidosis will cause intrauterine death (False) · Sulfonylureas are the treatment of choice for gestational diabetes (False) Question 5. Thyroid function tests: · Serum thyroid-stimulating hormone (TSH) is a sensitive test of hyperthyroidism (True) Explanation: Suppression of TSH is the first biochemical sign of hyperthyroidism. · Serum TSH can distinguish primary from secondary hypothyroidism (True) Explanation: In primary hypothyroidism, TSH is high; in secondary hypothyroidism, it is low. · Serum triiodothyronine can be an unreliable test for hypothyroidism (True) Explanation: Low triiodothyronine may result from intercurrent illness, particularly in elderly people, and can be misleading. · Hyperthyroid patients may have a raised serum triiodothyronine with a normal thyroxine (True)

By A. H.

MCQs VIA WEB 2005 Explanation: The condition of 'T3 toxicosis'.

Question 6. Rheumatoid factor is: · An antibody to sheep erythrocytes (False) · Present when rheumatoid nodules are present (True) Explanation: Nodules are associated with high titres of rheumatoid factor. · Diagnostic of rheumatoid arthritis (False) · Usually is of the IgA subtype (False) · Is not found in rheumatoid synovial, pleural or pericardial fluid (False) Question 7. In gout: · Tophi are an early sign (False) · Allopurinol is used to treat the acute attack (False) · Furosemide (frusemide) helps to increase urate excretion (False) · Large joints are not affected (False) · Raised serum urate makes the diagnosis certain (False) Question 8. Ankylosing spondylitis: · Is more common in females (False) · May present as a severe oligoarthritis (True) Explanation: Commonly affects several joints and often presents with back pain. · Is associated with the histocompatibility antigen HLA-DW3 (False) · Is associated with pulmonary fibrosis (True) Explanation: It is associated with upper lobe fibrosis and aortic incompetence. · Involves the proximal interphalangeal (PIP) (False) Question 9. Concerning osteomyelitis: · Debridement of infected bone is essential for cure in chronic bacterial osteomyelitis (True) Explanation: It is often difficult to remove all dead infected bone. · It is usually accompanied by a very high ESR (True) Explanation: Virtually always and it is a useful marker of response to treatment and relapse. · A distinctive feature of chronic osteomyelitis is a discharging sinus (True) Explanation: Although there are other causes of a sinus including actinomycosis, implanted foreign body (such as shrapnel), mycetoma (fungal soft tissue and bony infection of the leg in the tropics). · A positive culture from a sinus track is a good indication of the bacterial cause of the chronic osteomyelitis (False) · Usually 2 or 3 weeks' antibiotic therapy is adequate for cure (False) Question 10. With regard to reactive arthritis: · It may be caused by both Salmonella and Campylobacter spp. (True) Explanation: It usually occurs 3-12 weeks after the episode of diarrhoea. · It is usually chronic and unremitting over 3-4 years (False) · Confidence in the diagnosis rests on growing a bacterium from stool or other sites (False) · NSAIDs are appropriate therapy (True) Explanation: It is helpful if it can be done but failure does not rule out the diagnosis. · Rheumatic fever should be excluded (True) Module 1 (Master Medicine) Question 1. The anatomy of the heart: If you stand on the patient's right side with your right hand across the sternum and cardiac apex, the left ventricle lies under the sternum (False) On a postero-anterior (PA) chest radiograph, the left heart border is mostly formed by the left ventricle (True) Explanation: The right ventricle presses against the sternum; the left ventricle constitutes the apex and is felt under the fingers. In an ECG, disease of the interventricular septum causes changes in chest leads V3-4 (True)

By A. H.

MCQs VIA WEB 2005 Explanation: The right ventricle presses against the sternum; the left ventricle constitutes the apex and is felt under the fingers. When examining the heart, the cardiac apex is the point where the heart beat can be felt most strongly (False) Occlusion of the left anterior descending coronary artery causes infarction of the anterior wall of the left ventricle and interventricular septum (True) Explanation: Anterior myocardial infarction is caused by disease of the left anterior descending artery. Question 2. The ECG: The T wave corresponds to atrial contraction (False) If the S wave is greater than the R wave in lead I, there is right axis deviation (True) If the S wave is greater than the R wave in lead II, there is left axis deviation (True) ST segment depression may be a sign of cardiac ischaemia (True) Explanation: It may also be a digoxin effect. A tall R wave in V1 may be a sign of right ventricular hypertrophy (True) Explanation: It may also be a digoxin effect Question 3. Endocarditis: It is important to take blood cultures over at least 24 hour period to make the diagnosis (False) Transthoracic echocardiography is a sensitive means of making or confirming the diagnosis (False) Most patients with Staphylococcus aureus bacteraemia have endocarditis (False) Viral endocarditis leads to valvular abnormality (False) In patients with a new stroke, endocarditis can be ruled out if the patient is afebrile (False) Question 4. Treatment of endocarditis: Intravenous antibiotics for 6 weeks are necessary to cure viridans type streptococcal endocarditis (False) Staphylococcal endocarditis on the tricuspid valve in a drug addict is treated with flucloxacillin and valve replacement (False) Large vegetations are an indication for surgery (True) Explanation: Flucloxacillin (with gentamicin or rifampicin) is the medical treatment of choice but valve replacement is not appropriate. Insertion of a prosthetic heart valve into a drug addict is very likely to lead to prosthetic valve endocarditis subsequently because of their continuing habit. Combination antibiotic therapy is almost always appropriate for endocarditis (True) Explanation: For two reasons; first, the selected combinations are usually additive or synergistic. Second, to prevent the development of resistance. If gentamicin is used for treatment, it should not be used for more than 2 weeks (False) Question 5. In acute myocardial infarction: The diagnosis should be questioned if the jugular venous pressure is not raised (False) Streptokinase should not be given until the diagnosis has been confirmed by two sets of raised cardiac enzymes (False) Dysrhythmias in the early hours after presentation carry a poor prognosis (False) Lidocaine should routinely be given to prevent dysrhythmias (False) Rupture of the interventricular septum is an uncommon but serious complication (True) Explanation: This is of no proven value. Question 6. In acute dissection of the thoracic aorta: The operative mortality is about 30% (False) Spinal cord ischaemia may occur (True) Explanation: It is much higher. Hypertension should be treated aggressively (True) Explanation: Nitroprusside or labetolol infusion is a recommended treatment. Acute aortic stenosis may occur (False) The patient may develop myocardial ischaemia (True) Explanation: The coronary ostia may be occluded by the dissection Question 7. Hypertension: Treatment is of no proven benefit in patients over the age of 70 years (False) The symptoms of phaeochromocytoma include headache, sweating and palpitations (True)

By A. H.

MCQs VIA WEB 2005 Explanation: There is well-proven benefit, particularly in the prevention of stroke. Oral treatment producing a fall in diastolic blood pressure of 20 mmHg over 24 hours might be regarded as successful treatment of accelerated hypertension (True) Explanation: There is well-proven benefit, particularly in the prevention of stroke. ACE inhibitors are the drugs of choice for hypertension in pregnancy (False) Addison's disease should be considered a possible cause in a hypertensive patient with hirsutism (False) Question 8. In ischaemic heart disease: Prevalence is increased in chronic renal failure (True) Explanation: Cushing's syndrome, not Addison's disease. Untreated hypothyroidism predisposes to it (True) Explanation: Hypothyroidism causes hypercholesterolaemia and atherosclerosis. Polycythaemia may precipitate myocardial ischaemia (True) Explanation: By increasing blood viscosity and impairing blood flow. An alcohol intake of 18 units per week in a man increases the risk of ischaemic heart disease (False) A high plasma fibrinogen reduces the risk (False) Question 9. Cardiac dysrhythmias: Digoxin toxicity may cause supraventricular tachycardia (True) Explanation: Typically, paroxysmal atrial tachycardia. A patient with a completely irregular pulse of 180 beats/min is likely to be in atrial fibrillation (True) Explanation: Typically, paroxysmal atrial tachycardia. Complete heart block may be asymptomatic (True) Explanation: Particularly congenital complete heart block. Digoxin is effective in preventing paroxysms of atrial fibrillation (False) A QRS width less than 3 small squares on the ECG indicates that a tachycardia is supraventricular (True) Explanation: Digoxin slows the ventricular rate during paroxysms of atrial fibrillation but does not prevent them; sotalol or amiodarone may prevent them. Question 10. Hypoventilation occurs in the following: Central sleep apnoea syndrome (True) Explanation: Alveolar hypoventilation is a key feature. Severe kyphoscoliosis (True) Explanation: Severe kyphoscoliosis can produce mechanical ventilation problems because of the changed curvature of the spine. Anxiety (False) Benzodiazepine overdose (True) Explanation: Drugs such as benzodiazepines depress the respiratory centre. Exercise (False) Question 11. Pneumothorax is a recognised complication of: Rib fracture (True) Explanation: Pneumothorax can occur secondary to trauma. A bulla (True) Explanation: Any cavitating or cystic/bullous lung lesion can cause a pneumothorax. Bullae can be single or multiple. They are particularly common in emphysema including á1-antitrypsin deficiency. Kyphoscoliosis (False) Cystic fibrosis (False) Pneumocystis carinii pneumonia (True) Explanation: And lung abscesses (e.g. Staph. aureus) can lead to pneumothorax. Question 12. The following are features of fibrosing alveolitis: Cough (True) Explanation: Patients usually present with cough and breathlessness. Clubbing of the fingers in the majority of cases (True) Explanation: Clubbing occurs in about 60% of patients but is not essential for the diagnosis. Cyanosis in the early stages (False)

By A. H.

MCQs VIA WEB 2005 Circulating antibodies to alveolar tissues (False) Haemoptysis (False) Question 13. Useful drugs for tuberculosis include: Piperacillin (False) Isoniazid (True) Explanation: Isoniazid is a major, first-line agent. Ciprofloxacin (True) Explanation: Ciprofloxacin is a useful agent, less active than rifampicin; it may obscure infection in patients treated before diagnosis considered. Ethambutol (True) Explanation: Ethambutol is another major, but second-line agent. Amikacin (True) Explanation: Amikacin is a useful i.v. second-line agent Question 14. Causes of life-threatening pneumonia or pneumonitis in adults include: Pneumocystis carinii (True) Explanation: Pneumocystis carinii infection is usually seen in AIDS, but also in lymphoma, steroid-treated, transplant and hypogammaglobulinaemic patients. Influenza A virus (True) Explanation: Primary influenzal pneumonia or complicated by bacteria, e.g. Staph. aureus. Respiratory syncytial virus (False) Staphylococcus aureus (True) Explanation: S. aureus pneumonia is often rapidly fatal, especially following influenza. Legionella pneumophila (True) Explanation: L. pneumophilia pneumonia carries a high mortality if not treated appropriately. Question 15. Chronic bronchial sepsis: Is an uncommon feature of cystic fibrosis (False) Typically is caused by unusual, difficult-to-grow bacteria (False) May lead to haemoptysis (True) Explanation: Haemoptysis is also seen with dry bronchiectasis, chronic bronchial sepsis and with aspergillomas. Can usually be cured with oral antibiotics (False) May lead to pulmonary fibrosis (True) Explanation: It produces a fibrotic reaction Question 16. Pleural aspiration is useful in the following situations: In diagnosing mesothelioma (False) Pleural tuberculosis (False) Viral pleurisy (False) Empyema (True) Explanation: An empyema will require tube or surgical drainage for treatment. Relieving breathlessness in patients with malignant effusions (True) Explanation: Drainage in malignant effusions is often very helpful if litres of fluid are removed or a shunt can be inserted. Question 17. In the small intestine: If there is bile salt deficiency, micellar formation is reduced (True) Explanation: Bile salts are essential for micelle formation. Long-chain triglycerides are transported from the gut in the lymph as chylomicrons (False) There is no lymphatic tissue (False) The entire mucosa is turned over every 2-3 weeks (False) Is the site of most nutrient absorption (True) Explanation: The small intestine is the main area for the breakdown and absorption of nutrients. Question 18. Colorectal cancer: May arise from a metaplastic polyp (False)

By A. H.

MCQs VIA WEB 2005 Most often occurs in the rectum and sigmoid (True) Explanation: Metaplastic polyps have no malignant potential. There are further polyps in most cases (False) Involvement of local lymph nodes does not affect prognosis (False) Obstruction is more common in right compared with left-sided lesions (False) Question 19. Causes of acute pancreatitis include: Alcohol (True) Explanation: Most cases are associated with gall stones or high alcohol intake. Hypocalcaemia (False) Hyperlipidaemia (True) Explanation: There is an association with hyperlipidaemia, but it is an uncommon cause. Self poisoning with diazepam (False) Endoscopic retrograde cholangiopancreatography (ERCP) (True) Explanation: ERCP is used in the diagnosis of pancreatic disease but can precipitate an acute attack. Question 20. Coeliac disease: The patient will almost always have had symptoms since childhood (False) Is best diagnosed on colonic biopsy (False) Is associated with HLA-B8 (True) Explanation: It is associated with HLA-B8 and HLA-DRW3 antigens. The diagnosis is incorrect if a patient fails to respond to a gluten-free diet (False) Requires a diet free from wheat, barley and rye (True) Explanation: All contain gluten Question 21. In a ward with several patients where two of the nurses have had much vomiting and some diarrhoea over a 48-hour period, you should: Send the patients home (False) Culture stools (and vomitus) for viruses (False) Treat everyone with metronidazole (False) Exclude visitors from the ward (True) Explanation: To prevent further spread, unless necessary for, say, a dying patient. Prevent the patients (affected or not) leaving the ward for investigations, physiotherapy, etc. (True) Explanation: Unless the investigation was absolutely vital Question 22. The differential diagnosis of acute bloody diarrhoea includes: Amoebic dysentery (True) Explanation: This has much mucus and tenesmus. Campylobacter enteritis (True) Explanation: The amount of blood is usually small. Haemorrhagic colitis caused by E. coli (True) Explanation: The classic cause, with mostly blood and little stool and no fever. Traveller's diarrhoea (False) Cholera (False) Question 23. The following are correct: Hepatitis B can be acquired from serous fluid from a wound (True) Explanation: This is the likely mode of horizontal transmission among siblings in developing countries. Hepatitis C is not a cause of hepatocellular carcinoma (False) Hepatitis A is a cause of chronic liver disease (False) Hepatitis E can be acquired by sharing needles (False) A person with only a hepatitis B core IgG test positive is infectious for hepatitis B (False) Question 24. A 'fatty liver' may represent: Simply an obese person (False) Alcoholism (True) Explanation: A common 'early' abnormality.

By A. H.

MCQs VIA WEB 2005 Hepatitis C infection (True) Explanation: A common 'early' abnormality. Acute vitamin A poisoning (False) An ultrasound artefact (False) Question 25. Cushing's syndrome: Causes osteoporosis (True) Explanation: Also cardiorespiratory disease. The diagnosis is made by a high-dose dexamethasone test (False) Serum adrenocorticotrophic hormone (ACTH) is important in diagnosing the underlying cause (True) Explanation: Patients with primary adrenal Cushing's have unmeasurably low serum ACTH. A neoplasm causing a classical 'lemon-on-sticks' appearance is > 25% likely to be small cell carcinoma of the bronchus (False) Can only be cured by bilateral adrenalectomy (False) Module 2 (Master Medicine) Question 1. Acute renal failure is a likely complication of the following: Sepsis (or sepsis syndrome) (True) Explanation: Commonly caused by prerenal factors such as sepsis syndrome. Polycystic kidney disease (False) Major arterial surgery (True) Explanation: Major arterial surgery can cause renal ischaemia and acute tubular necrosis. Retroperitoneal tumours (False) Cardiogenic shock (True) Question 2. In patients with acute renal failure: Sodium bicarbonate should be given routinely (False) Most patients with acute renal failure need long-term dialysis (False) Skin turgor is a reliable guide to the need for i.v. fluid therapy (False) Urinary catheterisation is sometimes needed to monitor the response to therapy (True) Explanation: It is important to measure urine flow in the fluid management of acute renal failure. Intravenous pyelography is the investigation of choice to exclude urinary obstruction (False) Question 3. The following are causes of chronic renal failure: Gout (True) Explanation: In renal failure, the kidneys are unable to excrete urea so the urinary urea concentration is low. This distinguishes renal failure from, for example, volume depletion, in which plasma urea is high but the kidneys retain the capacity to concentrate urinary urea. Atherosclerosis (True) Explanation: As a result of extrarenal or intrarenal obstruction to the renal arterial circulation. Analgesic abuse (True) Explanation: As a result of extrarenal or intrarenal obstruction to the renal arterial circulation. Non-insulin-dependent diabetes (True) Explanation: Both insulin-dependent and non-insulin-dependent diabetes cause renal failure. Hypothyroidism (False) Question 4. The following may cause the nephrotic syndrome: Minimal change disease (True) Explanation: This is the characteristic disease associated with nephrotic syndrome, particularly in children. Treatment with beta-blockers (False) Rheumatoid arthritis (True) Explanation: It may be caused by amyloid associated with rheumatoid arthritis or by drugs used to treat the disease (gold or penicillamine). Rarely it is caused by a glomerulonephritis associated with the disease itself. Diabetes mellitus (True) Explanation: Although the full-blown nephrotic syndrome is a relatively uncommon presentation of diabetic nephropathy. Renal cell carcinoma (False)

By A. H.

MCQs VIA WEB 2005 Question 5. The following are features of urinary infections in elderly people: Patients usually complain of dysuria (False) They may present with falls (True) They may present with constipation (True) Explanation: Or it may be coexistent, perhaps reflecting anorexia and dehydration. Sterile pyuria is most likely caused by tuberculosis (False) Estrogen supplements may reduce their frequency in postmenopausal women (True) Explanation: Elasticity of the urethra is reduced postmenopausally and this can lead to infection. Local estrogen therapy helps. Question 6. Renal artery stenosis: Is invariably caused by atherosclerosis (False) May cause renal failure in patients given ACE (angiotensin-converting enzyme) inhibitor therapy (True) Explanation: Fibromuscular hyperplasia and radiation fibrosis are two other pathologies which can cause renal artery stenosis, although atherosclerosis is the most common pathology. Can be reliably diagnosed by auscultating for renal bruits (False) May be seen on ultrasound as a unilateral small kidney (True) Explanation: Hypoperfusion causes reduction in renal size. Is a cause of hypertension (True) Explanation: Hypoperfusion causes reduction in renal size. Question 7. The following are true: There is weakness of elbow extension in a crutch palsy (True) Explanation: The triceps is affected in a crutch palsy. Wasting of the hypothenar eminence occurs in the carpal tunnel syndrome (False) Abduction of the thumb is impaired in an ulnar nerve lesion (False) The index finger is hyperextended at the metacarpophalangeal (MCP) joint in an ulnar nerve lesion (False) Sensation is lost over the whole of the back of the hand in radial nerve damage (False) Question 8. The following are true: A cerebellar vermis lesion will result in a marked intention tremor (False) Macular sparing is a characteristic of lesions affecting the optic tract (False) In a patient with marked visuo-spatial inattention, the lesion is most likely in the left cerebral hemisphere (False) Agnosia means inability to plan and execute motor tasks (False) Dyscalculia is a feature of Alzheimer's disease (True) Explanation: Remember other higher cortical functions, e.g. dysphasia, dyslexia. Question 9. Features of a right sixth nerve palsy include: Convergent strabismus (True) Explanation: Complete paralysis of the lateral rectus leaves the medial rectus unopposed hence producing a convergent strabismus, though mostly the paralysis is only brought out when the eye is abducted. Diplopia worse on looking to the right (True) Explanation: Diplopia is maximal on looking in the direction of the primary action of the muscle. False image parallel to the true image (True) Explanation: Unlike a superior oblique palsy. False image occurs further to the left than the true image (False) Images become increasingly separated on looking to the left (False) Question 10. Parkinson's disease is associated with: Loss of dopamine transmission (True) Explanation: Although the mechanism is unclear, it does involve loss of dopaminergic neurons. Cogwheel rigidity (True) Explanation: Cogwheel rigidity is a superimposed tremor on the 'lead pipe' increase in tone. Tardive dyskinesia (False) Intention tremor (False) Festinant gait (True)

By A. H.

MCQs VIA WEB 2005 Question 11. In a young female with paraplegia, which of the following would suggest a diagnosis of multiple sclerosis: Periventricular lesions seen on MR scanning (True) Explanation: Periventricular plaques would imply disease remote from the spinal cord. MR scanning is the preferred imaging technique. Raised protein in cerebrospinal fluid (CSF) (False) Raised CSF globulin (True) Explanation: CNS immunology is disturbed in multiple sclerosis. Denervation of the muscles of the leg (False) Episode of visual disturbance (True) Explanation: Disturbances of visual acuity are an early sign. Question 12. The following are more suggestive of dementia than of depression: Several episodes of antisocial behaviour (True) Explanation: Antisocial behaviour is more in keeping with the personality change of dementia. Mutism (False) Duration of symptoms less than 1 month (False) Worsening of symptoms during the early morning (False) Marked impairment of concentration (False) Question 13. With respect to lumbar puncture: Coagulopathy is a contraindication (True) Explanation: However, if correctable (e.g. haemophiliac) and the indication for lumbar puncture is strong enough, then it should be corrected and the lumbar puncture carried out. Papilloedema is an absolute contraindication (False) The procedure may cause meningitis (False) The less CSF is removed, the less likely coning is to occur (False) Postlumbar puncture headache is related to the size of the needle used (False) Question 14. Outcome from bacterial meningitis relates to: Age of patient (True) Explanation: Mortality is highest in elderly people. Time to first administration of antibiotic (True) Explanation: Delays lead to increased mortality and morbidity. CSF concentration of antibiotic (True) Explanation: The CSF concentration of antibiotic needs to exceed by 20-fold the minimum inhibitory concentration of the infecting organism. This is the primary reason why i.v. therapy is necessary in meningitis. Development of antibiotic resistance during therapy (False) The causative organism (True) Explanation: Neisseria meningitidis has a lower mortality than S. pneumoniae meningitis. Furthermore about 5% of community-acquired cases are other organisms, such as Listeria monocytogenes. Listeria is intrinsically resistant to all cephalosporins, which are now the most common first line treatment for meningitis Question 15. The following statements are true: Hypocalcaemia causes prolongation of the prothrombin time (False) The prothrombin time is a sensitive test of hepatocellular dysfunction (True) Explanation: Because hepatocellular dysfunction impairs the synthesis of vitamin K-dependent clotting factors. The activated partial thromboplastin time (APTT) is prolonged by unfractionated heparin therapy (True) Explanation: This is used as a measure of heparinisation. The effect of heparin is reversed by vitamin K (False) Deep venous thrombosis can be reliably diagnosed by measuring fibrin degradation products (FDPs) (False) Question 16. The following may cause a microcytic anaemia: Sickle cell disease (False) The thalassaemias (False) Anaemia of chronic disease (False) Anticonvulsant therapy (False)

By A. H.

MCQs VIA WEB 2005 Haemolysis, whatever the cause (False)

Question 17. The following statements are true: A neutrophil count of only 0.8 × 109 cells/l is a major risk for infection (False) A neutrophil count in a febrile patient of 25 × 109 cells/l reflects mostly the production of new neutrophils from the bone marrow (False) In a patient with less than 0.1 × 109 cells/l neutrophils and a fever, treatment with antibiotics should await the results of blood culture (False) Neutropenia is common in AIDS (False) Neutropenia can be caused by carbimazole therapy (True) Explanation: Neutropenia occurs in 1:10000 patients treated with carbimazole for thyrotoxicosis. Question 18. Prognosis of diabetes: Cardiovascular mortality is higher in diabetic than in non-diabetic people up to the age of 80 (True) Explanation: A threefold increase. Diabetic patients with proteinuria have a higher cardiovascular risk than those without it (True) Explanation: It is indicative of nephropathy, which increases the risk of cardiovascular disease up to 100-fold. When sulfonylureas became available, there was a noticeable improvement in cardiovascular mortality (False) Good glycaemic control, on the balance of available evidence, can reduce cardiovascular mortality in both type 1 and type 2 DM (False) Even mildly 'impaired glucose tolerance' increases cardiovascular risk (True) Question 19. In secondary diabetes: A patient can be assumed not to be ketosis-prone (False) A patient is more than 85% likely to have clinical pancreatic exocrine deficiency (False) Classical diabetic complications do not occur (False) Thiazide diuretics and beta-blockers can both impair insulin secretion (True) Explanation: Secondary diabetes causes all the same complications as idiopathic diabetes. Most patients with acromegaly are diabetic (False) Question 20. Diabetic retinopathy: Characteristically causes arterio-venous nipping (False) Should be referred to an ophthalmologist only if the patient has visual symptoms (False) Inevitably causes blindness (False) May cause cotton wool spots (soft exudates) (True) Explanation: These may also occur in hypertension and other ischaemic retinopathies. Is more likely to cause blindness in type 1 than in type 2 DM (False) Question 21. In insulin treatment: Pen injectors are reserved for the small minority who take four or more injections per day (False) Only patients who cannot be controlled with once-daily insulin should have two or more injections (False) Insulin should be started without delay in a thin hyperglycaemic patient with ketonuria (True) Explanation: These are signs of type 1 DM. Insulin may sometimes be needed during short periods of illness in patients with type 2 DM (True) Explanation: These are signs of type 1 DM. All patients on insulin should be discouraged from changing their doses without first checking with the doctor or nurse (False) Question 22. Hypertension in diabetes: Is more prevalent in type 1 than in type 2 (False) Its treatment slows the deterioration of nephropathy in type 1 DM (True) Explanation: Hypertension is associated with type 2 more strongly than with type 1 DM. Thiazide diuretics should not be used in diabetes (False) Beta-blockers may increase the risk of severe hypoglycaemia in insulin-treated patients (True) Explanation: This is true primarily of non-cardioselective beta-blockers. Increases the risk of stroke in diabetes (True)

By A. H.

MCQs VIA WEB 2005 Explanation: This is true primarily of non-cardioselective beta-blockers

Question 23. The following are seen with NSAIDs: Improvement in renal function (False) Increase in serum potassium (True) Explanation: The change in renal function results in hyperkalaemia. Increased risk of peptic ulcer complications (True) Explanation: There is a clear relationship between NSAID use and complications such as perforation, bleeding and death particularly in old people. Improved long-term prognosis of rheumatoid arthritis (False) Improvement in coexistent asthma (False) Question 24. The following are features of systemic lupus erythematosus (SLE) Raynaud's phenomenon (True) Explanation: Pain during mastication is a characteristic feature of temporal arteritis. Mononeuritis multiplex (True) Explanation: Pain during mastication is a characteristic feature of temporal arteritis. Thrombocytopenia (True) Explanation: This is one of the typical blood-associated dyscrasias. Lymphopenia (True) Explanation: As with thrombocytopenia Question 25. In primary osteoarthritis: The ESR is normal (True) Explanation: There are no haematological abnormalities. PIP joints are not usually affected (False) Radiographs show characteristic erosions of articular margins (False) Morning stiffness usually lasts over 1 hour (False) First carpometacarpal joint involvement is a common finding (True) Explanation: This is common, resulting in 'squaring' of the hand Available from Davidson's Principles and Practice of Medicine Module 1 (Chapter 1) Question 1. The following infections may be acquired by the following means tetanus-respiratory droplets or dust (False) Explanation: Via wounds and abrasions listeriosis-eating contaminated cheese (True) Explanation: Can survive refrigeration legionellosis-water aerosols (True) schistosomiasis-via penetration of the skin (True) leptospirosis-via rat urine (True) Question 2. Diseases typically acquired from animals include leptospirosis (True) Explanation: From the urine of rats or dogs Mycobacterium tuberculosis (False) Explanation: Mycobacterium bovis toxoplasmosis (True) Explanation: From dog faeces psittacosis (True) Explanation: From birds hepatitis A (False) Explanation: Faecal-oral spread Question 3. Live viruses are usually used for active immunisation against

By A. H.

MCQs VIA WEB 2005 poliomyelitis (True) Explanation: Inactivated vaccine also available pertussis (False) typhoid fever (False) mumps, measles and rubella (True) Explanation: Do not give to immunosuppressed patients hepatitis B (False) Question 4. Pyrexia of unknown origin is defined as a temperature of more than 37.5°C persisting for more than 2 weeks (True) Explanation: Not elucidated by investigation in hospital is due to infection in 75% of cases (False) Explanation: In approximately 30% only may be factitious (True) Explanation: Suspect if ESR and CRP normal can be caused by granulomatous hepatitis (True) Explanation: And other forms of hepatitis may be elucidated by bone marrow biopsy (True) Explanation: May diagnose haematological malignancy Question 5. The following statements about infectious mononucleosis are true infection is usually attributable to the Epstein-Barr virus (EBV) (True) presentation is with fever, headache and abdominal pain (True) Explanation: And malaise and anorexia sore throat suggests cytomegalovirus rather than EBV infection (False) meningoencephalitis and pericarditis are recognised complications (True) severe oropharyngeal swelling requires prednisolone therapy (True) Explanation: Especially if there is dysphagia or breathing difficulty Question 6. Typical features of toxoplasmosis include the following infection is derived from cats, pigs and sheep (True) Explanation: Immunocompromised patients are most at risk peak age of onset is over 65 years of age (False) Explanation: 25-35 years congenital infection produces choroidoretinitis (True) Explanation: And sometimes microcephaly there is a positive heterophil antibody test (False) Explanation: This is typically negative pyrimethamine and sulfadiazine therapy is useful in immunocompromised patients (True) Question 7. Recognised features of brucellosis include a characteristically rapid response to penicillins (False) Explanation: Typically doxycycline and streptomycin fever, night sweats and back pain (True) Explanation: And joint pains and anorexia splenomegaly (True) Explanation: But a non-specific finding oligoarthritis and spondylitis (True) Explanation: Due to localised granulomatous disease thrombocytopenia (True) Explanation: Due to hypersplenism Question 8. The typical features of leptospirosis include incubation period of 1-3 months (False) Explanation: 7-14 days exposure risk in abattoirs, farms and inland waterways (True) fever, severe myalgia, headache and conjunctival suffusion (True)

By A. H.

MCQs VIA WEB 2005 Explanation: With abrupt onset meningitis in Leptospira icterohaemorrhagiae rather than L. canicola infection (False) Explanation: L. canicola infection is usually associated with aseptic meningitis possible diagnosis by examination of the urine (True) Explanation: Leptospires appear in the urine in the second week of illness Question 9. The clinical features of Lyme disease include infection with the tick-borne spirochaete Borrelia burgdorferi (True) Explanation: Ixodes species of tick an expanding erythematous rash (erythema chronicum migrans) (True) Explanation: An annular red lesion cranial nerve palsies (True) Explanation: Or meningitis or radiculopathy asymmetrical large joint recurrent oligoarthritis (True) Explanation: Not in acute stages response to tetracycline or penicillin therapy (True) Explanation: And cephalosporins Question 10. Features consistent with the diagnosis of Q fever include exposure to sheep, cattle and unpasteurised milk (True) Explanation: Especially butchers and abattoir workers meningoencephalitis (True) pneumonia in the absence of fever, headache or myalgia (False) Explanation: Acute Q fever is an influenza-like illness blood culture-negative endocarditis (True) prompt clinical response to sulphonamide therapy (False) Explanation: Responds to tetracyclines Question 11. The typical features of erysipelas include group A haemolytic streptococcal skin infection (True) Explanation: Streptococcus pyogenes absence of constitutional symptoms (False) Explanation: Systemic upset is common well-defined area of cutaneous erythema and oedema (True) Explanation: The rash has a palpably raised edge painless swelling (False) Explanation: Typically painful prompt response within 48 hours to benzylpenicillin (True) Question 12. Clinical features of anthrax include occupational exposure to animals and animal products (True) Explanation: Farmers, butchers and dealers in wool, hides and bone meal an incubation period of 1-3 weeks (False) Explanation: 1-3 days a painless cutaneous papule (True) Explanation: Painless but itchy gastroenteritis and bronchopneumonia (True) multiple antibiotic resistance (False) Explanation: The organism is widely sensitive Question 13. The features of herpes simplex (HS) virus infections include recurrent genital ulcers (True) Explanation: Especially HS type 2 acute gingivostomatitis (True) Explanation: HS type 1 encephalitis (True) Explanation: HS type 1

By A. H.

MCQs VIA WEB 2005 shingles (False) Explanation: Varicella zoster virus paronychia (True) Explanation: HS type 1-'herpetic whitlow' Question 14. In a schoolchild with measles infection is due to a paramyxovirus (True) rhinorrhoea and conjunctivitis occur at the onset (True) Explanation: The catarrhal phase Koplik's spots appear at the same time as the skin rash (False) Explanation: They precede the rash the skin rash typically desquamates as it disappears (True) infectivity is confined to the prodromal phase (False) Explanation: Contact should be avoided for 7 days after the onset of the rash Question 15. In patients with rubella infection the RNA virus spreads by the faecal-oral route (False) a prolonged fever is typical (False) Explanation: Typically only on the first day of the rash infectivity is present for 7 days before and after the rash (True) sub-occipital lymphadenopathy is typical (True) the risk of serious fetal damage is < 5% after the 16th week of pregnancy (True) Explanation: Greatest risk is in the first 8 weeks Question 16. The characteristic features of mumps include infection with an RNA paramyxovirus by airborne spread (True) high infectivity for 3 weeks after the onset of parotitis (False) Explanation: Infectivity is generally low presentation with an acute lymphocytic meningitis (True) abdominal pain attributable to mesenteric adenitis (False) Explanation: Pain suggests pancreatitis or oophoritis orchitis which predominantly occurs prepubertally (False) Explanation: It is usually unilateral and postpubertal Question 17. The clinical features of amoebic dysentery include an incubation period of 2-4 weeks (False) Explanation: May develop many months after exposure presentation with blood and mucus per rectum (True) Explanation: Acute colitic symptoms often seen in the old good response to metronidazole in intestinal disease (True) characteristic appearances of the mucosa on sigmoidoscopy (True) Explanation: Flask-shaped ulcers antibodies detectable by immunofluorescence in only a small minority of patients (False) Explanation: In 60-95% Question 18. The following statements about the life cycle of plasmodia are true sporozoites disappear from the blood within minutes of inoculation (True) Explanation: Sporozoites enter the liver within 30 minutes merozoites re-entering red blood cells undergo both sexual and asexual development (True) all plasmodia multiply in the liver then subsequently in red blood cells (True) Explanation: Duration of the pre-patent period varies dormant hypnozoites remain within the liver cells in all species (False) Explanation: Only P. vivax and P. ovale persist in this form fertilisation of the gametocytes occurs in the human red blood cells (False) Explanation: Fertilisation occurs in the mosquito Question 19. Recognised clinical features of malaria include

By A. H.

MCQs VIA WEB 2005 absence of P. vivax infection in subjects lacking the Duffy blood group (True) Explanation: West Africans and African Americans are protected asymptomatic P. malariae parasitaemia persisting for years (True) Explanation: With or without symptoms enhanced risk of infection in splenectomised patients (True) presentation with rigors, herpes simplex and haemolytic anaemia (True) Explanation: Especially in P. vivax and P. ovale infection excellent response to chloroquine (False) Explanation: Widespread resistance-quinine preferred Question 20. The features of typhoid fever include faecal-oral spread of Salmonella typhi by food handlers (True) Explanation: Usually asymptomatic carriers presentation with constipation (True) Explanation: But diarrhoea more common in children onset with fever, headache and myalgia (True) Explanation: And relative bradycardia 'rose spots' on the trunk and splenomegaly 7-10 days after onset (True) development of carrier state in 50% of survivors (False) Explanation: 5% Question 21. The following are possible causes of fever and a rash in a traveller returning from the tropics paratyphoid fever (True) leptospirosis (True) meningococcal infection (True) secondary syphilis (True) HIV seroconversion (True) Question 22. In the diagnosis of the enteric fevers blood cultures are usually positive 2 weeks after onset (False) Explanation: Bacteraemia in the first week stool cultures are usually positive within 7 days of onset (False) Explanation: More likely in the second or third week peripheral blood neutrophil leucocytosis is typically marked (False) Explanation: Leucopenia is typical the Widal reaction is typically positive within 7 days of onset (False) Explanation: There are frequent false negatives persistent fever despite antibiotics indicates resistant organisms (False) Explanation: It may suggest a septicaemic focus Question 23. Clinical features of dengue include mosquito-borne infection with an incubation period of 2-7 days (True) continuous or 'saddle-back' fever (True) Explanation: Fever may remit on day 4-5 ('saddle-back') rigors, headache, photophobia and backache (True) Explanation: But non-specific morbilliform rash and cervical lymphadenopathy (True) Explanation: Rash starts peripherally protection by vaccination every 10 years in endemic areas (False) Explanation: No vaccine is available Question 24. The typical features of African trypanosomiasis include transmission of the parasite by the tsetse cattle fly (True) an incubation period of 2-3 weeks (True) Explanation: Occasionally longer in T. gambiense infections onset with chancre-like skin lesion and local lymphadenopathy (True) Explanation: At the site of the bite

By A. H.

MCQs VIA WEB 2005 generalised lymphadenopathy, hepatosplenomegaly and encephalitis (True) good prognosis given prompt pentamidine or suramin therapy (True) Explanation: Unless cerebral infection has developed Question 25. Typical features of visceral leishmaniasis (kala-azar) include spread of Leishmania donovani by sandflies from dogs and rodents (True) Explanation: Also spread from infected blood transfusions an incubation period of 1-2 weeks (False) Explanation: 1 month to 10 years rigors with hepatomegaly but no splenomegaly (False) Explanation: Splenomegaly is characteristic diagnosis confirmed on peripheral blood film (False) Explanation: Diagnosis by examination of stained smears of bone marrow, spleen or liver clinical response to pentavalent antimonials, e.g. stibogluconate (True) Explanation: Amphotericin B is an alternative Question 26. In diphtheria heart block is a recognised complication (True) Explanation: Although cardiac involvement usually causes no long-term problems high fever is a typical early sign (False) Explanation: Fever rarely dominant-insidious onset isolation is usually unnecessary (False) Explanation: Isolation is vital paralysis of the soft palate, accommodation or ocular muscles may occur (True) Explanation: Occasionally with peripheral polyneuritis treatment is with antibiotics alone (False) Explanation: Diphtheria antitoxin is also important Question 27. The typical features of strongyloidiasis include skin penetration with migration to the gut via the lungs (True) Explanation: Producing an itchy rash larval penetration of the duodenal and jejunal mucosa (True) Explanation: With pain, diarrhoea, steatorrhoea and weight loss abdominal pain, diarrhoea and malabsorption (True) penetration of perianal skin producing a migrating linear weal (True) Explanation: Intensely itchy systemic spread in the immunosuppressed, resulting in pneumonia (True) Explanation: Seen in HIV infection Question 28. In infestation with the nematode Enterobius vermicularis adult threadworms occur in great numbers in the small bowel (False) Explanation: Seen in the colon presentation with intense pruritus ani is typical (True) Explanation: Worms may be visible identifiable ova are found on the perianal skin (True) malabsorption usually develops following heavy infestations (False) Explanation: The small bowel is unaffected all family members should take piperazine or mebendazole therapy (True) Explanation: Cross-infection and autoinfection are common Question 29. In onchocerciasis larval infection is transmitted by the Simulium fly (True) Explanation: A painful bite worms mature over 2-4 weeks and persist for up to 1 year (False) Explanation: Worms can live for over 15 years cutaneous nodules and eosinophilia commonly develop (True) Explanation: The nodules contain adult worms

By A. H.

MCQs VIA WEB 2005 conjunctivitis, iritis and keratitis are characteristic (True) ivermectin is the drug therapy of choice (True) Question 30. In schistosomal infection painless haematuria may be the presentation (True) Explanation: Due to bladder mucosal involvement diagnosis can be made by finding cercariae in the urine and/or stool (False) Explanation: Eggs are passed in urine and/or stool the helminths mature in the portal vein (True) peripheral neuropathy commonly causes lower limb weakness (False) Explanation: But transverse myelitis may praziquantel is the therapy of choice (True) Explanation: Or oxamniquine or metrifonate Question 31. Echinococcus granulosus infestation is usually associated with contact with sheep, cattle and dogs (True) Explanation: May be many years before clinical manifestations appear acquisition of hydatid cysts in childhood (True) Explanation: Usually an asymptomatic event cysts in the liver, brain and lungs (True) Explanation: Right lobe of the liver is the commonest site absence of dissemination during liver aspiration (False) Explanation: Care must also be taken during excision prompt response to albendazole therapy if surgically inoperable (False) Explanation: But further enlargement may be prevented Question 32. Typical features of cutaneous leishmaniasis include nasal and oral mucosal ulcers (True) Explanation: Secondary to initial cutaneous ulceration painful ulcers in the groins or axillae (False) Explanation: Typically painless and not involving nodes marked splenomegaly and lymphadenopathy (False) Explanation: These occur in visceral leishmaniasis ulcers which heal without scarring (False) negative leishmanin skin test (False) Explanation: Typically positive except in diffuse cutaneous leishmaniasis Question 33. Characteristic features of leprosy include an incubation period of 2-12 years (True) growth of the organism on Löwenstein-Jensen medium after 2-3 months (False) Explanation: The organism cannot be grown in artificial media spread of the tuberculoid form by prolonged patient contact (False) Explanation: There is no risk of infection in tuberculoid leprosy thickened palpable peripheral nerves (True) a cell-mediated immune response in the lepromatous form (False) Explanation: Characteristic of the tuberculoid form Question 34. Typical features of lepromatous leprosy include early and marked sensory loss (False) Explanation: Late and limited unlike the tuberculoid form, organisms are scanty in number (False) Explanation: Is a multibacillary disease blood-borne spread from the dermis throughout the body (True) Explanation: No cell-mediated immune response strongly positive lepromin skin test (False) Explanation: Suggests tuberculoid disease anaesthetic hypopigmented skin macules and plaques (False)

By A. H.

MCQs VIA WEB 2005 Explanation: Macules occur, but sensation is retained

Question 35. The following are likely causes of splenomegaly in a patient with fever returning from the tropics tuberculosis (False) trypanosomiasis (True) brucellosis (True) visceral leishmaniasis (True) infective endocarditis (True) Question 36. The following statements about syphilis are true infection is usually caused by Treponema pertenue (False) Explanation: Due to infection with Treponema pallidum cardiac murmurs are a typical early feature of infection (False) Explanation: A feature of late disease the primary lesion at the site of infection is initially macular (True) Explanation: But becomes papular, then chancrous the incubation period for primary syphilis is typically 2-4 weeks (True) Explanation: But may be up to 90 days tertiary syphilis usually develops within 1 year of infection (False) Explanation: Takes at least 2 years to develop Question 37. Characteristic features of late (tertiary and quaternary) syphilis include negative specific treponemal antigen tests (False) Explanation: The tests are typically positive destructive granulomas (gummas) in bones, joints and the liver (True) sensory ataxia (True) Explanation: Due to dorsal column spinal disease aneurysms of the ascending aorta (True) Explanation: Typically with calcification poor response of gummas to antibiotic therapy (False) Question 38. The typical clinical features of gonorrhoea include an incubation period of 2-3 weeks (False) Explanation: 2-10 days anterior urethritis and cervicitis (True) Explanation: Dysuria, discharge or no symptoms right hypochondrial pain due to perihepatitis (True) pharyngitis (True) good response to ciprofloxacin therapy in penicillin allergy (True) Explanation: Or spectinomycin Question 39. Anogenital herpes simplex is typically associated with type 2 herpes simplex infection only (False) Explanation: Type 2 and type 1 equally primary attacks more severe and prolonged than recurrent attacks (True) Explanation: Healing is more rapid in recurrent attacks fever with painful genital ulceration and lymphadenopathy (True) sacral dermatomal pain and urinary retention (True) absence of clinical response to oral aciclovir (False) Explanation: Shortens first attacks and may prevent recurrence Question 40. HIV infection is associated with an RNA retrovirus (True) heterosexual transmission in the majority of cases world-wide (True) Explanation: Superseding homosexual and parenteral involvement of CD4 lymphocytes (True)

By A. H.

MCQs VIA WEB 2005 a viral half-life of 1-2 hours in plasma (True) a better prognosis in the presence of Kaposi's sarcoma (False) Explanation: Prognosis is worse with Kaposi's sarcoma Question 41. In HIV infection 80% of vertically transmitted infections are transplacental (False) Explanation: Majority occur during parturition a child born to an infected mother has a 90% chance of acquiring HIV (False) Explanation: Under 50% chance transmission can occur via breast milk (True) Explanation: 10-20% additional risk for breast-fed babies risk of fetal transmission is unaffected by pre-partum antiviral agents (False) Explanation: HAART can reduce transmission rate Question 42. In the diagnosis of HIV infection ELISA testing has a low false negative rate (True) Explanation: ELISA testing therefore widely used as a screening test seroconversion invariably occurs in under 4 weeks (False) Explanation: 6-12 weeks or longer antibody detection tests are particularly helpful in neonates (False) Explanation: May have transplacentally acquired maternal antibody HIV-RNA can be directly measured as a confirmatory test (True) Explanation: Sometimes used as a confirmatory test HIV-RNA is typically detected before anti-HIV antibodies (True) Question 43. In the classification of HIV infection group A = acute seroconversion simulating glandular fever (True) Explanation: Also includes asymptomatic patients group B = persistent generalised lymphadenopathy (False) Explanation: Classed as group A infection group C = constitutional symptoms and oral candidiasis (False) Explanation: Group C includes conditions meeting CDC/WHO case definition group A1/B1/C1 all have absolute CD4 count > 500/mm3 (True) group B = asymptomatic infection (False) Explanation: Group A are asymptomatic Question 44. Presenting features of HIV infection include hairy leucoplakia (True) Explanation: Affects the tongue and mouth atypical pneumonia (True) Explanation: Especially Pneumocystis carinii thrombocytopenic purpura (True) pulmonary tuberculosis (True) Explanation: Sometimes with atypical mycobacteria candidiasis and cryptosporidiosis (True) Question 45. Cryptosporidiosis in an HIV-positive patient is an AIDS-defining diagnosis if chronic (True) likely to present with painless profuse diarrhoea (False) Explanation: Profuse diarrhoea, but usually with abdominal pain likely to be self-limiting if the CD4 count is > 200 cells/mm3 (True) preventable by the use of boiled tap water (True) usually diagnosed on stool microscopy (True) Question 46. Pneumocystis carinii infection in an HIV-positive patient is the commonest cause of respiratory infection in African patients (False) Explanation: Tuberculosis is more common

By A. H.

MCQs VIA WEB 2005 characterised by copious sputum production (False) Explanation: Dry cough and dyspnoea characterised by widespread fine pulmonary crackles (False) Explanation: Crackles would be unusual more likely to occur when the CD4 count is < 200/mm3 (True) Explanation: In 95% of cases excluded by the finding of a normal chest X-ray (False) Explanation: Normal chest radiograph is found in 15-20% of cases Question 47. In a patient with AIDS, cryptococcal meningitis is the commonest cause of meningitis (True) Explanation: Also causes pulmonary disease characterised by abrupt onset of the classical features of a bacterial meningitis (False) Explanation: Indolent onset diagnosed by India ink stain of cerebrospinal fluid (CSF) (True) Explanation: And serum/CSF culture typically associated with negative CSF culture (False) associated with deafness in survivors (True) Explanation: And blindness Question 48. In the treatment of HIV infection all useful drugs work via inhibition of reverse transcriptase (False) Explanation: Some are protease inhibitors nucleoside reverse transcriptase inhibitors may cause peripheral neuropathy (True) reverse transcriptase inhibitors prevent spread of infectious virus into uninfected cells (True) Explanation: But not replication drug-resistant strains of virus have not been recognised (False) Explanation: As with zidovudine monotherapy is preferred (False) Explanation: Survival rates improve with combination regimens Question 49. Antimicrobial therapy acts in the following ways aminoglycosides disrupt bacterial protein synthesis (True) Explanation: Via ribosomal binding sulphonamides interrupt bacterial folate synthesis (True) Explanation: And hence nucleic acid synthesis penicillins disrupt bacterial protein synthesis (False) Explanation: Affect cell wall synthesis cephalosporins disrupt bacterial cell wall synthesis (True) Explanation: As with penicillins tetracyclines disrupt bacterial protein synthesis (True) Explanation: Via ribosomal binding Question 50. The following statements about penicillins are true all penicillins are bactericidal (True) Explanation: By interfering with their cell wall synthesis like the cephalosporins, they contain a â-lactam ring (True) Explanation: Resistance by â-lactamase-producing organisms is common clavulanic acid inhibits bacterial â-lactamase (True) Explanation: Used in combination with amoxicillin as co-amoxiclav they are all safe in pregnancy (False) Explanation: Imipenem is not they are synergistic with aminoglycosides (True) Question 51. Erythromycin is active against the following microorganisms Campylobacter jejuni (True) Escherichia coli (False)

By A. H.

MCQs VIA WEB 2005 Explanation: Hence less likely to disrupt bowel flora Legionella pneumophila (True) Mycoplasma pneumoniae (True) Explanation: In appropriate dosage Clostridium welchii (True) Question 52. Aminoglycoside drug therapy is ototoxic and nephrotoxic (True) Explanation: Especially in the elderly is well absorbed orally (False) Explanation: Negligible oral absorption must be monitored using plasma drug concentrations (True) Explanation: Serum levels and duration of therapy correlate with risk of toxicity is effective against anaerobes and Streptococcus faecalis (False) Explanation: No anti-anaerobic activity is very effective against Gram-negative organisms (True) Question 53. Ciprofloxacin is highly active against the following microorganisms Escherichia coli (True) Haemophilus influenzae (True) Proteus mirabilis (True) Explanation: Active against most of the enterobacteria Streptococcus pneumoniae (False) Explanation: Only moderate activity Bacteroides fragilis (False) Question 54. The following antiviral agents are active against the following viruses ganciclovir-cytomegalovirus (True) amantadine-orthomyxovirus (True) Explanation: Used in prophylaxis of influenza A ribavirin-respiratory syncytial virus (True) Explanation: Also active in Lassa fever zidovudine-retrovirus (True) Explanation: Used in AIDS famciclovir-herpes simplex and herpes zoster viruses (True) Explanation: Like aciclovir, useful orally or parenterally Module 2 (Chapter 2) Question 1. 200 patients with hypertension are treated with a new drug to prevent strokes and compared with 200 similar patients who are given a placebo in a randomised controlled clinical trial (RCT). After 1 year of treatment 5 patients in the treatment group and 10 patients in the control group have suffered a stroke. Which of the following statements are true? the absolute risk reduction with treatment is 5% (False) Explanation: 2.5% the relative risk is 0.5 (True) Explanation: 50% relative risk reduction the number needed to treat is 200 (False) Explanation: 40 all patients with hypertension will benefit from this treatment (False) Explanation: Only patients similar to those in the trial benefit can be expected to be similar in following years of treatment (False) Explanation: Can only be derived from continuing the RCT

Question 2. Examples of pharmacokinetic interactions include the following allopurinol inhibits the metabolism of azathioprine (True) Explanation: And 6-mercaptopurine; both are metabolised by xanthine oxidase metoclopramide delays gastric emptying and the rate of drug absorption (False)

By A. H.

MCQs VIA WEB 2005 Explanation: It increases the rate of gastric emptying digoxin and verapamil compete for renal tubular secretion (True) Explanation: Similarly, quinidine and amiodarone compete with digoxin for renal excretion the effect of methotrexate is inhibited by NSAID therapy (False) Explanation: Increased effect due to inhibition of renal tubular secretion of methotrexate renal lithium excretion is inhibited by diuretics (True) Explanation: Recommend a barrier method as well for patients on the contraceptive pill and taking antibiotics Question 3. The following drugs should be avoided in severe renal failure gentamicin (False) Explanation: But reduce dose frequency and measure plasma concentrations daily oxytetracycline (True) Explanation: Induces protein catabolism and rapidly increasing uraemia morphine (False) Explanation: But reduce both dose and dose frequency mesalazine (True) Explanation: Like all NSAIDs, reduces renal blood flow by prostaglandin inhibition metformin (True) Explanation: Causes lactic acidosis Question 4. The following drugs exhibit high rates of hepatic clearance codeine phosphate (False) Explanation: Similar to paracetamol in this respect diazepam (False) Explanation: Low rates of clearance during its first passage through the liver simvastatin (True) Explanation: Lidocaine (lignocaine) is also rapidly cleared during its first passage through the liver ('first-pass' effect) propranolol (True) warfarin (False) Question 5. The actions of the following drugs are enhanced in liver disease warfarin (True) Explanation: Reduces the synthesis of clotting factors metformin (True) Explanation: Produces lactic acidosis chloramphenicol (True) Explanation: Induces bone marrow suppression sulphonylureas (True) Explanation: Increase the risk of hypoglycaemia naproxen (True) Explanation: Like other NSAIDs, increases the risk of gastrointestinal bleeding Question 6. The following statements about drug prescribing in elderly patients are true the error rate in patients taking prescribed drugs is similar to that found in younger adults (False) Explanation: Error rates of up to 60% can be found in patients over the age of 60 years adverse drug reactions are more likely to occur than in younger adults (True) Explanation: Adverse drug reactions are 2-3 times more common an increased proportion of body fat increases the accumulation of lipid-soluble drugs (True) Explanation: Propranolol accumulation is also increased by reduced drug metabolism drug excretion is typically increased due to impaired urinary concentrating ability (False) Explanation: Impaired renal clearance associated with a reduced glomerular filtration rate is common metabolism of paracetamol reduces with advancing age (True) Explanation: As with other drugs (e.g. theophylline and sedative drugs) doses should be reduced Question 7. The following are statutory requirements for the prescription of controlled drugs prescriptions must be typewritten not written by hand (False) Explanation: Prescriptions must be written entirely in the prescriber's own handwriting, in ink

By A. H.

MCQs VIA WEB 2005 prescriptions must specify the patient's name and address (True) prescriptions must specify the prescriber's name and address (True) prescriptions must state the dosage in both words and numbers (True) Explanation: Including the total quantity, number of doses, and form and strength of the drug prescriptions must be signed and dated by the prescriber (True) Module 3 (Chapter 3) Question 1. The use of oral activated charcoal is indicated following poisoning with paracetamol (True) Explanation: More effective if given early acetylsalicylic acid (True) Explanation: More effective if given early and repeated 4-hourly ('gut dialysis') ferrous sulphate (False) Explanation: Not absorbed by activated charcoal ethylene glycol (False) Explanation: Not absorbed by activated charcoal lithium carbonate (False) Explanation: Not absorbed by activated charcoal Question 2. Typical features 6-8 hours after paracetamol poisoning include nausea and vomiting (True) Explanation: Abdominal pain may develop coma and internuclear ophthalmoplegia (False) Explanation: Late features suggesting hepatic encephalopathy (after 3-5 days) prolongation of the prothrombin time (False) Explanation: Rare before 24 hours metabolic acidosis and hypoglycaemia (False) Explanation: Consequence of hepatic necrosis (after 36 hours) prevention of liver damage with N-acetylcysteine therapy (True) Explanation: But not useful beyond 15 hours Question 3. Features of salicylate poisoning in an adult may include metabolic acidosis (True) Explanation: A poor prognostic sign deafness, tinnitus and blurred vision (True) Explanation: Common features hypokalaemia and respiratory alkalosis (True) Explanation: Due to hyperventilation hyperventilation, sweating and restlessness (True) peripheral vasodilatation (True)

Question 4. The following treatments are clinically useful in poisoning with the following agents glucagons-â-blockers (True) DMPS (dimercaprol)-heavy metal poisons (True) Explanation: Useful in arsenic, gold and mercury poisoning flumazenil-opioid analgesics (False) Explanation: Used in benzodiazepine overdose N-acetylcysteine-paracetamol (True) Explanation: As indicated by plasma paracetamol concentrations post-ingestion desferrioxamine-iron salts (True) Question 5. Typical features following benzodiazepine poisoning include ataxia, dysarthria, nystagmus and drowsiness (True) severe systemic hypotension and respiratory depression (False) Explanation: Severe cardiorespiratory depression is rare nausea, vomiting and diarrhoea (False)

By A. H.

MCQs VIA WEB 2005 Explanation: Suspect mixed overdose convulsions, muscle spasms and papilloedema (False) Explanation: Suspect alternative or mixed overdose resolution of symptoms and signs within < 6 hours of poisoning (False) Explanation: Usually < 24 hours Question 6. The following are true of cocaine poisoning hypothermia is a typical feature (False) Explanation: Hyperthermia or pyrexia cerebellar signs may occur (True) Explanation: As may convulsions myocardial infarction occurs only in the presence of abnormal coronary arteries (False) Explanation: They may be normal activated charcoal is of benefit within 1 hour of ingestion (True) a dose of over 10 mg would usually be regarded as potentially fatal (False) Explanation: Over 1 g Question 7. Typical features of morphine poisoning include nausea, vomiting and pallor (True) coma with widely dilated pupils (False) Explanation: Pinpoint pupils hypoventilation and respiratory arrest (True) hypotension and hypothermia (True) Explanation: Use naloxone non-cardiac pulmonary oedema (True) Explanation: Characteristic and the commonest mode of death Question 8. Typical features of carbon monoxide poisoning include nausea, vomiting (False) Explanation: Common features include agitation, headache and confusion marked central cyanosis (False) Explanation: Usually skin pallor; patients may appear 'pink' due to carboxyhaemoglobin hypotension and myocardial ischaemia (True) Explanation: Especially in patients whose coma is prolonged cognitive impairment and personality changes following recovery (True) Explanation: Due to the effects of cerebral oedema and cerebral anoxia parkinsonian features following recovery (True) Explanation: Neuropsychiatric sequelae occur in 10% 2-4 weeks following recovery Module 4 (Chapter 4) Question 1. The following statements about pulmonary artery wedge pressure (PAWP) monitoring are correct PAWP provides an indirect measure of left atrial pressure (True) the normal range is 15-20 mmHg (False) Explanation: 6-12 mmHg the PAWP is reduced in acute left ventricular failure (False) Explanation: Increased, often > 35 mmHg complications of monitoring include pulmonary artery rupture (True) Explanation: Also pneumothorax, air embolism, sepsis and arrhythmias the optimum PAWP in acute circulatory failure is 12-15 mmHg (True)

Question 2. The following statements about monitoring of pulmonary function are correct oxygen saturation (SaO2) should be maintained in the range 75-85% (False) Explanation: Maintain > 90% the oxygenation index (PaO2/FIO2) is a useful measure of gas exchange (True) Explanation: As is alveolar arterial oxygen gradient end-tidal alveolar CO2 concentration measures the effectiveness of ventilation (True) Explanation: As does PaCO2

By A. H.

MCQs VIA WEB 2005 measurement of oxygen saturation requires arterial blood sampling (False) Explanation: Finger or earlobe spectrophotometry is satisfactory in most instances a decreasing cardiac output is likely to induce an abrupt fall in SaO2 (True) Question 3. The following statements about oxygen transport in the blood are correct the amount of oxygen carried by haemoglobin is equal to that dissolved in the plasma (False) Explanation: Hb carriage accounts for the majority an increase in PaCO2 shifts the oxygen/haemoglobin dissociation curve to the right (True) Explanation: Bohr effect-facilitates unloading of O2 to tissues the optimum haemoglobin concentration in a critically ill adult male is 15 g/dl (False) Explanation: 7-10 g/dl to minimise hyperviscosity problems at a PaO2 = 3.5 kPa, approximately 10% of the haemoglobin will be saturated (False) Explanation: Around 50% increasing the haemoglobin concentration of the blood will increase its oxygen content but not its partial pressure of oxygen (True) Explanation: Hb concentration and saturation are major determinants of O2 content Question 4. The following statements about oxygen consumption are correct VO2 (global oxygen consumption) can be calculated from the PaO2 and the PaCO2 (False) Explanation: Calculated from inspiratory/expiratory gas analysis mixed venous oxygen saturation (SvO2) is the pulmonary arterial oxygen saturation (True) Explanation: Equates to DO2 (oxygen delivery) - VO2 (global oxygen consumption) SvO2 reflects the amount of oxygen not consumed by the tissues (True) oxygen saturation of venous blood from differing tissues is identical (False) Explanation: Varies depending on metabolic rate VO2 rises 10-15% for every 1°C rise in body temperature (True) Explanation: Sepsis and trauma also increase VO2 Question 5. Diagnostic criteria for the systemic inflammatory response syndrome (SIRS) include temperature > 38°C or < 36°C (True) Explanation: Sepsis may cause hypothermia as well as fever respiratory rate > 30/min (False) Explanation: > 20/min heart rate > 90/min (True) white cell count > 12 000 or < 4000/mm2 (True) PaCO2 < 4.3 kPa (True) Question 6. The following statements about shock syndromes are correct in severe hypovolaemia, a source of blood/fluid loss is invariably apparent clinically (False) Explanation: Bleeding may be internal in cardiogenic shock, the peripheries are characteristically warm (False) Explanation: Peripheral cyanosis is characteristic massive pulmonary embolism typically presents with shock (True) Explanation: Due to central vessel obstruction anaphylactic shock is associated with profound allergen-induced systemic vasoconstriction (False) Explanation: Vasodilatation occurs arteriovenous shunting is a significant contributory factor in septic shock (True) Explanation: Capillary damage and vasodilatation also occur Question 7. Acute circulatory failure with an elevated central venous pressure are typical findings in acute pancreatitis (False) Explanation: Hypovolaemic shock occurs massive pulmonary embolism (True) Explanation: Acute right ventricular failure ruptured ectopic pregnancy (False) acute right ventricular infarction (True) pericardial tamponade (True)

By A. H.

MCQs VIA WEB 2005 Question 8. The acute respiratory distress syndrome (ARDS) is characterised by maintenance of a normal PaO2 despite profound dyspnoea (False) Explanation: Hypoxaemia is a cardinal feature increased pulmonary compliance (False) Explanation: Compliance decreases a normal chest radiograph (False) Explanation: Diffuse infiltrates are typical greatly elevated pulmonary artery wedge pressure (False) Explanation: Typically normal or slightly elevated elevated right heart pressure (True) Explanation: Pulmonary hypertension is common Question 9. The expected effects of the following vasoactive drugs include nitroprusside-reduction in systemic vascular resistance (True) Explanation: Blood pressure typically falls epoprostenol (prostacyclin)-increased pulmonary vascular resistance (False) Explanation: Reduces PVR isoprenaline-sinus tachycardia (True) Explanation: And moderate increase in myocardial contractility dopamine-sinus bradycardia (False) Explanation: Usually tachycardia adrenaline (epinephrine)-increased splanchnic blood flow (False) Explanation: Typically declines Question 10. The following statements about mechanical respiratory support are correct cardiac output increases with positive end-expiratory pressure (PEEP) (False) Explanation: Cardiac output often falls PEEP helps correct V/Q mismatch (True) Explanation: Improves oxygenation in atelectatic areas continuous positive airways pressure (CPAP) requires intubation (False) Explanation: A tightly fitting face or nasal mask can be used the correct position of an endotracheal tube is 4 cm above the carina (True) intermittent ventilation is useful in the transition to non-assisted ventilation (True) Question 11. In the management of raised intracranial pressure (ICP) normal ICP is < 15 mmHg (True) Explanation: A sustained pressure > 30 mmHg suggests a poor prognosis cerebral perfusion pressure = mean systemic arterial pressure minus intracranial pressure (True) Explanation: Should be > 70 mmHg modest hyperglycaemia facilitates a decrease in ICP (False) Explanation: Glycaemic control should be strict temporary hyperventilation reduces ICP (True) Explanation: Target (PaCO2 of 4 kPa for 24 hours the patient should be nursed with 30° head-up tilt (True) Explanation: And avoid excessive neck flexion Module 5 (Chapter 5) Question 1. The histological features useful in distinguishing benign from malignant lesions include a lower nuclear to cytoplasmic ratio (False) Explanation: Increased the presence of aberrations in nuclear morphology (True) the number of cell mitoses (True) Explanation: Increases with cell proliferation rate the presence of cellular invasion into surrounding tissues (True) Explanation: Evidence of metastatic spread the number of mitochondria in the cell cytoplasm (False)

By A. H.

MCQs VIA WEB 2005 Question 2. Useful serum tumour markers associated with the following diseases include human chorionic gonadotrophin in testicular seminoma (False) Explanation: Useful in testicular germ cell tumours alpha fetoprotein in primary hepatocellular carcinoma (True) Explanation: And testicular germ cell tumours carcinoembryonic antigen in bronchial adenoma (False) Explanation: Metastatic colorectal carcinoma placental alkaline phosphatase in cervical carcinoma (False) Explanation: There are no useful serum markers for cervical carcinoma CA-125 in breast carcinoma (False) Explanation: Useful in ovarian carcinoma Question 3. The paraneoplastic syndromes listed below are typical of the following tumours inappropriate ADH-adenocarcinoma of lung (False) Explanation: Small-cell carcinoma prothrombotic tendency-pancreatic carcinoma (True) polymyositis-gastric carcinoma (True) Explanation: And ovarian and nasopharyngeal carcinoma myasthenia-like syndrome-small-cell anaplastic lung carcinoma (True) Explanation: Lambert-Eaton syndrome acanthosis nigricans-gastric carcinoma (True) Explanation: And other gastrointestinal malignancy Question 4. Malignant diseases that are potentially curable using combination chemotherapy include cervical cancer (True) squamous cell bronchial carcinoma (False) Explanation: Refractory to chemotherapy choriocarcinoma (True) Explanation: Also testicular teratoma oesophageal carcinoma (False) Explanation: Resistant soft tissue sarcoma (False) Explanation: Resistant Question 5. The following statements about chemotherapy are true methotrexate is an antifolate-blocking nucleotide synthesis (True) Explanation: An antimetabolite vincristine is an alkylating agent blocking DNA transcription (False) Explanation: A mitotic spindle poison doxorubicin is a plant alkaloid which disrupts mitotic spindles (False) Explanation: An antibiotic anticancer drug which acts primarily as a topoisomerase antagonist taxanes act as mitotic spindle poisons (True) Explanation: E.g. docetaxel melphalan is an alkylating agent which blocks DNA replication (True) Explanation: And also blocks DNA transcription Module 6 (Chapter 6) Question 1. In the management of pain in patients with malignant diseases analgesia is best prescribed on an 'as required' basis (False) Explanation: Should be given regularly NSAID therapy is particularly valuable in bone pain (True) Explanation: Affects prostaglandin metabolism controlled-release morphine has a 4-hour duration of action (False) Explanation: 12 hours respiratory depression is a common feature of prolonged opiate use (False) Explanation: But can occur in acute dosing

By A. H.

MCQs VIA WEB 2005 opiates are of no value in neuropathic pain (False) Explanation: But other agents may be more effective Question 2. The following drugs have clinically useful antiemetic properties haloperidol (True) domperidone (True) Explanation: Blocks dopaminergic receptors ondansetron (True) Explanation: 5HT3 receptor antagonist dexamethasone (True) Explanation: Given parenterally with chemotherapy etoposide (False) Explanation: Chemotherapeutic agent which causes nausea and vomiting Question 3. The following treatments may be of benefit in a patient with the following cancer-related symptoms co-danthrusate-constipation (True) gabapentin-nausea (False) Explanation: Used for neuropathic pain trazodone-insomnia (True) Explanation: A sedating antidepressant eicosapentanoic acid-anorexia (True) Explanation: If combined with a high-protein diet amitriptyline-neuropathic pain (True) Module 7 (Chapter 7) Question 1. Expected physiological changes associated with normal ageing include decreased calcium phosphate content per 100 g bone (False) Explanation: Bone mass declines (osteoporosis) but mineralisation is normal increased tissue sensitivity to insulin (False) Explanation: Reduced insulin sensitivity and glucose tolerance declines reduced numbers of pacing cells within the sinoatrial node (True) Explanation: Limits ability to mount a tachycardia increased glomerular filtration rate (GFR) (False) Explanation: Decreased number of nephrons, GFR and medullary function increased chest wall rigidity (True)

Question 2. Likely causes of recurrent falls in the elderly include accidental slips and trips (True) Explanation: Exacerbated by poor mobility postural hypotension (True) Explanation: Often drug-induced vasovagal syncope (False) Explanation: More common in the young Parkinson's disease (True) Explanation: Multiple factors involved acute myocardial infarction (False) Explanation: May present with a single fall but not recurrent falls Question 3. The following interventions may be of value in a patient with falls oral fludrocortisone (True) Explanation: May help postural hypotension occupational therapy home visit (True) Explanation: To improve environmental safety programme of exercise training (True) soft cervical collar (False) Explanation: May help vertebrobasilar insufficiency oral calcium and vitamin D (True)

By A. H.

MCQs VIA WEB 2005 Explanation: Help reduce the risk of fall fractures

Question 4. In the frailty syndrome the following domains are impaired musculoskeletal function (True) aerobic capacity (True) cognitive function (True) integrative neurological function (True) nutritional status (True) Module 8 (Chapter 8) Question 1. Aetiological factors in psychiatric illness include family history of psychiatric illness (True) Explanation: Rarely, a single gene disorder is identified parental loss or disharmony in childhood (True) Explanation: Especially physical or sexual abuse stressful life events and difficulties (True) Explanation: E.g. bereavement, redundancy, retirement chronic physical ill health (True) Explanation: Also acute severe physical illness social isolation (True) Explanation: Particularly lack of a close relationship

Question 2. Important factors in the assessment of mental state include appearance and behaviour (True) Explanation: Including motor retardation mood state (True) Explanation: E.g. suicidal ideation speech and thought content (True) Explanation: Paranoid, grandiose or depressive abnormal perceptions and beliefs (True) Explanation: Depersonalisation, illusions and hallucinations cognitive function (True) Explanation: Concentration, memory and orientation Question 3. The following psychiatric definitions are true delusions-abnormal perceptions of normal external stimuli (False) Explanation: Illusions illusions-unreasonably persistent, firmly held, false beliefs (False) Explanation: Delusions hallucinations-abnormal perceptions without external stimuli (True) Explanation: Suggest psychosis depersonalisation-perception of altered reality (True) Explanation: Often with derealisation phobia-abnormal fear leading to avoidance behaviour (True) Explanation: Typical pattern in neurosis Question 4. Diseases mimicking anxiety disorders include alcohol withdrawal (True) Explanation: Delirium may also occur hyperthyroidism (True) Explanation: Exclude biochemically hypoglycaemia (True) Explanation: Measure blood glucose temporal lobe epilepsy (True) Explanation: EEG may be necessary phaeochromocytoma (True)

By A. H.

MCQs VIA WEB 2005 Explanation: Rare-measure urinary catecholamines

Question 5. Factors associated with a higher suicide risk following attempted suicide include females aged < 45 years (False) Explanation: Older males self-poisoning rather than more violent methods of self-harm (False) Explanation: Self-poisoning is frequently parasuicidal absence of a suicide note or previous suicide attempts (False) Explanation: Suicide note often left and usually a history of previous attempts chronic physical or psychiatric illness (True) Explanation: And drug or alcohol misuse living alone and/or recently separated from partner (True) Explanation: Or bereavement Question 6. Cardinal elements in cognitive therapy include restructuring psychological conflicts and behaviour (False) Explanation: Undertaken in psychotherapy identification of negative patterns of automatic thoughts (True) Explanation: E.g. in depression awareness of connections between thoughts, mood and behaviour (True) Explanation: Altering thoughts may alter behaviour reorientation of negative views of the past, present and future (True) Explanation: And development of positive views personality assessment and transactional analysis (False) Explanation: Features of psychotherapy Question 7. The typical features of alcohol dependence include expansion of the drinking repertoire (False) Explanation: Narrowing of choices of alcoholic beverages increasing tolerance of alcohol (False) Explanation: Decreasing tolerance subjective compulsion to drink (True) use of alcohol to relieve withdrawal symptoms (True) Explanation: Classical recurrent withdrawal symptoms (True) Question 8. The typical features of depression include depressed mood for most of the day (True) Explanation: But diurnal variation may occur insomnia or hypersomnia (True) Explanation: Or early morning wakening loss of pleasure, self-esteem and hope (True) Explanation: 'Anhedonia'-loss of sense of enjoyment loss of energy, libido and interest (True) Explanation: Perhaps with other somatic symptoms psychomotor retardation and suicidal thoughts (True) Explanation: With delusions of worthlessness Question 9. Clinical features of generalised anxiety disorders include feelings of worthlessness and excessive guilt (False) Explanation: Suggest depression depersonalisation and derealisation (True) Explanation: May be seen in affective disorders feelings of apprehension and impending disaster (True) Explanation: With irritability breathlessness, dizziness, sweating and palpitation (True)

By A. H.

MCQs VIA WEB 2005 Explanation: Typical somatic symptoms claustrophobia and agoraphobia (False) Explanation: Features of phobic anxiety states Question 10. Typical features of anorexia nervosa include only adolescent girls are affected (False) Explanation: Either sex, rarely non-adolescent amenorrhoea or loss of libido > 3 months (True) Explanation: With avoidance of high-calorie foods weight loss > 25% or weight < 25% below normal (True) Explanation: In contrast to bulimia nervosa normal perception of body weight and image (False) Explanation: Emaciation is unrecognised by the patient progression to death in 20% (False) Explanation: In 5% Module 9 (Chapter 9) Question 1. In a normal 65 kg man, the following statements are true total body water is approximately 40 litres (True) Explanation: Relatively constant in health 70% of the total body water is intracellular (True) Explanation: Approximately 28 litres 75% of extracellular water is intravascular (False) Explanation: 25% intravascular, 75% interstitial sodium, bicarbonate and chloride ions are mainly intracellular (False) Explanation: Extracellular potassium, magnesium, phosphate and sulphate ions are mainly extracellular (False) Explanation: Intracellular

Question 2. Typical causes of hyponatraemia include diabetes insipidus (False) Explanation: But may be seen in the syndrome of inappropriate antidiuretic hormone (ADH) secretion hepatocellular failure (True) Explanation: Water retention exceeds sodium retention psychogenic polydipsia (True) Explanation: Increased total body water Cushing's syndrome (False) Explanation: But seen in adrenocortical insufficiency diuretic drug therapy (True) Explanation: Salt loss exceeds water loss Question 3. Predominant water depletion is a recognised complication of primary hyperparathyroidism (True) Explanation: Renal tubular insensitivity to ADH toxic confusional states (True) Explanation: Inadequate intake oesophageal carcinoma (True) Explanation: Inadequate intake lithium therapy (True) Explanation: Renal tubular insensitivity to ADH enteral feeding (True) Explanation: High solute load Question 4. The following statements about potassium balance are true 85% of the daily potassium intake is excreted in the urine (True) intracellular potassium ion concentrations are about 150 mmol/l (True) Explanation: Compared with extracellular concentrations of about 4 mmol/l

By A. H.

MCQs VIA WEB 2005 cellular uptake of potassium is enhanced by adrenaline and insulin (True) alkalosis predisposes to hyperkalaemia (False) the normal dietary potassium intake is about 100 mmol per day (True) Question 5. Hyperkalaemia is a recognised finding in severe untreated diabetic ketoacidosis (True) Explanation: Insulin promotes movement into the cells primary hypoadrenalism (True) Explanation: Impairment of secretion in the distal nephron rhabdomyolysis (True) Explanation: Increased tissue breakdown prostaglandin inhibitor therapy in renal impairment (True) Explanation: Especially if given with an ACE inhibitor angiotensin-converting enzyme (ACE) inhibitor therapy (True) Explanation: Avoid concurrent supplementation Question 6. The emergency treatment of severe hyperkalaemia should include dietary restriction of coffee and fruit juices (False) Explanation: But may be necessary to prevent recurrence parenteral dextrose and glucagon therapy (False) Explanation: Give parenteral dextrose and insulin parenteral calcium gluconate therapy (True) Explanation: Cardioprotective effect restoration of sodium and water balance (True) Explanation: Also correct metabolic acidosis if present with 1.26% sodium bicarbonate i.v. Calcium Resonium orally and/or rectally (True) Explanation: The resin binds potassium in exchange for calcium Question 7. Recognised causes of potassium depletion include metabolic alkalosis (True) Explanation: Renal tubular cell K+ concentration increased, excretion increased cardiac failure (True) Explanation: Secondary hyperaldosteronism corticosteroid treatment (True) Explanation: Mineralocorticoid-like effect renal tubular acidosis (True) Explanation: Primary or secondary tubular defect; also occurs with activation of renin and angiotensin amiloride diuretic therapy (False) Explanation: Causes hyperkalaemia by an effect on the distal convoluted tubules Question 8. Metabolic acidosis would be an expected finding in chronic alveolar hyperventilation (False) Explanation: Chronic respiratory alkalosis acute insulin deficiency (True) Explanation: Diabetic ketoacidosis acute inflammatory polyneuropathy (Guillain-Barré syndrome) (False) Explanation: Acute respiratory acidosis due to alveolar hypoventilation failure of distal renal tubular hydrogen ion secretion (True) Explanation: Distal (type I) renal tubular acidosis methanol poisoning (True) Question 9. Metabolic alkalosis may be caused by hyperventilation (False) Explanation: Respiratory alkalosis aspiration of gastric contents (True) Explanation: Or vomiting mineralocorticoid deficiency (False)

By A. H.

MCQs VIA WEB 2005 Explanation: Can produce mild acidosis excessive liquorice ingestion (True) Explanation: Due to excessive mineralocorticoid activity diuretic therapy (True) Explanation: And hypokalaemia Question 10. Magnesium deficiency is a cause of confusion, depression and epilepsy (True) Explanation: And tremor and choreiform movements usually due to prolonged vomiting and diarrhoea (True) Explanation: Also from chronic diuretic therapy found in uncontrolled diabetes mellitus and alcoholism (True) Explanation: Excess losses in the urine found in primary hyperparathyroidism and hyperaldosteronism (True) Explanation: Including secondary hyperaldosteronism best treated with oral magnesium sulphate (False) Explanation: Very poorly absorbed orally Module 10 (Chapter 10) Question 1. A healthy daily diet for a slim man with a physical job should include 1500 kcal (8.4 MJ) (False) Explanation: About 11.3 MJ (2700 kcal) 60% of total energy requirements as carbohydrate (True) Explanation: 55-75% no less than 10 g salt per day (False) Explanation: No more than 6 g/day 35 g of dietary fibre (True) Explanation: 27-40 g/day no more than 10% of total energy requirements as fat (False) Explanation: 15-30%

Question 2. Recognised medical complications of weight gain include osteoporosis (False) Explanation: Bone density increases rheumatoid arthritis (False) Explanation: Osteoarthritis gallstones (True) Explanation: Often asymptomatic type 2 diabetes mellitus (True) Explanation: With insulin resistance hyperlipidaemia (True) Explanation: And coronary artery disease Question 3. Ideal weight-reducing diets in the treatment of moderate obesity should provide no more than 2.5 MJ (600 kcal) per day (False) Explanation: Aim to reduce intake by no more than 2.5 MJ (600 kcal) per day achieve a theoretical weight loss of at least 2 kg per week (False) Explanation: 0.5 kg per week (2.5 MJ or 600 kcal deficit/day = 17.15 MJ or 4200 kcal/week = 0.6 kg human tissue) aim to achieve a weight loss of 10% (True) Explanation: Sufficient to achieve a significant improvement in health be part of a multiple risk factor intervention (True) Explanation: E.g. cessation of smoking reduce carbohydrate intake much more than total fat intake (False) Explanation: Fat restriction < 50 g/day (calorific values fat = 38 KJ or 9 kcal/g, CHO = 17 KJ or 4 kcal/g) Question 4. The benefits of a sustained 10 kg weight reduction in the obese include fall in the blood pressure of 10 mmHg (systolic) and 20 mmHg (diastolic) (True)

By A. H.

MCQs VIA WEB 2005 reduction in total mortality of 20-25% (True) reduction in fasting glucose of 15% (False) Explanation: 50% reduction in total cholesterol of 50% (False) Explanation: Reduction in total cholesterol of 10% reduction in high-density lipoprotein cholesterol of 8% (False) Explanation: Increases by 8% Question 5. Drug therapies known to increase appetite and body weight include orlistat (False) Explanation: Has a role in promoting weight loss fenfluramine (False) Explanation: But side-effects preclude use amitriptyline (True) fluoxetine (False) Explanation: Stimulates satiety and can help some patients lose weight sibutramine (False) Explanation: Can support weight loss Question 6. The function of the main lipoproteins include the following chylomicrons transport mainly cholesterol (False) Explanation: Mainly triglycerides; not present in the normal fasting plasma very low-density lipoprotein transports endogenous triglycerides (True) Explanation: VLDL is synthesised in the liver and is the precursor of LDL low-density lipoprotein transports cholesterol (True) Explanation: Generated from VLDL in the blood stream high-density lipoprotein transports cholesterol from the peripheral tissues to the liver (True) low-density lipoprotein is important for the excretion of cholesterol and is cardioprotective (False) Explanation: HDL aids cholesterol excretion and is cardioprotective Question 7. Common causes of secondary hyperlipidaemia include chronic renal failure (True) Explanation: Increases triglycerides and VLDL but decreases HDL diabetes mellitus (True) Explanation: Increases triglycerides and VLDL but decreases HDL hyperthyroidism (False) Explanation: Hypothyroidism increases cholesterol and LDL alcohol misuse (True) Explanation: Increases triglycerides, VLDL and HDL thiazide diuretics (True) Question 8. In the classification of hyperlipidaemias, the following findings are typical chylomicronaemia in types I and V (True) Explanation: Risk of pancreatitis with both types I and V but no atherogenic risk hypertriglyceridaemia in types III, IV and V (True) Explanation: Triglycerides variably abnormal in all except type IIa hypercholesterolaemia in types II, III and IV (True) Explanation: And all are associated with increased atherosclerosis tendon xanthomata in type IIa hypercholesterolaemia (True) Explanation: And premature coronary atherosclerosis defective LDL catabolism and receptor binding in type V hyperlipidaemia (False) Explanation: Defective LDL receptor gene is typical of type II familial hypercholesterolaemia Question 9. The actions of the lipid-lowering drugs include the following the statins inhibit HMG CoA reductase and reduce cholesterol synthesis (True) Explanation: Increase LDL catabolism the statins increase plasma LDL and triglycerides (False)

By A. H.

MCQs VIA WEB 2005 Explanation: Decrease plasma LDL and cholesterol nicotinic acid increases lipolysis and lowers HDL (False) Explanation: Decreases lipolysis and plasma triglycerides but increases plasma HDL fibrates increase VLDL lipolysis (True) Explanation: Decrease plasma triglycerides and plasma LDL and increase plasma HDL colestipol diverts hepatic cholesterol synthesis into an increased bile acid production (True) Explanation: Like colestyramine, it blocks bile acid reabsorption in the gut Question 10. Clinical features of protein-energy malnutrition in adults include a body mass index of between 20 and 22 (False) Explanation: BMI < 16. N.B. BMI is calculated from the formula weight (kg) ÷ height2 (m) oedema in the absence of hypoalbuminaemia (True) Explanation: 'Famine oedema' nocturia, cold intolerance and diarrhoea (True) Explanation: And weakness, amenorrhoea or impotence skin depigmentation, hair loss and covert infection (True) Explanation: Adolescents may maintain hair growth cerebral atrophy and sinus tachycardia (False) Explanation: Brain weight is preserved; bradycardia is the rule Question 11. The clinical features of protein-energy malnutrition in children include marked muscle-wasting and abdominal distension in marasmus (True) Explanation: And absence of oedema weight loss more than growth retardation in marasmus (True) Explanation: Weight < 60% standard for age hepatic steatosis and hypoproteinaemic oedema in kwashiorkor (True) Explanation: With low plasma lipids desquamative dermatosis, stomatitis and anorexia in marasmus (False) Explanation: Features of kwashiorkor associated zinc deficiency in kwashiorkor (True) Explanation: Contributing to dermatosis Question 12. Vitamin A is a fat-soluble vitamin (True) Explanation: A, D, E, and K are the fat-soluble vitamins present as retinol in carrots and certain green vegetables (False) Explanation: Occurs as retinol in animal produce and as carotene in plants the treatment of choice in xerophthalmia and keratomalacia (True) Explanation: Both conditions are the result of vitamin A deficiency and lead to blindness associated with teratogenicity if administered in pregnancy (True) present in high concentrations in fish liver oils (True) Explanation: Present as retinol Question 13. Vitamin D is present in high concentrations in dairy products (False) Explanation: Some margarines are fortified is non-essential in the diet given adequate sunlight exposure (True) Explanation: But less efficiently produced in old age like vitamin A is stored mainly in the liver (False) Explanation: But metabolism partly occurs in the liver is converted from cholecalciferol to 1,25-dihydroxycholecalciferol (True) Explanation: 1-alpha hydroxylation occurs in the kidney and 25-hydroxylation in the liver enhances calcium absorption by the induction of specific enterocyte transport proteins (True) Explanation: And stimulates osteoclast proliferation Question 14. Deficiency of the following B vitamins is associated with the disorders listed below niacin-pellagra (True)

By A. H.

MCQs VIA WEB 2005 Explanation: Dermatitis, diarrhoea and dementia pyridoxine-isoniazid-induced peripheral neuropathy (True) Explanation: Add to anti-tuberculosis regimens using isoniazid pyridoxine-haemolytic anaemia (False) Explanation: Sideroblastic anaemia may respond riboflavin-angular stomatitis (True) Explanation: And also nasolabial seborrhoea riboflavin-cheilosis (True) Explanation: Also seen in niacin deficiency Question 15. In the classification of acute and non-acute porphyrias ä-aminolaevulinic acid synthetase activity is increased in all porphyrias (True) Explanation: Rate-limiting step in biosynthesis of haem porphobilinogen deaminase activity is reduced in acute porphyrias (True) Explanation: Porphobilinogen accumulates neuropsychiatric features are typical of the non-acute porphyrias (False) Explanation: Typical of acute porphyria photosensitivity is typical of the acute porphyrias (False) Explanation: Typical of the non-acute porphyrias variegate porphyria and coproporphyria are mixed porphyrias (True) Explanation: Both are hepatic porphyrias Question 16. Disorders associated with amyloid deposition include familial Mediterranean fever (True) Explanation: Reactive (AA) amyloidosis bronchiectasis (True) chronic haemodialysis (True) Alzheimer's disease (True) Explanation: Also the spongiform encephalitides rheumatoid arthritis (True) Explanation: Reactive AA amyloidosis Module 11 (Chapter 11) Question 1. In humans somatic cell nuclei contain 22 pairs of homologous autosomes (True) Explanation: In addition there are 2 X chromosomes in females and 1 X and 1 Y in males gamete nuclei are haploid with a single X or Y chromosome (True) Explanation: In contrast to somatic cell nuclei which are diploid the haploid male cell (sperm) contains 22 autosomes and a Y chromosome (False) Explanation: The haploid male cell (sperm) may contain an X or a Y chromosome the long and short arms of a chromosome meet at the telomere (False) Explanation: Centromere both X chromosomes in females are genetically active (False) Explanation: One X chromosome is inactive and appears as the Barr body in the nucleus Question 2. In the chromosomal disorders aneuploidy is the addition or loss of a chromosome (True) Explanation: The most common form of numerical chromosome aberration deletions arise from the loss of a segment of a chromosome (True) the majority of affected conceptions result in miscarriage (True) Explanation: Liveborn frequency is 0.6% identical deletions produce the same effects whether inherited from father or mother (False) Explanation: Gene expression can be affected by the parental origin of the abnormal chromosome translocation is the exchange of segments between chromosomes (True) Explanation: No genetic material is lost Question 3. In polycystic kidney disease

By A. H.

MCQs VIA WEB 2005 inheritance is commonly autosomal dominant (True) hepatic cysts commonly coexist (True) intracranial aneurysms are present in 70% of patients (False) Explanation: Incidence = 10% DNA testing is useful in determining the presence of PKD1 mutations (False) renal ultrasound after the age of 18 is the best screening test (True) Explanation: Detects > 95% of individuals Question 4. The karyotype of a normal male is 45, XY (False) Explanation: 46, XY female with Down's syndrome is 46, XX, -21 (False) Explanation: 47, XX, +21 male with Klinefelter's syndrome is 47, XXY (True) female with Turner's syndrome is 45, XO (True) male with trisomy 18 (Edwards' syndrome) is 47, XX, +18 (False) Explanation: 47, XY, +18 Question 5. The following conditions arise as a result of the noted genetic abnormality haemochromatosis-DNA point mutation (True) Explanation: HFE gene cystic fibrosis-DNA point mutation (False) Explanation: Three base-pair deletion Huntington's disease-triplet repeat expansions (True) Explanation: On 4p16 Down's syndrome-chromosomal deletion (False) Explanation: Chromosomal aneuploidy (trisomy 21) DiGeorge syndrome-chromosomal microdeletion (True) Explanation: The commonest microdeletion syndrome Question 6. In autosomal dominant inheritance affected individuals are usually heterozygotes (True) affected individuals rarely have an affected parent (False) Explanation: Parent is almost always affected male offspring are more likely to be affected than female (False) Explanation: An equal chance unaffected children of an affected parent have a 50% chance of transmitting the condition (False) Explanation: Unaffected children are free of the mutant gene clinical disease is always found in genetically affected individuals (False) Explanation: Some affected individuals are clinically normal-'non-penetrance' Question 7. Given a husband with haemophilia and his unaffected wife none of their sons will be affected (True) Explanation: Absence of male to male transmission is a key feature of all X-linked inheritance all of their daughters will carry the haemophilic gene (True) a daughter with Turner's syndrome may also have haemophilia (True) Explanation: If the X chromosome is inherited from the father all of his sisters will be carriers (False) Explanation: 50% of his sisters will be carriers and 50% normal his maternal grandfather could have had haemophilia (True) Explanation: All the female children of an affected grandfather would carry the gene Question 8. The following disorders are caused by single gene disorders cleft lip (False) Explanation: Multifactorial disorder sickle-cell anaemia (True) Explanation: Autosomal recessive

By A. H.

MCQs VIA WEB 2005 retinitis pigmentosa (True) cystic fibrosis (True) Explanation: Autosomal recessive familial hypercholesterolaemia (True) Explanation: Autosomal dominant Module 12 (Chapter 12) Question 1. In the normal human heart the atrioventricular (AV) node is usually supplied by the left circumflex coronary artery (False) Explanation: Supplied by the right coronary artery in 90% â1-adrenoceptors mediate chronotropic responses (True) Explanation: These receptors also mediate inotropic responses pulmonary artery systolic pressure normally varies between 90 and 140 mmHg (False) Explanation: Varies between 15 and 30 mmHg in health the annulus fibrosus aids conduction of impulses from the atria to the ventricles (False) Explanation: Restricts electrical connections between the atria and ventricles to the AV node cardiac output is the product of heart rate and ventricular end-diastolic volume (False) Explanation: The product of heart rate and ventricular stroke volume Question 2. With regard to cardiovascular physiology cardiac output is approximately 10 l/min at rest (False) Explanation: Measured in l/min (70/min × 700 ml = 5 l/min) coronary blood vessels are innervated only by the parasympathetic nerves (False) Explanation: Also by sympathetic-both have dominant vasodilating effect intracoronary acetylcholine provokes vasoconstriction if atheroma is present (True) Explanation: But endothelial-derived relaxing factor (EDRF)-mediated vasodilatation occurs in normal vessels an atheromatous coronary lesion restricts blood flow during exercise if > 40% (False) Explanation: Must be > 70% bradykinin is an endogenous vasodilator (True) Explanation: Others include adenosine, prostaglandins and nitric oxide Question 3. In the normal electrocardiogram (ECG) the PR interval is measured from the end of the P wave to the beginning of the R wave (False) Explanation: Measured from the start of the P wave to the start of the R wave each small square represents 40 milliseconds at a standard paper speed of 25 mm/sec (True) the heart rate is 75 per minute if the R-R interval measures 4 cm (True) Explanation: Heart rate = 1500/R-R interval (mm) or 300/R-R interval (cm) R waves become progressively larger from leads V1-V6 (True) Explanation: Reflecting the electrical dominance of the left ventricle the P wave represents sinoatrial node depolarisation (False) Explanation: Represents atrial depolarisation Question 4. In the normal ECG depolarisation proceeds from epicardium to endocardium (False) Explanation: Proceeds from endocardium to epicardium depolarisation away from the positive electrode produces a positive deflection (False) Explanation: Produces a negative deflection depolarisation of the interventricular septum is recorded by the Q wave in V5 and V6 (True) Explanation: Absent in left bundle branch block (BBB) the aVR lead = right arm positive with respect to the other limb leads (True) Explanation: Hence the predominant S wave as depolarisation moves away from aVR voltage amplitudes vary with the thickness of cardiac muscle (True) Explanation: An aid to the diagnosis of left ventricular hypertrophy Question 5. In the investigation of patients with suspected heart disease the normal upper limit for the cardiothoracic ratio (CTR) on chest radiograph is 0.75 (False) Explanation: The CTR should not be > 0.5

By A. H.

MCQs VIA WEB 2005 a negative exercise ECG excludes the diagnosis of ischaemic heart disease (False) Explanation: False negative tests occur in 15-20% a 'step-up' in oxygen saturation at cardiac catheterisation suggests an intracardiac shunt (True) Doppler echocardiography reliably assesses pressure gradients between cardiac chambers (True) Explanation: Pressure gradients can be extrapolated from measuring intracardiac flow velocities radionuclide blood pool scanning accurately quantifies left ventricular function (True) Explanation: Ejection fraction is usually measured using this technique Question 6. The pain of myocardial ischaemia is typically induced by exercise and relieved by rest (True) Explanation: Typical chest pain occurring at rest does not exclude myocardial ischaemia radiates to the neck but not the jaw (False) Explanation: May also radiate to the shoulders, arms or back rarely lasts longer than 10 seconds after resting (False) Explanation: Rapid resolution is atypical-pain usually lasts for minutes is easily distinguished from oesophageal pain (False) Explanation: Oesophageal pain may mimic angina-precipitation by swallowing may be useful invariably worsens as exercise continues (True) Explanation: Can disappear as exercise continues-'second wind' effect ('walk through' angina) Question 7. In a patient with central chest pain at rest intrascapular radiation suggests the possibility of aortic dissection (True) Explanation: As does a tearing quality postural variation in pain suggests the possibility of pericarditis (True) Explanation: As does variation with respiration chest wall tenderness is a typical feature of Tietze's syndrome (True) Explanation: The syndrome is a form of costochondritis relief of pain by nitrates excludes an oesophageal cause (False) Explanation: And oesophageal pain may also be precipitated by exercise features of autonomic disturbance are specific to cardiac pain (False) Explanation: May occur in severe pain from any cause Question 8. In the treatment of cardiac failure associated with acute pulmonary oedema controlled oxygen therapy should be restricted to 28% oxygen in patients who smoke (False) Explanation: High-flow oxygen in concentrations > 35% should be administered morphine reduces vasoconstriction and dyspnoea (True) furosemide (frusemide) therapy given intravenously reduces preload and afterload (True) nitrates should be avoided if the systolic blood pressure < 140 mmHg (False) Explanation: Can safely be used with systolic pressures > 110 mmHg ACE inhibitors decrease the afterload but increase the preload (False) Explanation: Both preload and afterload are reduced Question 9. Recognised features of severe cardiac failure include tiredness (True) Explanation: Due to severe reduction in cardiac output weight loss (True) Explanation: 'Cardiac cachexia'-however, weight gain due to oedema is more common epigastric pain (True) Explanation: Due to hepatic and gastrointestinal congestion nocturia (True) Explanation: Diuresis is induced by adopting the supine position nocturnal cough (True) Explanation: A manifestation of pulmonary congestion Question 10. With regard to angiotensin-converting enzyme (ACE) inhibitors ACE inhibitors reduce the conversion of angiotensinogen to angiotensin I (False) Explanation: Angiotensin I to angiotensin II

By A. H.

MCQs VIA WEB 2005 enalapril has a longer half-life than lisinopril (False) Explanation: Converted to enalaprilat in the liver cough is a less common side-effect of ACE inhibitors than angiotensin II antagonists (False) Explanation: Cough is a more common side-effect of ACE inhibitors-probably due to bradykinin accumulation first-dose hypotension occurs less commonly in patients pretreated with diuretics (False) Explanation: Omitting diuretics pretreatment minimises risk treatment is of no benefit until symptomatic left ventricular systolic dysfunction has developed (False) Question 11. In the management of chronic heart failure ACE inhibitor therapy reduces subsequent hospitalisation rates (True) Explanation: And reduces mortality coagulation is impaired and thromboembolic risk therefore declines (False) Explanation: Other factors favouring thromboembolism outweigh this effect salt restriction may be beneficial (True) â-adrenoceptor antagonists (â-blockers) should always be avoided (False) Explanation: There is evidence that they reduce mortality in some patients digoxin is only of benefit if atrial fibrillation coexists (False) Explanation: Reduces need for hospitalisation Question 12. Complications of systemic hypertension include retinal microaneurysms (False) Explanation: Arteriolar thickening, irregularity and tortuosity are detectable aortic dissection (True) renal artery stenosis (True) Explanation: Hypertension predisposes to atheroma formation lacunar strokes of the internal capsule (True) subdural haemorrhage (False) Explanation: Hypertension predisposes to intracerebral and subarachnoid haemorrhage Question 13. Recognised causes of secondary hypertension include persistent ductus arteriosus (False) Explanation: In contrast to coarctation of the aorta primary hyperaldosteronism (True) Explanation: Conn's syndrome acromegaly (True) oestrogen-containing oral contraceptives (True) Explanation: And pregnancy thyrotoxicosis (True) Question 14. In the treatment of systemic hypertension treatment has more effect on the risk of stroke than the risk of coronary heart disease (CHD) (True) Explanation: 30% reduction in stroke, 20% in CHD thresholds for treatment are higher in the elderly (False) Explanation: Absolute risk is higher treatment is less likely to be of benefit if cardiac or renal disease is present (False) there are no proven benefits of therapy in patients aged over 70 years (False) Explanation: Good evidence of efficacy in the elderly moderation of alcohol consumption is likely to improve blood pressure control (True) Explanation: Excessive consumption of alcohol is a significant factor in 10-15% of hypertensives Question 15. Important explanations for hypertension refractory to medical therapy include poor compliance with drug therapy (True) inadequate drug therapy (True) Explanation: Common particularly in asymptomatic patients phaeochromocytoma (True) Explanation: But rare primary hyperaldosteronism (True) Explanation: Conn's syndrome is suggested by a hypokalaemic alkalosis

By A. H.

MCQs VIA WEB 2005 renal artery stenosis (True) Explanation: May also develop during follow-up Question 16. The auscultatory findings listed below are associated with the following phenomena third heart sound-opening of mitral valve (False) Explanation: Occurs in mid-diastole due to rapid ventricular filling varying intensity of first heart sound-atrioventricular dissociation (True) Explanation: Due to variations in stroke volume soft first heart sound-mitral stenosis (False) Explanation: Typically loud in mitral stenosis reversed splitting of second heart sound-left bundle branch block (True) Explanation: Due to delayed closure of the aortic valve compared with the pulmonary valve fourth heart sound-atrial fibrillation (False) Explanation: Coincides with atrial contraction and hence cannot occur in atrial fibrillation Question 17. Syncope followed by facial flushing suggests a tachyarrhythmia (False) Explanation: Suggests episodic bradycardia- Adams-Stokes attacks without warning suggests a vasovagal episode (False) Explanation: Nausea and lightheadedness typically precede vasovagal attacks on exercise is a typical feature of mitral regurgitation (False) Explanation: Exertional syncope is a feature of severe aortic stenosis can sometimes be treated by â-blockers (True) may be a feature of Parkinson's disease (True) Explanation: Due to severe postural hypotension Question 18. Atrial fibrillation (AF) is present in 10% of the elderly population over the age of 75 years (True) usually readily converted to permanent sinus rhythm using DC cardioversion (False) Explanation: Underlying structural heart disease is common and promotes the recurrence of AF associated with an annual stroke risk of 5% if structural heart disease is present (True) Explanation: Warfarin therapy reduces the annual risk to about 1.5% a common presenting feature of the sick sinus syndrome (True) Explanation: Episodes of sinus bradycardia or sinus arrest may coexist making drug therapy difficult usually associated with a ventricular rate < 100/min before treatment (False) Explanation: Indicates concomitant AV nodal disease, a common finding in elderly patients Question 19. In cardiac arrest a sharp blow to the praecordium may be useful (True) Explanation: In witnessed arrest only asystole is the commonest finding on ECG (False) Explanation: Ventricular fibrillation is the commonest underlying arrhythmia a normal ECG may suggest profound hypovolaemia (True) Explanation: A cause of 'electromechanical' dissociation if cardioversion fails, intracardiac adrenaline (epinephrine) should be given (False) Explanation: Adrenaline (epinephrine) should be given intravenously the compression to ventilation ratio should be 15:2 (True) Question 20. Atrial tachycardia is typically associated with 1:1 AV conduction (False) Explanation: 2:1, 3:1 or variable an atrial rate of 300/min (False) Explanation: Atrial rate is 140-220/min presence of P waves identical to those found during sinus rhythm (False) Explanation: An ectopic atrial focus with abnormal P waves digoxin toxicity and intracellular potassium depletion (True) bizarre broad QRS complexes on ECG (False)

By A. H.

MCQs VIA WEB 2005 Explanation: QRS complexes are usually narrow

Question 21. Typical features of the Wolff-Parkinson-White (WPW) syndrome include tachyarrhythmias resulting from re-entry phenomenon (True) Explanation: Re-entrant circuit includes AV node and the accessory bundle ventricular pre-excitation via an accessory AV pathway (True) atrial fibrillation with a ventricular response of > 160/min (True) Explanation: Consider WPW in young patients with episodes of atrial fibrillation ECG between bouts showing prolonged PR interval with narrow QRS complexes (False) Explanation: PR interval is shortened and a delta wave is seen in the QRS complex useful therapeutic response to verapamil or digoxin (False) Explanation: Differential effects on the normal and anomalous pathways can increase cardiac rate Question 22. In ventricular tachycardia (VT) underlying cardiac disease is usually present (True) Explanation: Often ischaemic heart disease amiodarone is useful in the prevention of recurrent episodes (True) Explanation: A class III agent a shortened QT interval on ECG predisposes to recurrent episodes (False) Explanation: A prolonged QT interval predisposes to recurrent VT carotid sinus massage usually slows the cardiac rate transiently (False) Explanation: No effect on cardiac rate complicated by acute cardiac failure, cardioversion should be avoided (False) Explanation: The treatment of choice in acute heart failure with VT Question 23. The following statements about atrioventricular block are true first-degree block is usually asymptomatic (True) the PR interval is fixed in Mobitz type I second-degree block (False) Explanation: Fixed PR = Mobitz type II; variable PR (Wenckebach's phenomenon) = Mobitz type I decreasing PR intervals suggest Wenckebach's phenomenon (False) Explanation: PR intervals gradually increase irregular cannon waves in the jugular venous pressure suggest complete heart block (True) Explanation: Due to AV dissociation the QRS complex in complete heart block is always broad and bizarre (False) Explanation: Can be narrow if the escape rhythm arises from within the bundle of His Question 24. In the classification of anti-arrhythmic drugs, the following statements are true class I agents inhibit the fast sodium channel (True) Explanation: E.g. lidocaine (lignocaine)-like drugs class II agents are â-adrenoceptor antagonists (True) class III agents prolong the action potential (True) Explanation: E.g. amiodarone class IV agents inhibit the slow calcium channel (True) Explanation: E.g. verapamil, nifedipine many anti-arrhythmic agents have actions in more than one class (True) Explanation: E.g. sotalol and amiodarone Question 25. The cardiac drugs listed below are associated with the following adverse effects digoxin-acute confusional state (True) Explanation: And lidocaine (lignocaine) therapy verapamil-constipation (True) Explanation: Calcium channel-blocking effect on smooth muscle amiodarone-photosensitivity (True) propafenone-corneal microdeposits (False) Explanation: An adverse effect of amiodarone therapy

By A. H.

MCQs VIA WEB 2005 lidocaine (lignocaine)-convulsions (True)

Question 26. Amiodarone therapy prolongs the plateau phase of the action potential (True) Explanation: In common with other class III drugs potentiates the effect of warfarin (True) is useful in the prevention of ventricular but not supraventricular tachycardia (False) Explanation: Effective in both may cause corneal deposits (True) Explanation: But no effect on vision has a significant negative inotropic action (False) Explanation: Can be safely used in heart failure Question 27. Digoxin shortens the refractory period of conducting tissue (False) Explanation: Prolongs the refractory period of conducting tissue; shortens it in cardiac muscle usually converts atrial flutter to sinus rhythm (False) Explanation: Often converts atrial flutter to atrial fibrillation is excreted primarily by the kidney (True) is a class II anti-arrhythmic (False) is a recognised cause of ventricular arrhythmias (True) Explanation: Increases myocardial excitability Question 28. The risk of developing clinical evidence of coronary artery disease is increased by exogenous oestrogen use in postmenopausal females (False) Explanation: Risk is decreased by oestrogen therapy diminished by stopping smoking (True) Explanation: Effect is measurable within 6 months of stopping reduced by the moderate consumption of alcohol (True) Explanation: Not more than 21 units per week increased in hyperfibrinogenaemia (True) increased by hypercholesterolaemia but not hypertriglyceridaemia (False) Explanation: Both confer increased risk Question 29. In the investigation of suspected angina pectoris the resting ECG is usually abnormal (False) Explanation: Usually normal exercise-induced elevation in blood pressure indicates significant ischaemia (False) Explanation: Fall in blood pressure suggests significant ischaemia a normal ECG during exercise excludes angina pectoris (False) Explanation: False negatives may occur coronary angiography is only indicated if an exercise tolerance test (ETT) is abnormal (False) Explanation: Useful in patients with convincing history but normal ETT physical examination is of no clinical value (False) Explanation: Important to exclude anaemia and valvular stenosis

Question 30. In the treatment of patients with angina pectoris aspirin reduces the frequency of anginal attacks (False) Explanation: But it improves the prognosis glyceryl trinitrate is equally effective when swallowed as when taken sublingually (False) Explanation: Extensive first-pass hepatic metabolism calcium antagonists may cause peripheral oedema (True) Explanation: Common adverse effect tissue levels of nitrates must be consistently high for maximum therapeutic effect (False) Explanation: A nitrate-free period should be achieved

By A. H.

MCQs VIA WEB 2005

â-blockers are more effective than other anti-anginal agents (False) Explanation: Nitrates, calcium antagonists and â-blockers are all equally efficacious Question 31. The clinical features of acute myocardial infarction include nausea and vomiting (True) Explanation: Due to activation of the autonomic nervous system breathlessness and angor animi (True) hypotension and peripheral cyanosis (True) Explanation: Suggest a large infarct sinus tachycardia or sinus bradycardia (True) absence of any symptoms or physical signs (True) Explanation: 15% of infarcts are believed to be clinically 'silent' Question 32. In the treatment of acute myocardial infarction aspirin given within 6 hours of onset reduces the mortality (True) Explanation: 30% reduction in short-term mortality streptokinase therapy reduces infarct size and mortality by > 25% (True) Explanation: The earlier thrombolysis is given, the better the results diamorphine is better given intravenously than by any other route (True) Explanation: Intramuscular injections predispose to haematoma immediate calcium channel blocker therapy reduces the early mortality rate (False) Explanation: Similarly, nitrate therapy has no effect on the early mortality rate mobilisation should be deferred until cardiac enzymes normalise (False) Explanation: Mobilisation should begin on day 2 in the absence of cardiac failure Question 33. Drug therapies which improve the long-term prognosis after myocardial infarction include aspirin (True) Explanation: Vascular events are reduced by 25% nitrates (False) calcium antagonists (False) ACE inhibitors (True) Explanation: Limit infarct expansion â-blockers (True) Explanation: Reduce mortality by 25% Question 34. The following statements about the prognosis of acute myocardial infarction are true 50% of all deaths occur within the first 24 hours (True) Explanation: Of which half occur within the first 20 minutes, often before help arrives stress and social isolation adversely affect the prognosis (True) Explanation: Rehabilitation programmes can be helpful the 5-year survival is 75% for those who leave hospital (True) late mortality is determined by the extent of myocardial damage (True) Explanation: Limiting infarct size improves prognosis in hospital mortality for those aged over 75 years is over 25% (True) Explanation: Five times greater than < 55 years of age Question 35. In intermittent claudication due to atherosclerosis pain is typically relieved by rest and elevation of the leg (False) Explanation: Rest relieves but elevation worsens pain the commonest cause of death is lower limb gangrene (False) Explanation: Myocardial infarction or stroke pedal pulses are often still palpable (False) Explanation: Anaemia or diabetes may produce claudication without loss of the pulses exercise which causes pain should be avoided (False) Explanation: Exercise promotes growth of the collateral circulation the risk of progression is lessened by warfarin (False) Explanation: Anticoagulation is unhelpful

By A. H.

MCQs VIA WEB 2005 Question 36. Characteristic features of aortic dissection include haemopericardium (True) Explanation: Type A aneurysms acute paraparesis (True) Explanation: Due to infarction of the spinal cord interscapular back pain (True) Explanation: The pain is often described as 'tearing' early diastolic murmur (True) Explanation: Type A aneurysms pleural effusion (True) Explanation: Haemothorax Question 37. In patients with significant mitral stenosis the mitral valve orifice is reduced from 5 cm2 to about 1 cm2 (True) Explanation: First symptoms appear at valve areas of around 2 cm2 a history of rheumatic fever or chorea is elicited in over 90% of patients (False) Explanation: Only in 50% of patients left atrial enlargement cannot be detected on the chest radiograph (False) Explanation: Produces a double right heart border and an enlarged left atrial appendage the risk of systemic emboli is trivial in sinus rhythm (False) Explanation: Embolic risk over 10 years is 10% compared with 35% if atrial fibrillation is present mitral balloon valvuloplasty is not advisable if there is also significant mitral regurgitation (True) Explanation: Mitral regurgitation is a contraindication Question 38. Recognised features of chronic mitral regurgitation include soft first heart sound and loud third heart sound (True) presentation with signs of right ventricular failure (True) Explanation: Due to pulmonary hypertension left ventricular dilatation (True) a pansystolic murmur and hyperdynamic displaced apex beat (True) atrial fibrillation requiring anticoagulation (True) Question 39. Clinical features suggesting aortic stenosis include late systolic ejection click (False) Explanation: Early systolic click implies the stenosis is valvular narrow pulse pressure (True) heaving apex beat (True) Explanation: Implies left ventricular hypertrophy syncope associated with angina (True) loud second heart sound (False) Explanation: Quiet S2 if the valve is heavily calcified and immobile Question 40. Disorders associated with aortic regurgitation include ankylosing spondylitis (True) Explanation: Also Reiter's disease and psoriatic arthritis Marfan's syndrome (True) Explanation: Due to cystic medial necrosis syphilitic aortitis (True) Explanation: Typically affects the ascending aorta persistent ductus arteriosus (False) Explanation: Produces the 'machinery murmur' congenital bicuspid aortic valve (True) Question 41. In infective endocarditis streptococci and staphylococci account for over 80% of cases (True) Explanation: Streptococcus viridans alone accounts for 30-40% of cases

By A. H.

MCQs VIA WEB 2005 left heart valves are more frequently involved than right heart valves (True) normal cardiac valves are not affected (False) Explanation: About 30% have no identifiable predisposing cardiac lesion glomerulonephritis usually occurs due to immune complex disease (True) a normal echocardiogram excludes the diagnosis (False) Explanation: Vegetations may be too small to be detected Question 42. Central cyanosis in infancy is an expected finding in the following congenital heart diseases persistent ductus arteriosus (False) Explanation: With a left to right shunt transposition of the great arteries (True) Explanation: Usually due to a shunt through a ventricular septal defect coarctation of the aorta (False) Explanation: No shunt Fallot's tetralogy (True) Explanation: Right to left shunt through a ventricular septal defect atrial septal defect (False) Explanation: Left to right shunt Question 43. The following statements about persistent ductus arteriosus are true blood usually passes from the pulmonary artery to the aorta (False) Explanation: This only happens if the shunt reverses the onset of heart failure usually occurs in early infancy (False) Explanation: Typically presents with a murmur in an otherwise healthy infant a systolic murmur around the scapulae is typical (False) Explanation: Continuous 'machinery' murmur is typical (systolic and diastolic) shunt reversal is indicated by cyanosis of the lower limbs (True) Explanation: A rare sign prophylactic antibiotic therapy to prevent endocarditis is indicated (True) Question 44. Typical clinical features of coarctation of the aorta include an association with a bicuspid aortic valve (True) Explanation: Frequently coexists cardiac failure developing in male adolescents (False) Explanation: Cardiac failure is more likely to develop in infancy palpable collateral arteries around the scapulae (True) Explanation: A useful but unusual finding rib notching on chest radiograph associated with weak femoral pulses (True) Explanation: Rib notching is due to enlarged collateral vessels ECG showing right ventricular hypertrophy (False) Explanation: Left (not right) ventricular hypertrophy develops Question 45. In atrial septal defect the lesion is usually of secundum type (True) Explanation: Due to a patent fossa ovalis the initial shunt is right to left (False) Explanation: Occurs late, and rarely splitting of the second heart sound increases in expiration (False) Explanation: Splitting is fixed and wide the ECG typically shows right bundle branch block (True) Explanation: In primum defect there may be left axis deviation surgery should be deferred until shunt reversal occurs (False) Explanation: Surgery is indicated when the pulmonary/systolic flow ratio is > 3:2 Question 46. In small ventricular septal defects the murmur is confined to late systole (False)

By A. H.

MCQs VIA WEB 2005 Explanation: It is pansystolic the heart is usually enlarged (False) Explanation: No cardiomegaly there is a risk of infective endocarditis (True) Explanation: Prophylaxis is indicated surgical repair before adolescence is usually indicated (False) Explanation: Surgery is only indicated if right-sided pressures rise most patients are asymptomatic (True) Explanation: Symptomless murmur is a frequent presentation Question 47. Dilated (congestive) cardiomyopathy is usually idiopathic (True) associated with pathognomonic ECG changes (False) Explanation: ECG changes are non-specific a recognised complication of HIV infection (True) associated with chronic alcohol misuse (True) caused by Coxsackie A infection (True) Explanation: And influenza, HIV and others Question 48. Clinical features compatible with hypertrophic cardiomyopathy include family history of sudden death (True) Explanation: 50% of cases are autosomal dominant angina pectoris and exertional syncope (True) Explanation: Mimicking aortic stenosis jerky pulse and heaving apex beat (True) murmurs suggesting both aortic stenosis and mitral regurgitation (True) Explanation: Left ventricular outflow obstruction and secondary mitral regurgitation soft or absent second heart sound (False) Explanation: Suggests calcific aortic stenosis Question 49. Typical features of acute pericarditis include chest pain identical to that of myocardial infarction (False) Explanation: Sharp pain worsened by posture and movement a friction rub that is best heard in the axilla in mid-expiration (False) Explanation: Localisation and character vary greatly ST elevation on the ECG with upward concavity (True) Explanation: In contrast to ischaemia elevation of the serum creatine kinase (False) Explanation: May occur in pericarditis complicating acute myocardial infarction ECG changes that are only seen in the chest leads (False) Explanation: Widespread ECG changes Question 50. In pericardial tamponade high amplitude QRS complexes are a typical ECG feature (False) Explanation: Low amplitude the systemic arterial pressure falls dramatically on inspiration (True) Explanation: This is pulsus paradoxus echocardiography is the definitive investigation (True) an effusion > 250 ml must be present before detrimental haemodynamic effects ensue (False) Explanation: As little as 75-100 ml a normal chest radiograph excludes the diagnosis (False) Explanation: But the cardiac shadow usually appears globular Module 13 (Chapter 13) Question 1. Typical chest findings in a large right pleural effusion include normal chest expansion (False) Explanation: Expansion is reduced on the affected side

By A. H.

MCQs VIA WEB 2005 dull percussion note (False) Explanation: Stony dull absent breath sounds (True) vocal resonance decreased (True) Explanation: As is tactile vocal fremitus pleural friction rub (False) Question 2. Hypercapnia is a typical feature of pulmonary embolism (False) Explanation: Hyperventilation unless embolism is massive severe chest wall injury (True) Explanation: With type II respiratory failure salicylate intoxication (False) Explanation: Hyperventilation pulmonary fibrosis (False) Explanation: Hyperventilation and type I failure severe chronic bronchitis (True) Explanation: Type II respiratory failure may ensue Question 3. Typical chest findings in right lower lobe collapse include decreased chest expansion (True) Explanation: On the affected side stony dull percussion note (False) Explanation: Implies effusion bronchial breath sounds (False) Explanation: Diminished or absent breath sounds decreased vocal resonance (True) Explanation: As for vocal fremitus crepitations (False) Explanation: No specific added sounds Question 4. The following statements about pulmonary function tests are true over 80% of vital capacity can normally be expelled in 1 second (False) Explanation: More than 70% is normal the transfer factor is measured using inspired oxygen (False) Explanation: Carbon monoxide is used residual volume is increased in chronic bronchitis and emphysema (True) Explanation: The lungs are hyperinflated analysis of flow volume curves is of value in suspected central airflow obstruction (True) peak expiratory flow rates accurately reflect the severity of restrictive lung disorders (False) Explanation: They measure obstructive ventilatory defects Question 5. In a patient with severe acute breathlessness a normal arterial PaO2 invariably suggests psychogenic hyperventilation (False) Explanation: The patient may have a metabolic acidosis pulsus paradoxus is pathognomonic of acute asthma (False) Explanation: Also found in pericardial tamponade a normal chest radiograph excludes pulmonary embolism (False) Explanation: Although subtle changes are frequently present the extremities are typically cool and sweaty in left ventricular failure (True) Explanation: With basal pulmonary crepitations left bundle branch block is strongly suggestive of pulmonary embolism (False) Explanation: Right bundle branch block or S1Q3T3 pattern Question 6. The following are recognised causes of haemoptysis tuberculosis (True) chronic obstructive pulmonary disease (False)

By A. H.

MCQs VIA WEB 2005 Explanation: Another cause should be sought bronchiectasis (True) Explanation: May be massive Goodpasture's syndrome (True) Explanation: With associated renal disease mitral stenosis (True) Explanation: With pulmonary hypertension Question 7. A pleural effusion with a pleural fluid:serum protein ratio of > 0.5 would be typical of congestive cardiac failure (CCF) (False) Explanation: Transudate in CCF renal failure (False) subphrenic abscess (True) Explanation: Most frequently on the right pneumonia (True) Explanation: With polymorphonuclear leucocytes nephrotic syndrome (False) Explanation: Severe hypoalbuminaemia produces transudates Question 8. The sleep apnoea syndrome is associated with obesity (True) Explanation: Found in two-thirds of patients and may be associated with alcohol misuse an increased risk of road traffic accidents (True) Explanation: Increased threefold due to day-time sleepiness nocturnal restlessness apparent to the patient (False) a good response to inhaled bronchodilator therapy administered at bedtime (False) Explanation: Ineffective; continuous positive airway pressure (CPAP) may be effective nocturnal hypotension (False) Explanation: Typically episodic hypertension Question 9. The following disorders characteristically produce type I respiratory failure kyphoscoliosis (False) Explanation: Typically type II failure Guillain-Barré polyneuropathy (False) Explanation: Respiratory muscle paralysis causes type II failure acute respiratory distress syndrome (ARDS) (True) Explanation: Arterial PCO2 is typically normal extrinsic allergic alveolitis (True) Explanation: Ventilatory drive is usually maintained inhaled foreign body in a major airway (False) Explanation: Causes acute type II failure-asphyxia Question 10. In the treatment of acute COPD exacerbations associated with type II respiratory failure the inspired oxygen content should be at least 40% (False) Explanation: Controlled oxygen therapy at about 24-28% is usual nebulised doxapram improves small airways obstruction (False) Explanation: A central respiratory stimulant flapping tremor is a sensitive indicator of hypercapnia (False) Explanation: It may be absent-blood gases are vital corticosteroid therapy is usually contraindicated (False) Explanation: May help relieve bronchospasm BIPAP may be valuable if pH falls (True) Explanation: But not all patients are candidates for such support Question 11. The following statements about oxygen are true at sea level, the pressure of oxygen in inspired air is approximately 20 kPa (True) Explanation: PaO2 declines with altitude

By A. H.

MCQs VIA WEB 2005 chronic domiciliary oxygen therapy is indicated only when PaO2 is < 6 kPa (False) Explanation: Indicated when PaO2 < 7.3 breathing air dissolved oxygen contributes to tissue oxygenation in anaemia (True) Explanation: Also in other situations when Hb is maximally saturated oxygen toxicity in adults can produce retrolental fibroplasia (False) Explanation: Occurs only in neonates central cyanosis unresponsive to 100% oxygen indicates right-to-left shunting of > 20% (True) Explanation: Such shunts may be extra- or intrapulmonary Question 12. In the management of chronic obstructive pulmonary disease influenza immunisation should only be offered once (False) Explanation: Immunisation should be offered yearly long-term antibiotic treatment decreases the frequency of exacerbations (False) Explanation: This encourages drug resistance regular inhaled steroids are of no proven value (True) supplemental oxygen during air travel is necessary if the resting PaO2 < 9 kPa (True) Explanation: PaO2 will be < 7 kPa in such a patient at altitude long-term controlled oxygen therapy improves symptoms but not the prognosis (False) Explanation: Survival has been demonstrated to improve Question 13. Typical findings in severe chronic obstructive pulmonary disease include elevation of the jugular venous pressure (True) Explanation: A feature of right heart failure tracheal descent on inspiration (True) Explanation: Tracheal 'tug' due to mediastinal descent indrawing of the intercostal muscles (True) Explanation: A sign of hyperinflation contraction of the scalene muscles (True) Explanation: And other accessory respiratory muscles pursed lip breathing (True) Explanation: Decreases air trapping Question 14. Typical pathological features of asthma include eosinophilic bronchial infiltrate (True) Explanation: And T lymphocytes increased airway macrophages (True) mucus gland hyperplasia (True) Explanation: May contribute to development of fixed airways obstruction epithelial shedding (True) Explanation: A recognised feature in fatal asthma in particular T lymphocyte activation and cytokine release (True) Question 15. In the management of chronic persistent asthma inhaled â2-agonist use more than once per day is an indication for inhaled steroid therapy (True) Explanation: Typically low-dose steroids sodium cromoglicate therapy is often useful as an alternative to inhaled steroids in adults (False) Explanation: But may be valuable in childhood patients taking high doses of inhaled steroids should use a large-volume spacer device (True) Explanation: Reduces oropharyngeal and gastric deposition leukotriene antagonists are valuable substitutes for inhaled steroids (False) Explanation: Use in addition to steroids and â2-agonist anticholinergic agents should be avoided (False) Explanation: May be valuable Question 16. Features compatible with severe acute asthma include pulse rate = 120 per minute (True) Explanation: But bradycardia may occur in life-threatening attacks

By A. H.

MCQs VIA WEB 2005 peak expiratory flow (PEF) rate = < 70% of expected (False) Explanation: Usually < 50% of expected PEF pulsus paradoxus (True) Explanation: But may diminish in severe attacks arterial PaO2 = 14 kPa while breathing air (False) Explanation: PaO2 < 8 kPa in life-threatening attacks arterial PaCO2 = 5 kPa (True) Explanation: PaCO2 may remain normal until the late stages Question 17. The initial management of severe acute asthma should include 24% oxygen delivered by a controlled flow mask (False) Explanation: High concentration, high flow should be used salbutamol 5 mg by inhalation (True) Explanation: Intravenous â2-adrenoceptor agonists can also be used ampicillin 500 mg orally and sodium cromoglicate 10 mg by inhalation (False) Explanation: Of no proven value in acute attacks hydrocortisone 200 mg i.v. or prednisolone 40 mg orally (True) Explanation: Maintain corticosteroid therapy for at least 7 days in severe attacks arterial blood gas analysis and chest radiograph (True) Explanation: Exclude pneumothorax and ventilatory failure Question 18. Typical clinical features of bronchiectasis include chronic cough with scanty sputum volumes (False) Explanation: Copious sputum production recurrent pleurisy (True) Explanation: Recurrent pneumonia haemoptysis (True) Explanation: Secondary to inflammatory bronchial change finger clubbing (True) crepitations on auscultation (True) Explanation: In the presence of large amounts of secretions Question 19. Cystic fibrosis is associated with an incidence of 1 in 2500 live births (True) Explanation: The commonest severe autosomal recessive disorder in Caucasians a decreased sweat sodium concentration (False) Explanation: Increased sweat sodium concentration male infertility (True) Explanation: Due to failure of development of the vas deferens abnormal lung function at birth (False) Explanation: It is normal; hence prospect for gene therapy recurring pneumococcal pulmonary infections (False) Explanation: Pseudomonas and staphylococcal sepsis Question 20. In pneumonia, the following features are classically associated with the specific organisms noted erythema nodosum and Mycoplasma pneumoniae (True) hyponatraemia and Legionella pneumoniae (True) contact with sick birds and Klebsiella pneumoniae (False) Explanation: Chlamydia psittaci abscess formation and Staphylococcus aureus (True) haemolytic anaemia and Streptococcus pneumoniae (False) Explanation: Mycoplasma Question 21. A non-pneumococcal pneumonia should be considered if the clinical features include respiratory symptoms preceding systemic upset by several days (False) Explanation: The converse is typical of 'atypical' organisms lobar consolidation (False)

By A. H.

MCQs VIA WEB 2005 rigors (False) the absence of a neutrophil leucocytosis (True) Explanation: Leucopenia can occur in severe pneumococcal infection palpable splenomegaly (True) Explanation: Rare in pneumococcal disease Question 22. The following features suggest a poor prognosis in pneumonia diastolic blood pressure of 90 mmHg (False) Explanation: < 60 mmHg confusion (True) respiratory rate of 20 breaths per minute (False) Explanation: > 30/min blood urea of 9 mmol/l (True) Explanation: > 7 mmol/l white cell count of 3000 × 109/l (True) Explanation: < 4000 × 109/l Question 23. Typical features of primary tuberculosis include a sustained pyrexial illness (False) Explanation: Typically symptomless caseation within the regional lymph nodes (True) Explanation: Mediastinal, cervical or mesenteric nodes are most frequently involved bilateral hilar lymphadenopathy on chest radiograph (False) Explanation: Suggests sarcoidosis erythema nodosum (True) Explanation: Can also accompany pulmonary sarcoid pleural effusion with a negative tuberculin skin test (False) Explanation: A hypersensitivity phenomenon typically associated with positive tuberculin test Question 24. Recognised complications of post-primary tuberculosis include aspergilloma (True) Explanation: Superinfection of a cavity amyloidosis (True) Explanation: Associated with chronic immune stimulation massive haemoptysis (True) bronchiectasis (True) Explanation: Suggested by chronic productive cough paraplegia (True) Explanation: Due to vertebral or paraspinal abscess formation Question 25. In the treatment of post-primary pulmonary tuberculosis combination drug therapy is always indicated (True) Explanation: Minimises resistance and reduces duration of treatment sputum remains infectious for at least 4 weeks after the onset of therapy (False) Explanation: Patients can be regarded as non-infectious after 2 weeks of therapy at least 12 months' daily therapy is required for 100% effectiveness (False) Explanation: 6- and 9-month regimes are of proven efficacy isoniazid and pyrazinamide do not cross the blood-brain barrier (False) Explanation: Hence their great value in the treatment of tuberculous meningitis treatment failure is invariably due to multiple drug resistance (False) Explanation: More often due to non-compliance Question 26. Recognised adverse reactions to antituberculous drugs include streptomycin-renal failure (False) Explanation: Causes vestibular disturbance and deafness isoniazid-hypothyroidism (False) Explanation: Polyneuropathy

By A. H.

MCQs VIA WEB 2005 rifampicin-optic neuritis (False) Explanation: Ethambutol causes optic neuritis pyrazinamide-hepatitis (True) Explanation: And rifampicin ethambutol-vestibular neuronitis (False) Explanation: Streptomycin causes this Question 27. Pulmonary infection with Aspergillus fumigatus is a recognised cause of the following bullous emphysema (False) Explanation: No association mycetoma (True) Explanation: Usually in a tuberculous cavity necrotising pneumonitis (True) Explanation: A severe, rapidly progressive illness bronchopulmonary eosinophilia (True) Explanation: Typically with wheeze, pulmonary infiltrates and peripheral eosinophilia extrinsic allergic alveolitis (False) Explanation: Type III and IV immune responses Question 28. Bronchial carcinoma accounts for 10% of all male deaths from cancer (False) Explanation: 50% of all male deaths from malignant disease typically presents with massive haemoptysis (False) Explanation: Streaking of sputum with blood in a smoker is more typical histology reveals adenocarcinoma in 50% of patients (False) Explanation: Squamous 35%, adenocarcinoma 30% is associated with asbestos exposure (True) Explanation: As is mesothelioma is 40 times more common in smokers than in non-smokers (True) Explanation: Smoking is the major aetiological factor Question 29. Non-metastatic manifestations of bronchial carcinoma include cerebellar degeneration (True) Explanation: With ataxia and nystagmus myasthenia (True) Explanation: Eaton-Lambert syndrome gynaecomastia (True) Explanation: Usually bilateral polyneuropathy (True) Explanation: Usually distal sensorimotor dermatomyositis (True) Explanation: Skin rash and proximal myopathy Question 30. The following are contraindications to surgical resection in bronchial carcinoma distant metastases (True) malignant pleural effusion (True) FEV1 < 0.8 litres (True) ipsilateral mediastinal lymphadenopathy (False) Explanation: But contralateral nodes are a contraindication oesophageal involvement (True) Question 31. Mediastinal opacification on the chest radiograph is a typical feature of thymoma (True) Explanation: May be associated with myasthenia gravis retrosternal goitre (True) Explanation: Anterior superior mediastinum Pancoast tumour (False)

By A. H.

MCQs VIA WEB 2005 Explanation: Pulmonary apical mass hiatus hernia (True) Explanation: A retrocardiac opacity neurofibroma (True) Explanation: Can be multiple Question 32. The following statements about sarcoidosis are true pulmonary lesions typically cavitate (False) Explanation: Caseating granulomata (e.g. TB) are associated with cavitation the tuberculin tine test is usually positive (False) Explanation: Typically negative erythema marginatum is a characteristic finding (False) Explanation: Erythema nodosum is the typical skin lesion spontaneous resolution is unusual (False) Explanation: The normal course in stage I and stage II disease hypercalcaemia suggests skeletal involvement (False) Explanation: Due to increased vitamin D sensitivity Question 33. Typical features of cryptogenic fibrosing alveolitis include hypercapnic respiratory failure (False) Explanation: Typically type I respiratory failure positive antinuclear and rheumatoid factors (True) Explanation: With or without evidence of connective tissue disease finger clubbing (True) recurrent wheeze and haemoptysis (False) Explanation: Dyspnoea, dry cough and crackles increased carbon monoxide transfer factor (False) Explanation: Reduced Question 34. Clinical features compatible with a diagnosis of extrinsic allergic alveolitis include expiratory rhonchi and sputum eosinophilia (False) Explanation: Acute dyspnoea without wheeze is characteristic dry cough, dyspnoea and pyrexia (True) Explanation: Influenza-like symptoms may exist end-inspiratory crepitations (True) Explanation: Typically bilateral FEV1/FVC ratio of 50% (False) Explanation: Airway obstruction is absent positive serum precipitin tests (True) Explanation: May also be positive in healthy subjects Question 35. The following statements about asbestos-related disease are true pleural plaques usually progress to become mesotheliomas (False) Explanation: Often calcify pleural effusions are always malignant (False) Explanation: But malignancy should be excluded finger clubbing and basal crepitations suggest pulmonary asbestosis (True) Explanation: Although cryptogenic fibrosing alveolitis is possible the FEV1/FVC ratio is typically decreased (False) Explanation: A restrictive not an obstructive ventilatory defect mesothelioma can only be reliably diagnosed at thoracotomy (False) Explanation: Seldom necessary Question 36. Characteristic features of pulmonary eosinophilia include an association with ascariasis and microfilariasis (True) Explanation: And Toxocara infestation eosinophilic pneumonia without peripheral blood eosinophilia (False)

By A. H.

MCQs VIA WEB 2005 Explanation: Eosinophilia is necessary for the diagnosis prominent asthmatic features (False) Explanation: Wheeze may be absent induction by exposure to drugs (True) Explanation: Imipramine, phenylbutazone or others opacities on chest radiograph (True) Explanation: Pulmonary infiltrates and eosinophilia Question 37. Clinical features characteristic of massive pulmonary embolism include central and peripheral cyanosis (True) Explanation: With profound hypoxaemia pleuritic chest pain and haemoptysis (False) Explanation: Suggests pulmonary infarction breathlessness and syncope (True) Explanation: Non-specific tachycardia and elevated jugular venous pressure (True) Explanation: Non-specific Q waves in leads I, II and aVL on ECG (False) Explanation: Classical ECG pattern is S1, Q3, T3 Question 38. Typical features of an empyema thoracis include bilateral effusions on chest radiograph (False) Explanation: Typically unilateral a fluid level on chest radiograph suggesting a bronchopleural fistula (True) Explanation: Or a recent diagnostic aspiration persistent pyrexia despite antibiotic therapy (True) Explanation: Suggests lung abscess, antibiotic resistance or hypersensitivity recent abdominal surgery (True) Explanation: Perhaps complicating subphrenic infection bacteriological culture of the organism despite preceding antibiotic therapy (False) Explanation: Frequently sterile post-antibiotic therapy Question 39. The following statements about spontaneous pneumothorax are true breathlessness and pleuritic chest pain are often present (True) Explanation: A small pneumothorax may be asymptomatic bronchial breathing is audible over the affected hemithorax (False) Explanation: Diminished or absent breath sounds absent peripheral lung markings on chest radiograph suggest tension (False) Explanation: Mediastinal shift suggests tension surgical referral is required if there is a bronchopleural fistula (True) Explanation: Pleurectomy may also be necessary pleurodesis should be considered for recurrent pneumothoraces (True) Explanation: Particularly if bilateral Question 40. The following are causes of an elevated hemidiaphragm recurrent laryngeal nerve paralysis (False) Explanation: Phrenic nerve paralysis surgical lobectomy (True) subphrenic abscess (True) severe pleuritic pain (True) Explanation: But underlying pathology should be sought chronic severe asthma (True) Module 14 (Chapter 14) Question 1. The following statements about renal physiology in health are correct each kidney comprises approximately 1 000 000 nephrons (True) the kidneys receive approximately 5% of the cardiac output (False)

By A. H.

MCQs VIA WEB 2005 Explanation: 25% of the cardiac output variations in the calibre of afferent and efferent arterioles control the filtration pressure (True) the glomerular capillaries are supplied by the afferent arterioles (True) the kidney produces erythropoietin (True) Question 2. Microscopic haematuria would be an expected finding in urinary tract infection (True) renal papillary necrosis (True) Explanation: Risk factors include diabetes mellitus, chronic non-steroidal anti-inflammatory drug (NSAID) misuse and alcoholism membranous glomerulonephritis (False) Explanation: Typically proteinuria infective endocarditis (True) Explanation: Associated with a mesangiocapillary glomerulonephritis renal infarction (True) Explanation: May be frank haematuria Question 3. Urinary protein excretion in Bence Jones proteinuria is readily detectable by stick tests (False) Explanation: Immunoelectrophoresis required > 3.5 g/day is invariably due to glomerular disease (True) Explanation: Often with oedema and hypoalbuminaemia is greater in the night than during the day (False) Explanation: Greater when the person is upright-'orthostatic proteinuria' can be assessed by the albumin/creatinine ratio in a single sample (True) Explanation: Easier to undertake than 24-hour collection in early diabetic nephropathy typically comprises albumin predominantly (True) Explanation: Microalbuminuria is a sensitive predictor Question 4. Typical features of the nephrotic syndrome include bilateral renal angle pain (False) Explanation: Typically painless generalised oedema and pleural effusions (True) Explanation: Transudates hypoalbuminaemia and proteinuria > 3.5 g/day (True) Explanation: Serum albumin concentration < 30 g/l and urinary protein > 3.5 g/day hypertension and polyuria (False) Explanation: But may occur in chronic renal failure urinary sodium concentration > 50 mmol/l (False) Explanation: Marked sodium retention-urinary sodium < 10 mmol/l Question 5. The following findings would support a diagnosis of pre-renal rather than established acute renal failure oliguria < 700 ml per day (False) Explanation: Pre-renal acute failure is not always oliguric urine/plasma urea ratio > 10:1 (True) Explanation: Indicating preservation of renal medullary function a urinary osmolality > 600 mOsm/kg (True) Explanation: Indicating preservation of renal medullary function a urinary sodium concentration < 20 mmol/l (True) Explanation: Indicating preservation of renal medullary function hypertension rather than hypotension (False) Explanation: Suggests primary renal disease Question 6. Typical causes of rapidly progressive glomerulonephritis include post-infectious glomerulonephritis (True) systemic vasculitis (True) Explanation: Causes focal necrotising glomerulonephritis

By A. H.

MCQs VIA WEB 2005 Goodpasture's disease (True) IgA nephropathy (True) Explanation: Including Henoch-Schönlein purpura membranous glomerulonephritis (False) Question 7. Typical biochemical features of chronic renal failure include polycythaemia (False) Explanation: Anaemia is atypical hypophosphataemia (False) Explanation: Hyperphosph ataemia hypercalcaemia (False) Explanation: Hypocalcaemia metabolic acidosis (True) Explanation: Resulting in hyperpnoea impaired urinary concentrating ability (True) Explanation: Hence polyuria; urinary diluting ability also impaired Question 8. Complications of chronic renal failure include macrocytic anaemia (False) Explanation: Typically normocytic or microcytic peripheral neuropathy (True) Explanation: Can improve with haemodialysis bone pain (True) Explanation: Renal osteodystrophy with osteomalacia pericarditis (True) Explanation: Even haemorrhagic pericarditis with tamponade metabolic alkalosis (False) Explanation: Chronic metabolic acidosis Question 9. The features of Alport's syndrome include an autosomal dominant mode of inheritance (False) Explanation: Autosomal recessive and X-linked modes degeneration of the glomerular basement membrane (True) mutation of genes encoding type IV collagen (True) Explanation: Located at Xq22 association with progressive chronic renal failure (True) Explanation: Second most common inherited form of chronic renal failure association with high-tone deafness (True) Explanation: Characteristic feature preceding severe sensorineural deafness Question 10. Characteristic features of minimal change nephropathy are occurrence in adults usually follows an acute infection (False) Explanation: Usually children; accounts for 25% of nephrotic syndrome in adults marked mesangial cell proliferation on renal biopsy (False) Explanation: Minor or absent nephrotic syndrome with unselective proteinuria (False) Explanation: Selective proteinuria hypertension and microscopic haematuria (False) Explanation: Suggest an alternative cause progression to chronic renal failure in patients not responding to corticosteroid therapy (False) Explanation: Renal function is otherwise unimpaired Question 11. In the treatment of minimal change nephropathy therapy should be deferred pending renal biopsy in childhood (False) Explanation: Diagnosis in children rarely requires histological confirmation diuretics should be avoided to minimise the risk of renal impairment (False) Explanation: Useful in management of oedema

By A. H.

MCQs VIA WEB 2005 high-dose steroids usually control proteinuria (True) immunosuppressant therapy is indicated for frequent relapses (True) Explanation: E.g. cyclophosphamide impaired renal function commonly develops in the long term (False) Explanation: Rarely, even in relapsing disease Question 12. Typical features of acute post-infectious glomerulonephritis include subendothelial immune deposits on the glomerular basement membrane (True) bacterial rather than viral infections (True) Explanation: Especially haemolytic streptococci; rare in the UK diffuse glomerular involvement (True) recurrent haemoptysis (False) Explanation: Suggests Goodpasture's disease a poor prognosis when the disease occurs in childhood (False) Explanation: Usually resolves spontaneously, especially in children Question 13. Typical features of acute interstitial nephritis (AIN) include skin rashes, arthralgia and fever (False) Explanation: Less than 30% of patients with drug-induced AIN have features of generalised hypersensitivity peripheral blood eosinophilia (False) Explanation: Eosinophilia occurs in 30% in the peripheral blood and 70% in the urine renal biopsy evidence of an eosinophilic interstitial nephritis (True) Explanation: And neutrophil or monocytic infiltrate renal impairment typically follows withdrawal of the drug (False) Explanation: Typically resolves onset following antibiotic or anti-inflammatory drug therapy (True) Explanation: E.g. penicillin or naproxen Question 14. Causes of chronic interstitial nephritis include Sjögren's syndrome (True) Explanation: Also associated with sarcoidosis and systemic lupus erythematosus Wilson's disease (True) Explanation: And other heavy metal poisoning sickle-cell nephropathy (True) chronic transplant rejection (True) analgesic misuse (True) Explanation: Resulting in medullary ischaemia Question 15. Chronic pyelonephritis in adults accounts for the majority of patients with chronic renal failure (CRF) in the UK (False) Explanation: Diabetes mellitus is the commonest cause is usually attributable to vesicoureteric reflux in childhood (True) Explanation: Other aetiological factors may also be important has pathognomonic histopathological features on renal biopsy (False) Explanation: Similar to chronic interstitial nephritis is usually associated with demonstrable ureteric reflux (False) Explanation: Reflux is often no longer demonstrable in adulthood producing hypotension should be treated with oral sodium salts (True) Explanation: As a result of a 'salt-losing' nephropathy Question 16. The clinical features of adult polycystic renal disease include an autosomal recessive mode of inheritance (False) Explanation: Autosomal dominant cystic disease of the liver and pancreas (True) Explanation: But liver function tests are normal renal angle pain and haematuria (True) Explanation: And hypertension and urinary tract infection

By A. H.

MCQs VIA WEB 2005 aortic and mitral regurgitation (True) Explanation: Common but rarely severe aneurysms of the circle of Willis (True) Explanation: 10% will have a subarachnoid haemorrhage Question 17. Characteristic features of renal tubular acidosis (RTA) include normal anion gap (True) Explanation: Anion gap = plasma (Na+ + K+) - (Cl- + HCO3-) normally < 15 mmol/l hyperchloraemic acidosis (True) Explanation: increased chloride preserves anion gap inappropriately high urinary pH > 5.4 (True) Explanation: Even in presence of systemic acidosis decreased glomerular filtration rate (GFR) (False) Explanation: GFR is normal normocytic normochromic anaemia (False) Explanation: No features of uraemia Question 18. The typical features of acute pyelonephritis in adult females include normal anatomy of the urinary tract (True) Explanation: But ureteric obstruction may be a predisposing factor vomiting, rigors and renal angle tenderness (True) Explanation: With loin or epigastric pain pyuria (True) peritubular neutrophil infiltration (True) loin pain and fullness in the flank (False) Explanation: Suggest perinephric abscess Question 19. In the treatment of renal calculi anuria indicates the need for urgent surgical intervention (True) Explanation: Suggests total obstruction the urine should be alkalinised if the stone is radio-opaque (False) Explanation: Acidification with ammonium chloride may benefit bendroflumethiazide (bendrofluazide) increases urinary calcium excretion (False) Explanation: Decreases urinary calcium excretion by 30% in hypercalciuric patients allopurinol increases urinary urate excretion in gouty patients (False) Explanation: Decreases urinary urate and may reduce oxalate stone formation renal pelvic stones require removal at open surgery (False) Explanation: Fragmentation by lithotripsy and endoscopic removal is possible Question 20. Recognised features of renal carcinoma include persistent fever (True) Explanation: Occurs in 20% and is due to increased interleukin release bone metastases (True) Explanation: Typically osteolytic metastases haematuria (True) Explanation: Due to blood clot or direct tumour obstruction of ureter polycythaemia (True) Explanation: Erythropoietin secretion serum alphafetoprotein in high titre (False) Explanation: Suggests hepatoma Question 21. The typical features of benign prostatic hypertrophy include peak incidence in the age-group 40-60 years (False) Explanation: Aged over 60 years acute urinary retention and haematuria (True) Explanation: Sometimes precipitated by urinary tract infection a response to á-adrenoceptor blocker therapy in > 50% of patients (True)

By A. H.

MCQs VIA WEB 2005 elevated serum prostate specific antigen (False) Explanation: Suggests prostatic carcinoma hard, nodular prostatic enlargement on rectal examination (False) Explanation: Suggests prostatic carcinoma Question 22. Typical features of prostatic carcinoma include slowly progressive obstructive uropathy (True) Explanation: As also benign prostatic disease presentation with urinary frequency and nocturia (True) Explanation: Or haematuria preservation of the normal anatomy on digital rectal examination (False) Explanation: Hard with obliteration of median furrow local spread along the lumbosacral nerve plexus (True) Explanation: And may involve ureters osteolytic rather than osteosclerotic bone metastases (False) Explanation: Osteosclerotic metastases Question 23. Characteristic features of testicular tumours include testicular pain in seminoma of the testis (False) Explanation: Typically painless secretion of alphafetoprotein and chorionic gonadotrophin by teratomas (True) Explanation: Helps in the assessment of treatment response absence of distant metastases (False) peak incidence after the age of 60 years (False) Explanation: Peak incidence aged 25-34 years seminomas are both radio- and chemosensitive (True) Explanation: Chemotherapy is given if disease is widespread Module 15 (Chapter 15) Question 1. Type 1 insulin-dependent diabetes mellitus (IDDM) is associated with 'insulitis'-T lymphocyte infiltrate of the islets of Langerhans (True) Explanation: Patchy distribution in pancreas feeding of cows' milk in early infancy (True) Explanation: Cross-reactivity of antibodies to bovine serum albumin destruction of over 70% of pancreatic beta cells (True) Explanation: Symptoms occur only when 70-90% of beta cells have been destroyed concordance rates in monozygotic twins of 35% (True) Explanation: Indicating the importance of environmental factors possession of HLA antigens DR3 and DR4 (True) Explanation: Linkage with HLA-DQA1 and DQB1 genes encoded on the short arm of chromosome 6

Question 2. The following statements about type 2 diabetes mellitus (NIDDM) are true there is clear evidence of disordered autoimmunity in most patients with type 2 diabetes mellitus (False) Explanation: In contrast to type 1 diabetes mellitus monozygotic twins show almost 100% concordance for type 2 diabetes mellitus (True) Explanation: Compare 35% concordance in monozygotic twins with type 1 diabetes mellitus patients with type 2 diabetes mellitus typically exhibit hypersensitivity to insulin (False) Explanation: Variable insulin resistance obesity predisposes to type 2 diabetes mellitus in genetically susceptible individuals (True) Explanation: Especially if combined with underactivity hypertension, hypercholesterolaemia and hyperinsulinaemia often coexist (True) Explanation: Syndrome X (Reaven's syndrome) associated with macrovascular disease Question 3. Secondary diabetes mellitus is associated with thiazide diuretic therapy (True) Explanation: Hypokalaemic alkalosis impairs insulin secretion haemochromatosis (True)

By A. H.

MCQs VIA WEB 2005 Explanation: Pancreatic fibrosis primary hyperaldosteronism (True) Explanation: Conn's syndrome produces a hypokalaemic alkalosis pancreatic carcinoma (True) Explanation: Islet cell destruction thyrotoxicosis (True) Explanation: Also occurs in phaeochromocytoma and acromegaly Question 4. The physiological effects of insulin include increased glycolysis (True) decreased glycogenolysis (True) increased lipolysis (False) Explanation: Decreased lipolysis and ketogenesis increased gluconeogenesis (False) Explanation: Decreased gluconeogenesis increased protein catabolism (False) Explanation: Decreased protein catabolism Question 5. In decompensated diabetes mellitus thirst results from the increased osmolality of glomerular filtrate (True) Explanation: And produces an increase in plasma osmolality hyperpnoea is the result of acidosis due to increased lactic and ketoacid production (True) Explanation: Resulting in a metabolic acidosis negative nitrogen balance results from the increased protein catabolism (True) Explanation: Insulin deficiency increases protein degradation lipolysis increases as a result of relative insulin deficiency (True) Explanation: More profound ketogenesis occurs in type 1 diabetes mellitus insulin deficiency inhibits the peripheral utilisation of ketoacids (False) Explanation: Insulin deficiency increases ketoacid production Question 6. In the diagnosis of diabetes mellitus glycated haemoglobin (HbAlc) is a sensitive screening test (False) Explanation: Too insensitive to detect all cases absence of glycosuria excludes diabetes (False) Explanation: Renal threshold may be high glycosuria is usually due to a reduced renal threshold in young patients (True) Explanation: But it should never be assumed to be so 2% of patients have significant diabetic complications at presentation (False) Explanation: 20% have significant diabetic complications random plasma glucose concentrations > 11 mmol/l are diagnostic (True) Explanation: When symptoms suggest diabetes Question 7. Typical presentations of diabetes mellitus include weight loss (True) Explanation: Catabolism balanitis or pruritus vulvae (True) Explanation: Predisposition to monilial infection nocturia (True) Explanation: Osmotic diuresis limb pains with absent ankle reflexes (True) Explanation: Small vessel disease and neuropathy asymptomatic glycosuria in the elderly (True) Explanation: Often detected on routine urine testing Question 8. In the dietary management of diabetes mellitus 90% of patients also require hypoglycaemic drug therapy (False) Explanation: 50% of new diabetics can be controlled on diet alone

By A. H.

MCQs VIA WEB 2005 carbohydrate should provide 50% of the total calorie intake (True) Explanation: Higher than that in the average UK diet 10 g carbohydrate exchanges provide an ideal method of monitoring intake (False) Explanation: Not advocated as the method takes no account of glycaemic effects or fat intake fat intake should not exceed 35% of total calorie intake (True) Explanation: UK national diet tends to higher proportion of fat in obese patients, calorie intake should not exceed 750 kcal/day (False) Explanation: Calorie restriction of 500 kcal/day is more realistic and sustainable Question 9. Sulphonylurea drug therapy in diabetes mellitus causes less weight gain than biguanide therapy (False) Explanation: Causes more weight gain increases hepatic gluconeogenesis (False) Explanation: Stimulates pancreatic insulin secretion decreases the number of peripheral insulin receptors (False) Explanation: Such an action would produce insulin resistance decreases hepatic glycogenolysis (True) Explanation: Also decreases hepatic gluconeogenesis to reduce hyperglycaemia causes alcohol-induced flushing (True) Explanation: Disulfiram-like reaction Question 10. Biguanide drug therapy in diabetes mellitus is more likely to cause weight loss than weight gain (True) Explanation: Sometimes a useful adjunct to calorie-restricted diets Increases plasma immunoreactive insulin concentration (False) Explanation: Hence does not cause hypoglycaemia in non-diabetics Decreases pancreatic glucagon release (False) Explanation: Increases the sensitivity of peripheral insulin receptors Inhibits hepatic glycogenolysis (True) Explanation: Thus limiting hyperglycaemia Causes troublesome constipation (False) Explanation: Causes diarrhoea which may limit drug compliance Question 11. The following statements about other drug therapies in diabetes mellitus are true Thiazolidinediones enhance endogenous insulin sensitivity (True) Explanation: Activate peroxisome proliferator-activated receptor ã (PPARã agonists) Thiazolidinediones produce hyperinsulinaemia and hypoglycaemia (False) Explanation: Insulin sensitivity in adipose tissue is only increased in patients with insulin resistance Thiazolidinediones are best prescribed in combination with biguanides, sulphonylureas or insulin (True) Explanation: Glitazones promote weight gain and fluid retention similar to sulphonylureas and insulin Meglitinides increase peripheral insulin sensitivity (False) Explanation: Stimulate postprandial insulin secretion Alpha-glucosidase inhibitors induce carbohydrate malabsorption (True) Explanation: Selectively inhibit intestinal disaccharidases producing flatulence and diarrhoea Question 12. The following statements about insulin therapy are true Short-acting, regular insulin should be injected at least 30 minutes pre-prandially (True) Explanation: Onset of effect at least 30 minutes after injection the duration of action of short-acting regular insulin is 4-8 hours (True) intermediate-acting isophane insulin action peaks at 1-3 hours (False) Explanation: Isophane insulin action has an onset at 1-3 hours and lasts 7-14 hours The standard UK solution strength is 100 units/ml (True) Explanation: Different in other countries Once absorbed, insulin has a plasma half-life of 2 hours (False) Explanation: Plasma insulin has a half-life of 7 minutes Question 13. Typical symptoms of hypoglycaemia in diabetic patients include

By A. H.

MCQs VIA WEB 2005 Feelings of faintness and hunger (True) Explanation: But 50% of long-term type 1 diabetes mellitus patients have no symptoms Tremor, palpitation and dizziness (True) Explanation: Sympathetic nervous system activation Headache, diplopia and confusion (True) Explanation: Neuroglycopenia Abnormal behaviour despite a normal plasma glucose (False) Explanation: But plasma glucose concentration does not mirror cerebrospinal fluid glucose perfectly Nocturnal sweating, nightmares and convulsions (True) Explanation: Nocturnal hypoglycaemia may be difficult to recognise Question 14. In the treatment of severe hypoglycaemia in a diabetic patient patients should be taught to self-administer 50 ml of 50% glucose intravenously (False) Explanation: Defined as hypoglycaemia requiring the assistance of another person for recovery glucagon should be given intramuscularly (True) Explanation: Diabetics and close family members should be taught the technique the patient is more likely to have been taking metformin therapy alone rather than a sulphonylurea (False) Explanation: Hypoglycaemia does not occur with biguanides reversal of cognitive impairment is complete within 30 minutes of the restoration of normoglycaemia (False) Explanation: Can take 60-90 minutes after normoglycaemia is restored cerebral oedema should be considered if consciousness is not rapidly restored (True) Explanation: Other possibilities include stroke, hypoglycaemia-induced seizures and alcohol intoxication Question 15. Factors predisposing to frequent hypoglycaemic episodes in a diabetic patient include Delayed meals (True) Explanation: Or inadequate size of meal unusual exercise (True) Explanation: Often unanticipated Excessive alcohol intake (True) Explanation: A problem with patients on sulphonylurea drugs Development of hypoadrenalism (True) Explanation: Increased sensitivity to insulin; weight loss and nocturia should signal the possibility Errors in drug administration (True) Explanation: Inadvertent and occasionally even deliberate Question 16. The typical clinical features of diabetic ketoacidosis include Abdominal pain and air hunger (True) Explanation: Due to the acidosis Rapid, weak pulse and hypotension (True) Explanation: Due to dehydration and acidosis Profuse sweating with skin pallor (False) Explanation: Suggests hypoglycaemia-skin is typically dry with loss of turgor in diabetic ketoacidosis Vomiting and constipation (True) Explanation: Due to ketosis and dehydration coma with focal neurological signs (False) Explanation: Suggests severe hypoglycaemia Question 17. Expected findings in moderately severe diabetic ketoacidosis include water deficit of at least 6 litres (True) Explanation: Average deficit = 6 l (50% intracellular + 50% extracellular) sodium and potassium deficits of at least 400 mmol (True) Explanation: Chloride deficit similar plasma bicarbonate less than 12 mmol/l (True) Explanation: Check the arterial blood pH and PaCO2 absence of ketones in the urine (False) Explanation: Absence of ketonuria suggests another cause for the metabolic acidosis peripheral blood leucocytosis (True)

By A. H.

MCQs VIA WEB 2005 Explanation: Even in absence of infection due to acidosis

Question 18. In the management of diabetic ketoacidosis intracellular water deficit is best restored using half-strength saline (0.45% saline) (False) Explanation: Use isotonic solutions; change to 5% dextrose when blood glucose falls below 15 mmol/l potassium should be given immediately anticipating a low serum potassium concentration (False) Explanation: Avoid until the serum K+ result is available and do not give if the serum K+ > 5.5 mmol/l bicarbonate infusion is rarely necessary in the absence of renal failure (True) Explanation: Or in severe acidosis, i.e. pH < 7.0 (H+ concentration > 100 nmol/l) 5% dextrose solution should be avoided unless hypoglycaemia supervenes (False) Explanation: Dextrose is used to correct intracellular fluid depletion and if blood glucose < 15 mmol/l peripheral circulatory failure requires rapid volume replacement initially (True) Explanation: Give plasma expander if blood pressure does not improve rapidly; monitor urine output and central venous pressure Question 19. The clinical features of diabetic retinopathy include arteriolar spasm with arteriovenous nipping (False) Explanation: Suggests hypertensive change venous dilatation and increased venous tortuosity (True) Explanation: Sausage-like venous 'beading' soft and hard exudates (True) Explanation: Soft exudates indicate retinal ischaemia; hard exudates indicate plasma leakage retinal haemorrhages (True) Explanation: Appearance of haemorrhages corresponds with their site in the layers of the retina microaneurysms (True) Explanation: Major risk factor for macrovascular disease Question 20. The following statements about the long-term complications of diabetes are correct cardiovascular disease accounts for 70% of all deaths associated with diabetes (True) Explanation: Renal failure accounts for 10% of deaths associated with diabetes the excess mortality associated with diabetes is mainly attributable to microvascular complications (False) Explanation: Mainly macrovascular complications due to atherosclerosis the frequency and severity of microvascular complications can be minimised by strict glycaemic control (True) Explanation: Both in type 1 and type 2 diabetes cardiovascular complications can be minimised by strict control of the blood pressure (True) Explanation: Both in type 1 and type 2 diabetes diabetic patients with hypercholesterolaemia and cardiovascular disease benefit from statin therapy (True) Explanation: Both type 1 and type 2 diabetics at high risk from cardiovascular disease also benefit from angiotensinconverting enzyme inhibitor therapy Module 16 (Chapter 16) Question 1. The hypothalamic releasing factors listed below stimulate the pituitary gland to secrete the following hormones dopamine-prolactin (False) Explanation: Dopamine inhibits prolactin release; dopamine antagonists and TRH both stimulate prolactin release somatostatin-growth hormone (False) Explanation: Somatostatin inhibits growth hormone release thyrotrophin-releasing hormone (TRH)-thyroid-stimulating hormone (TSH) and prolactin (True) Explanation: In vivo significance of effect on prolactin is uncertain gonadotrophin-releasing hormone (GnRH)-luteinising hormone (LH) and follicle-stimulating hormone (FSH) independently (True) Explanation: Gonadal steroids and inhibin modify GnRH effects corticotrophin-releasing hormone (CRH)-â-lipotrophic hormone (LPH) and adrenocorticotrophic hormone (ACTH) (True) Explanation: Arginine vasopressin also effects ACTH release

By A. H.

MCQs VIA WEB 2005 Question 2. Causes of hyperprolactinaemia include oral contraceptive therapy (True) Explanation: And pregnancy chlorpromazine therapy (True) Explanation: Dopamine antagonist like metoclopramide primary hypothyroidism (True) Explanation: High plasma TRH hypothalamic disease (True) Cushing's disease (True) Explanation: High plasma ACTH Question 3. The clinical features of hyperprolactinaemia include hypogonadism and galactorrhoea (True) Explanation: Unilateral galactorrhoea suggests a breast tumour infertility associated with secondary amenorrhoea (True) Explanation: Typical an increased likelihood of macroadenoma in males (True) bitemporal hemianopia associated with microadenomas (True) Explanation: Pressure effects are only associated with macroadenomas prompt response to dopamine agonist therapy (True) Explanation: E.g. cabergoline and quinagolide Question 4. The clinical features of acromegaly include arthropathy and myopathy (True) Explanation: Also carpal tunnel syndrome hypertension and impaired glucose tolerance (True) Explanation: Both occur in 25% goitre and cardiomegaly (True) Explanation: Visceromegaly and hepatomegaly increased sweating and headache (True) Explanation: The commonest of all the symptoms skin atrophy and decreased sebum secretion (False) Explanation: The skin is thickened with increased sebum production Question 5. Typical results of investigations in a patient with acromegaly include failure of the plasma growth hormone (GH) to rise during a glucose tolerance test (GTT) (False) Explanation: Failure to suppress plasma GH-may even rise; GH normally falls during the GTT decreased serum prolactin (False) Explanation: Increased serum prolactin occurs in 30% increased serum insulin-like growth factor (IGF-1) (True) abnormality of the pituitary fossa on plain radiograph (False) Explanation: Rarely abnormal-MR scanning is used for preoperative assessment tumour shrinkage in response to octreotide therapy (False) Explanation: Somatostatin analogues reduce GH secretion but not tumour size Question 6. Typical features of anterior pituitary hormone deficiency in adults include loss of growth hormone function before luteinising hormone (True) Explanation: Then loss of ACTH and finally loss of TSH hypertension due to ACTH deficiency (False) Explanation: Hypotension due to the effects of cortisol deficiency on the vascular bed and kidneys skin pigmentation (False) Explanation: Striking pallor due to the effects of â-LPH deficiency on melanocytes myxoedema due to TSH deficiency (False) Explanation: Unlike primary hypothyroidism, skin changes do not occur dilutional hyponatraemia (True) Explanation: Due to increased ADH release and ADH sensitivity induced by hypotension and cortisol deficiency-ADH deficiency occurs if there is posterior pituitary damage

By A. H.

MCQs VIA WEB 2005 Question 7. Causes of hypopituitarism include Kallmann's syndrome (True) Explanation: GnRH deficiency associated with hypogonadotrophic hypogonadism and anosmia craniopharyngioma (True) Explanation: Any tumour close to the pituitary fossa including meningiomas head injury (True) Explanation: Including radiotherapy Sheehan's syndrome (True) Explanation: Post-partum necrosis of the pituitary gland sarcoidosis (True) Explanation: Also tuberculosis causing chronic basal meningitis Question 8. Causes of diabetes insipidus (DI) include congenital sex-linked recessive disorder (True) Explanation: Nephrogenic DI; also congenital cranial DI-autosomal dominant craniopharyngioma (True) Explanation: Any tumour or radiotherapy close to the pituitary fossa DIDMOAD syndrome (True) Explanation: DI, diabetes mellitus, optic atrophy and deafness severe hypocalcaemia (False) Explanation: Severe hypokalaemia and hypercalcaemia sarcoidosis (True) Explanation: Also tuberculosis causing chronic basal meningitis Question 9. The typical features of cranial diabetes insipidus include serum sodium concentration > 150 mmol/l with urine specific gravity < 1.001 (False) Explanation: Severe hypernatraemia only when water access denied increased polyuria following corticosteroid therapy for hypopituitarism (True) Explanation: Glucocorticoid insufficiency may mask diabetes insipidus onset following basal meningitis or hypothalamic trauma (True) Explanation: Or secondary to pituitary tumours or sarcoid decreased renal responsiveness to ADH following carbamazepine therapy (False) Explanation: Carbamazepine stimulates ADH release unlike psychogenic polydipsia, the response to ADH is invariably normal (True) Explanation: An effect of long-term overhydration in psychogenic polydipsia Question 10. The insulin tolerance test is mandatory to confirm the diagnosis of hypopituitarism (False) Explanation: An ACTH stimulation test is often the more appropriate test best terminated as soon as the plasma glucose falls below 2.4 mmol/l (True) Explanation: Or if severe hypoglycaemic symptoms develop contraindicated in ischaemic heart disease (True) Explanation: Needs an adequate hypoglycaemic stimulus and runs the risk of hypoglycaemia contraindicated in severe hypopituitarism (True) Explanation: Plasma cortisol at 0800 hrs < 180 nmol/l an unreliable test of hypothalamic function (False) Explanation: Test of hypothalamic-pituitary-adrenal axis Question 11. The following statements about thyroid hormones are true T3 and T4 are both stored in colloid vesicles as thyroglobulin (True) Explanation: Thyroglobulin is synthesised within thyroid cells T4 is metabolically more active than T3 (False) Explanation: T4 should be regarded as a prohormone T3 and T4 are mainly bound to albumin in the serum (False) Explanation: Bound to thyroxine-binding globulin and also to pre-albumin 85% of the circulating T3 arises from extra-thyroidal T4 (True)

By A. H.

MCQs VIA WEB 2005 Explanation: T4 is deiodinated in liver, muscle and kidney conversion of T4 to T3 decreases in acute illness (True) Explanation: Production of reverse T3 may increase Question 12. The finding of reduced serum free T4 and thyroid-stimulating hormone (TSH) concentrations is compatible with the following conditions hypopituitarism (True) Explanation: With secondary hypothyroidism primary hypothyroidism (False) Explanation: Serum TSH would be elevated nephrotic syndrome (False) Explanation: Free T4 is normal but total T4 is often increased (high thyroxine-binding globulin (TBG) concentrations) pneumonia (True) Explanation: Sick euthyroid syndrome-total and free T4 may be reduced pregnancy (False) Explanation: Free T4 and TSH are normal; total T4 is often increased (high TBG concentrations) Question 13. The following statements about thyrotoxicosis are true most patients have Graves' disease (True) Explanation: 75% of cases multinodular goitre is more common than uninodular goitre (True) Explanation: 15% multinodular, 5% single nodule amiodarone treatment should be considered as a possible cause (True) Explanation: May also cause hypothyroidism the thyroid gland is diffusely hyperactive in Graves' disease (True) Explanation: A goitre is therefore usually present there is an increased prevalence of HLA-DR3 in Graves' disease (True) Explanation: And HLA-B8 and DR2 Question 14. The clinical features of thyrotoxicosis include atrial fibrillation with a collapsing pulse (True) Explanation: Or persisting resting sinus tachycardia weight loss and oligomenorrhoea (True) Explanation: Appetite is maintained peripheral neuropathy (False) Explanation: Muscular weakness may occur proximal myopathy and exophthalmos (True) Explanation: Occasionally with ophthalmoplegia decreased insulin requirements in type 1 diabetes mellitus (False) Explanation: Insulin requirements may increase Question 15. In the treatment of thyrotoxicosis propranolol should not be given in atrial fibrillation (False) Explanation: Controls ventricular response rate carbimazole blocks the secretion of T3 and T4 by the thyroid (False) Explanation: Inhibits the iodination of tyrosine persistent suppression of the serum TSH is an indication for surgery (False) Explanation: TSH measurement alone should not guide therapy serum TSH receptor antibodies usually persist despite carbimazole (False) Explanation: But titres correlate poorly with disease activity surgery is more likely to be necessary in young men than in women (True) Explanation: Especially patients with large goiters Question 16. The clinical features of primary hypothyroidism include carpal tunnel syndrome and proximal myopathy (True) Explanation: Both, however, are non-specific cold sensitivity and menorrhagia (True)

By A. H.

MCQs VIA WEB 2005 Explanation: And infertility and impotence deafness and dizziness (True) Explanation: Perhaps due to oedema of the middle ear puffy eyelids and malar flush (True) Explanation: And rarely alopecia, vitiligo and dry hair absent ankle tendon reflexes (False) Explanation: Reflexes preserved with delayed relaxation Question 17. Biochemical findings in primary hypothyroidism include decreased serum free T4 and decreased serum TSH concentration (False) Explanation: Decreased serum free T4 and increased serum TSH concentration increased serum prolactin concentration (True) Explanation: Rarely causing galactorrhoea inappropriate ADH secretion (True) Explanation: Producing hyponatraemia increased serum alkaline phosphatase concentration (False) Explanation: Serum lactate dehydrogenase and creatine kinase may be elevated increased serum cholesterol concentration (True) Explanation: And serum triglyceride levels Question 18. The development of a goitre is associated with Coxsackie B viral infection (False) Explanation: May cause painful thyroiditis with transient hypothyroidism dietary iodine deficiency (True) Explanation: Hypothyroidism if iodine deficiency is severe excess dietary calcium intake (False) Explanation: No association cranial irradiation (True) Explanation: Secondary hypothyroidism lithium carbonate therapy (True) Explanation: Inhibits release of thyroid hormones Question 19. Thyroid carcinoma of lymphomatous type usually presents as a single 'hot' thyroid nodule (False) Explanation: 'Hot' nodules are almost always benign anaplastic type is usually cured by local radiotherapy (False) Explanation: Radiotherapy provides brief symptomatic relief only follicular type is best treated by 131I radio-iodine therapy alone (False) Explanation: Total thyroidectomy, radio-iodine and long-term thyroxine papillary type should be treated with total thyroidectomy (True) Explanation: Papillary tumours are the most common cell type medullary type secretes calcitonin causing severe hypocalcaemia (False) Explanation: Rare despite high calcitonin levels; carcinoid syndrome can occur Question 20. The serum calcium concentration is typically increased in hypoalbuminaemia (False) Explanation: 40% of calcium is protein-bound; normal after correction for serum albumin pyloric stenosis (False) Explanation: But metabolic alkalosis decreases the level of ionised calcium carcinomatosis (True) Explanation: Due to bone metastases (often microscopic) hypoparathyroidism (False) Explanation: Decreases serum calcium levels chronic sarcoidosis (True) Explanation: Increased vitamin D synthesis with decreased PTH concentration Question 21. Typical clinical features of primary hyperparathyroidism include

By A. H.

MCQs VIA WEB 2005 recurrent acute pancreatitis and renal colic due to calculi (True) Explanation: But 50% of patients are asymptomatic hyperplasia of all the parathyroid glands on histology (False) Explanation: Solitary parathyroid adenoma in 90% osteitis fibrosa on bone radiographs at presentation (False) Explanation: A relatively late feature the complications of pseudogout and hypertension (True) Explanation: And peptic ulceration and myopathy nephrogenic diabetes insipidus (True) Explanation: With characteristic polyuria Question 22. Typical biochemical findings in primary hyperparathyroidism include increased serum calcium and phosphate concentrations (False) Explanation: Phosphate is usually low decreased serum 1,25-dihydroxycholecalciferol concentration (False) Explanation: Increased 1,25-dihydroxycholecalciferol concentration hypercalciuria and hyperphosphaturia (True) Explanation: Predisposing to stone formation increased serum alkaline phosphatase with bony involvement (True) Explanation: Indicating osteoblastic activity increased serum calcium and PTH concentrations (True) Explanation: Serum chloride concentration is usually elevated Question 23. Causes of hypercalcaemia include bone metastases (True) Explanation: Often via production of osteoclast-activating factors carcinomas secreting PTH-like peptides (True) Explanation: Undetectable using standard PTH assays severe Addison's disease (True) Explanation: Increased vitamin D synthesis with low PTH concentration severe hypothyroidism (False) Explanation: Hyperthyroidism is a rare cause chronic sarcoidosis (True) Explanation: Increased vitamin D production with low PTH concentration Question 24. The following statements about adrenal gland physiology are true ACTH normally controls the adrenal secretion of aldosterone (False) Explanation: Principally under control of angiotensin II ACTH increases adrenal androgen and cortisol secretion (True) Explanation: In the zona reticularis and zona fasciculata respectively the plasma cortisol concentration normally peaks in the evening (False) Explanation: Cortisol levels fall to a nadir at around midnight hyperglycaemia increases the rate of cortisol secretion (False) Explanation: Hypoglycaemia stimulates cortisol release cortisol enhances gluconeogenesis and lipogenesis (True) Explanation: Anti-insulin effects Question 25. The typical clinical features of Cushing's syndrome include generalised osteoporosis (True) Explanation: Protein catabolism in bone systemic hypotension (False) Explanation: Hypertension may occur hirsutism and amenorrhoea (True) Explanation: Impotence in men proximal myopathy (True) Explanation: Muscle protein catabolism

By A. H.

MCQs VIA WEB 2005 hypoglycaemic episodes (False) Explanation: Impaired glucose tolerance Question 26. Adverse effects of oral corticosteroid therapy include peptic ulceration (True) Explanation: Decreases mucosal resistance hypertension (True) Explanation: Increased renal sodium reabsorption avascular bone necrosis (True) Explanation: Particularly likely to affect the femoral heads pseudogout (False) Explanation: Sometimes used to treat severe pseudogout insomnia (True) Explanation: Typical; causes day-night reversal of biorhythms Question 27. In primary hyperaldosteronism (Conn's syndrome) peripheral oedema is usually marked (False) Explanation: Unlike oedema in patients with secondary hyperaldosteronism proximal myopathy is due to hypokalaemia (True) Explanation: Rarely hypokalaemic paralysis polyuria and polydipsia are characteristic (True) Explanation: Hypertension and hypokalaemia are also characteristic diabetes mellitus is often present (False) Explanation: Type 2 diabetes mellitus is, however, associated with primary hypoadrenalism hypertension is associated with hyperreninaemia (False) Explanation: Associated with renin suppression Question 28. Causes of primary adrenocortical insufficiency include haemochromatosis (True) Explanation: Rare cause autoimmune adrenalitis (True) Explanation: Commonest cause amyloidosis (True) Explanation: Rare sarcoidosis (False) Explanation: May cause hypercalcaemia tuberculosis (True) Explanation: Now a rare cause Question 29. Typical features of primary adrenocortical insufficiency include anorexia, weight loss and diarrhoea (True) Explanation: All features of glucocorticoid insufficiency pigmentation of scars from surgery preceding hypoadrenalism (False) Explanation: Only new scars become pigmented vitiligo, weakness and hypotension (True) Explanation: Vitiligo is seen in 10-20% of patients increased insulin requirements in diabetic patients (False) Explanation: Increased insulin sensitivity with hypoglycaemia amenorrhoea and loss of body hair (True) Explanation: Loss of adrenal androgen Question 30. The typical features of phaeochromocytoma include predominantly adrenaline rather than noradrenaline secretion (False) Explanation: Noradrenaline is a precursor of adrenaline episodic nausea with sweating and marked skin pallor (True) Explanation: Catecholamine secretion underlying malignant tumour in the majority (False)

By A. H.

MCQs VIA WEB 2005 Explanation: 90% are benign presentation with hypertension and hypercalcaemia (True) Explanation: Occurs in MEN type II syndrome control of symptoms following propranolol therapy alone (False) Explanation: Symptoms worsen due to unopposed á-adrenoceptor activity Question 31. Causes of gynaecomastia include androgen deficiency and/or excessive oestrogen production (True) Explanation: E.g. hypogonadism or chronic liver failure microprolactinoma or macroprolactinoma (True) Explanation: Inhibition of LH/FSH secretion caused by prolactin cimetidine therapy (True) Explanation: Also spironolactone and anti-androgen therapy (e.g. cyproterone + GnRH analogues) haemochromatosis (True) Explanation: Hypergonadotrophic hypogonadism human chorionic gonadotrophin-secreting tumour (True) Explanation: Rare cause of excessive oestrogen production Question 32. In cryptorchidism with inguinal testes in a child the individual is usually otherwise normal (True) Explanation: Chromosomal abnormalities are rare hypogonadotrophic hypogonadism should be excluded (True) Explanation: Occurs in the minority the seminiferous tubules are typically normal (False) Explanation: Sterility follows if bilateral testicular interstitial cell function is usually normal (True) Explanation: Secondary sexual characteristics are preserved treatment with chorionic gonadotrophin or GnRH is contraindicated (False) Explanation: Testicular descent ensues in 40% Question 33. Causes of secondary amenorrhoea include pituitary microprolactinoma (True) Explanation: Suppression of LH and FSH secretion by prolactin anorexia nervosa (True) Explanation: Failure of gonadotrophin secretion Cushing's syndrome (True) Explanation: Associated with hyperprolactinaemia renal failure (True) Explanation: Or other severe systemic disease Stein-Leventhal syndrome (True) Explanation: Polycystic ovary disease Question 34. The typical features of the menopause include decreased plasma LH and FSH concentrations (False) Explanation: Gonadotrophins elevated hirsutism and clitoral hypertrophy (False) Explanation: Features of androgen excess bone fractures due to osteomalacia (False) Explanation: Osteoporosis develops prematurely superficial dyspareunia and dysuria (True) Explanation: Due to oestrogen deficiency normal age at onset 40 years (False) Explanation: Normal menopause occurs at age 50-55 years Question 35. Causes of hirsutism include idiopathic familial hirsutism (True) Explanation: Commonest cause and treated with anti-androgens (e.g. cyproterone)

By A. H.

MCQs VIA WEB 2005 polycystic ovarian syndrome (PCOS) (True) Explanation: PCOS is associated with obesity and infertility; plasma LH:FSH ratio > 2.5:1 Cushing's syndrome (True) Explanation: Modest increase in adrenal androgen synthesis autoimmune polyglandular syndrome (False) Explanation: Primary adrenal, thyroid, parathyroid, gastric parietal and gonadal failure syndromes ovarian tumour (True) Explanation: Ectopic androgen production does not suppress with dexamethasone (unlike excessive androgen production in congenital adrenal hyperplasia) Module 17 (Chapter 17) Question 1. Causes of mouth ulcers include gluten enteropathy (True) Explanation: And systemic lupus erythematosus, Beh[sfgr ]et's syndrome, Reiter's syndrome Crohn's disease (True) Explanation: And ulcerative colitis lichen planus (True) Explanation: And pemphigoid and pemphigus adverse drug reaction (True) Explanation: Stevens-Johnson syndrome due to either drugs or infections herpes simplex (True) Explanation: Aphthous mouth ulcers are usually idiopathic rather than viral-induced Question 2. Causes of salivary gland enlargement include alcoholic liver disease (True) Explanation: Also associated with malnutrition and autoimmune hepatitis Sjögren's syndrome (True) Explanation: Associated with dry mouth and keratoconjunctivitis sicca (dry eyes) bacterial infection (True) Explanation: May be associated with calculi in the parotid duct sarcoidosis (True) Explanation: Uveoparotid fever (Heerfordt's syndrome) measles (False) Explanation: Associated with mumps Question 3. Recognised causes of dysphagia include iron deficiency anaemia (True) Explanation: Via formation of an oesophageal web-'sideropenic dysphagia' pharyngeal pouch (True) Explanation: May also be associated with regurgitation and recurrent aspiration Barrett's oesophagus (False) Explanation: Asymptomatic unless complicated by malignancy myasthenia gravis (True) Explanation: More commonly caused by stroke; typically worse with fluids than with solids achalasia (True) Explanation: Best diagnosed on oesophageal manometry Question 4. Typical features of oesophageal achalasia include recurrent pneumonia (True) Explanation: Due to regurgitation and aspiration spasm of the lower oesophageal sphincter (LOS) (False) Explanation: Failure to relax the LOS with loss of ganglion cells in Auerbach's plexus on histology heartburn and acid reflux (False) Explanation: Acid reflux is prevented by the non-relaxing LOS predisposition to oesophageal carcinoma (True) Explanation: Even if the obstruction is treated symptomatic response to pneumatic balloon dilatation (True)

By A. H.

MCQs VIA WEB 2005 Explanation: If this fails, Heller's myotomy may be indicated

Question 5. Gastro-oesophageal reflux disease is associated with the following factors decreased intra-abdominal pressure (False) Explanation: Associated with increased intra-abdominal pressure (e.g. pregnancy) delayed gastric emptying (True) prolonged oesophageal transit time (True) Explanation: Delayed oesophageal clearance is more common in the elderly increased lower oesophageal sphincter tone (False) Explanation: Associated with decreased lower oesophageal sphincter tone presence of a hiatus hernia (True) Question 6. Oesophageal carcinoma in the UK is associated with gluten enteropathy (True) Explanation: Squamous rather than adenocarcinoma more likely to be due to adenocarcinoma than squamous carcinoma (False) Explanation: 80-90% are squamous cell associated with Barrett's oesophagus (True) Explanation: Adenocarcinoma is associated with chronic oesophagitis more likely to arise in the upper third rather than the lower third of the oesophagus (False) Explanation: 90% are in the lower two-thirds associated with alcohol and tobacco consumption (True) Explanation: And betel nut chewing in the East Question 7. Typical features of oesophageal carcinoma at presentation include acid reflux and odynophagia (False) Explanation: More suggestive of reflux with oesophagitis and stricture formation painless obstruction to the passage of a food bolus (True) Explanation: Painless due to destruction of the mucosal innervation nausea and weight loss (True) Explanation: Weight loss relates to poor food intake metastatic spread in the majority of patients (True) Explanation: 75% have lymph node, liver and/or mediastinal spread overall survival rates at 5 years of approximately 50% (False) Explanation: 5-year survival is about 5% Question 8. Factors associated with chronic peptic ulcer disease include oral contraceptive therapy (False) non-steroidal anti-inflammatory drug therapy (True) Explanation: Plays a role in gastric ulcer pernicious anaemia (False) Explanation: Associated with achlorhydria-'no acid, no ulcer' Helicobacter pylori - associated gastritis (True) Explanation: Implicated in > 90% of instances tobacco consumption (True) Explanation: Associated with both gastric and duodenal ulcer recurrence rates Question 9. Typical features of peptic ulcer dyspepsia include pain relieved by eating (True) Explanation: Hunger pain well-localised pain relieved by vomiting (True) Explanation: Perhaps with the 'pointing sign' pain-free remissions lasting many weeks (True) Explanation: Pain is characteristically periodic nausea and epigastric pain (False) Explanation: More suggestive of biliary colic; pain rarely lasts > 2 hours

By A. H.

MCQs VIA WEB 2005 absence of symptoms prior to acute perforation (True

Question 10. In the investigation and treatment of chronic dyspepsia most patients aged < 55 years have an underlying peptic ulcer (False) Explanation: Only about 20%; most have reflux dyspepsia or functional dyspepsia 25% of duodenal ulcers relapse unless H. pylori has been eradicated (False) Explanation: 85% relapse if H. pylori has not been eradicated magnesium-containing antacids produce constipation (False) Explanation: Cause diarrhoea; aluminium-containing antacids cause constipation bismuth compounds should not be used for maintenance therapy (True) Explanation: Due to potential accumulation of bismuth, acid-lowering drugs are preferable gastric ulcers associated with NSAID therapy are less likely to be associated with H. pylori gastritis than gastric ulcers occurring in patients not taking NSAIDs (True) Explanation: 30% of gastric ulcers are not associated with H. pylori (NSAID-induced ulcers) Question 11. Gastroduodenal haemorrhage in the UK is more often due to peptic ulcer than to oesophageal varices (True) Explanation: Peptic ulcer 35-50%, varices < 5% associated with a 5% mortality when due to chronic peptic ulceration (True) Explanation: Higher mortality in the elderly and especially in patients who rebleed a recognised complication of severe head injury (True) Explanation: Cushing's stress ulcers best investigated by endoscopy (True) Explanation: Diagnostic yield reduces with time post-admission significantly associated with anti-inflammatory drug therapy (True) Explanation: 75% of patients with gastrointestinal bleed have recently taken NSAIDs (only 50% of 'controls') Question 12. Typical features of major acute gastroduodenal haemorrhage include severe abdominal pain (False) Explanation: Typically pain-free angor animi and restlessness (True) Explanation: Sympathetic activation syncope preceding other evidence of bleeding (True) Explanation: Particularly in older patients elevated blood urea and creatinine concentrations (False) Explanation: Blood urea but not creatinine rises due to digestion of the blood in the gut peripheral blood microcytosis (False) Explanation: Only present if preceding iron deficiency Question 13. When acute gastroduodenal haemorrhage is suspected a pulse rate > 100/min is most likely to be due to anxiety (False) Explanation: A sign of hypovolaemia hypotension without a tachycardia suggests an alternative diagnosis (False) Explanation: Bradycardia may occur in profound blood loss or in the elderly the absence of anaemia suggests the volume of blood loss is modest (False) Explanation: Haemoglobin concentration remains unaltered until haemodilution occurs nasogastric aspiration provides an accurate estimate of blood loss (False) Explanation: Monitoring the urine output as a measure of perfusion is important endoscopy is best deferred pending blood volume replacement (True) Explanation: Patients should first be haemodynamically stable if possible Question 14. In resuscitating a patient with an acute gastrointestinal bleed oxygen should be administered if there are signs of hypovolaemia (True) Explanation: Especially in patients with shock transfusion requires whole blood rather than packed red cells (False) Explanation: Colloid infusion and packed red cells are adequate for volume replacement

By A. H.

MCQs VIA WEB 2005 volume replacement with colloids is preferable to crystalloids (True) Explanation: Crystalloids rapidly redistribute to the extravascular space monitoring central venous pressure and/or urine output is advisable (True) Explanation: Facilitates restoration of optimal circulating volume surgical intervention should be considered if rebleeding occurs despite ulcer sclerotherapy (True) Explanation: Consider surgical options in all patients with continuing bleeding Question 15. Perforation of a peptic ulcer is typically associated with acute rather than chronic ulcers (False) Explanation: 25% occur in acute ulcers duodenal more often than gastric ulcers (True) Explanation: Especially anterior wall ulcers abdominal pain radiating to the shoulder tip (True) Explanation: Diaphragmatic pain referred to one or both shoulder tips the absence of nausea and vomiting (False) Explanation: Vomiting is common symptomatic improvement several hours following onset (True) Explanation: But abdominal rigidity typically persists Question 16. Characteristic features of gastric outlet obstruction include metabolic acidosis (False) Explanation: Hypokalaemic metabolic alkalosis bile vomiting (False) Explanation: Suggests more distal obstruction urinary pH < 5 (True) Explanation: Paradoxical aciduria due to renal tubular mechanisms symptomatic relief after vomiting (True) Explanation: Unusually, patients may feel like eating immediately after vomiting absent gastric peristalsis (False) Explanation: Often prominent gastric peristalsis and a succussion splash Question 17. Complications of partial gastrectomy include early satiety (True) Explanation: Smaller stomach and loss of vagally mediated gastric relaxation iron deficiency anaemia (True) Explanation: Malabsorption is common and can produce folate, B12 and vitamin D deficiency weight loss (True) Explanation: Most patients will lose at least 5 kg reactive hypoglycaemia (True) Explanation: Late dumping syndrome with exaggerated insulin release vomiting and diarrhoea soon after meals (True) Explanation: Early dumping syndrome with the exaggerated release of upper gastrointestinal hormones Question 18. The typical features of non-ulcer dyspepsia include onset under the age of 40 years (True) Explanation: Women are more commonly affected than men nausea and bloating (True) Explanation: Dysmotility state weight loss and anaemia (False) Explanation: Features suggesting serious underlying disease constipation with pellety stools (True) Explanation: Often associated with an irritable bowel syndrome symptoms of anxiety and depression (True) Explanation: Often associated with stressful life events and difficulties Question 19. Carcinoma of the stomach is associated with adenomatous gastric polyps (True)

By A. H.

MCQs VIA WEB 2005 chronic hypochlorhydria (True) Explanation: Pernicious anaemia and partial gastrectomy Helicobacter pylori infection (True) Explanation: H. pylori may account for 60% of gastric carcinoma Ménétrier's disease (True) Explanation: Hypertrophic gastritis with protein-losing enteropathy alcohol and tobacco consumption (True) Question 20. Typical features of gastric carcinoma in the UK include progression to involve the duodenum (False) Explanation: Extraordinary but true origin within a chronic peptic ulcer (False) Explanation: But may present as a malignant ulcer overall 5-year survival rate of 50% (False) Explanation: 10% 5-year survival folate deficiency anaemia on presentation (False) Explanation: Iron deficiency anaemia is typical supraclavicular lymphadenopathy (True) Explanation: Virchow's node Question 21. In gluten enteropathy (coeliac disease) the peak at onset is 11-19 years (False) Explanation: Peak incidence in the age groups 1-5 years and 40-59 years there is a predisposition to gut lymphoma and carcinoma (True) Explanation: Symptoms return without dietary indiscretion the toxic agent is the polypeptide á-gliadin (True) Explanation: A component of the gluten protein gluten-free diets improve absorption but not the villous atrophy (False) Explanation: Villous atrophy should resolve serum anti-endomysium IgA antibody titres are characteristically elevated (True) Explanation: Also antigliadin IgA antibody titres Question 22. Causes of small bowel bacterial overgrowth (blind loop syndrome) include diabetic autonomic neuropathy (True) Explanation: Reduced small intestinal motility chronic hypochlorhydria (True) Explanation: E.g. long-term proton pump inhibitor therapy and pernicious anaemia jejunal diverticulosis (True) Explanation: Best demonstrated by barium meal progressive systemic sclerosis (True) Explanation: Reduced small intestinal motility enterocolic fistula (True) Explanation: E.g. Crohn's disease Question 23. Clinical features suggesting the carcinoid syndrome include facial blanching and sweating (False) Explanation: Facial telangiectasia, flushing and wheezing constipation (False) Explanation: Diarrhoea is characteristic intestinal ischaemia (True) Explanation: Due to mesenteric infiltration and/or vasospasm tricuspid valve dysfunction (True) Explanation: And pulmonary stenosis late occurrence of metastatic disease (False) Explanation: Typically associated with widespread liver metastases Question 24. Causes of acute pancreatitis include

By A. H.

MCQs VIA WEB 2005 measles (False) Explanation: Mumps and Coxsackie B viral infections hypothermia (True) Explanation: And hyperlipidaemia choledocholithiasis (True) Explanation: 50% of cases are associated with biliary tract disease azathioprine therapy (True) Explanation: And thiazides and corticosteroids alcohol misuse (True) Explanation: Common cause in the UK Question 25. The following are characteristic of acute pancreatitis abdominal guarding develops soon after the onset of pain (False) Explanation: Guarding occurs relatively late normal serum amylase concentration in the first 4 hours after onset (False) Explanation: Serum amylase rises and falls rapidly persistent serum hyperamylasaemia suggests a developing pseudocyst (True) Explanation: Or pancreatic abscess or non-pancreatic cause hypercalcaemia 5-7 days after onset (False) Explanation: Hypocalcaemia hyperactive loud bowel sounds (False) Explanation: Bowel sounds usually absent or diminished due to paralytic ileus Question 26. Adverse prognostic factors in acute pancreatitis include arterial hypoxaemia with a PaO2 < 8 kPa (True) Explanation: Administer high-flow oxygen therapy leucopenia with white blood cell count < 5 × 109/l (False) Explanation: Poorer prognosis indicated by white blood cell count > 15 × 109/l serum albumin < 30 g/l and serum calcium < 2 mmol/l (True) Explanation: Reflect extent of peritoneal reaction hypoglycaemia < 2.3 mmol/l (False) Explanation: Hyperglycaemia > 10 mmol/l blood urea > 16 mmol/l after rehydration (True) Question 27. In the management of acute pancreatitis early laparotomy is advisable to exclude alternative diagnoses (False) Explanation: Diagnostic laparotomy is rarely required opiates should be avoided because of spasm of the sphincter of Oddi (False) Explanation: Effective pain relief is important intravenous fluids are unnecessary in the absence of a tachycardia (False) Explanation: Heart rate alone is a poor guide to volume losses the urine output and PaO2 should be monitored (True) Explanation: Shock and respiratory failure are serious complications persistent elevation in the serum amylase suggests pancreatic duct obstruction (True) Explanation: Resulting in pancreatic pseudocyst Question 28. In the investigation of chronic pancreatic disease glucose tolerance is typically normal in pancreatic carcinoma (False) Explanation: Typically impaired glucose tolerance test (GTT) duodenal ileus is a characteristic feature of chronic pancreatitis (False) Explanation: Occurs in acute pancreatitis transabdominal ultrasound scanning is more sensitive than CT (False) Explanation: Pancreatic visualisation is superior with CT endoscopic retrograde cholangiopancreatography (ERCP) can reliably distinguish carcinoma from chronic pancreatitis (False) Explanation: Surgery may be necessary pancreatic calcification suggests alcohol as the cause (True)

By A. H.

MCQs VIA WEB 2005 Explanation: Biliary tract disease is rarely the cause

Question 29. Features consistent with the diagnosis of chronic pancreatitis include abdominal or back pain persisting for days (True) Explanation: Sometimes relieved by crouching or leaning forward chronic opiate dependency (True) Explanation: In 20% increased sodium concentration in the sweat (False) Explanation: Occasionally in cystic fibrosis abdominal pain induced and relieved by alcohol intake (True) pancreatic calcification on plain radiograph or ultrasound (True) Explanation: But insensitive diagnostic tests Question 30. Typical causes of chronic pancreatitis include annular pancreas (False) Explanation: Associated with pancreas divisum alcoholism (True) Explanation: Accounts for 70-80% of instances gallstones (False) Explanation: Common but not the cause of chronic pancreatitis cystic fibrosis (True) mumps (False) Question 31. Typical complications of chronic pancreatitis include pancreatic pseudocyst formation (True) Explanation: Also associated with acute pancreatitis obstructive jaundice (True) Explanation: Due to stricture of the common bile duct as it passes the head of the pancreas portal vein thrombosis (True) Explanation: And splenic vein thrombosis leading to gastric varices diabetes mellitus (True) Explanation: Occurs in 30% overall opiate drug dependence (True) Explanation: May occur in up to 20% of patients Question 32. The typical features of pancreatic carcinoma include adenocarcinomatous histology (True) Explanation: The vast majority origin in the body of the pancreas in 60% of patients (False) Explanation: Head of pancreas is the origin in 60% of patients presentation with diabetes mellitus (True) Explanation: Indicating advanced disease back pain and weight loss indicate a poor prognosis (True) Explanation: Even in the absence of metastatic spread presentation with painless jaundice (True) Explanation: Usually due to a tumour in the head of pancreas Question 33. Characteristic features of ulcerative colitis include invariable involvement of the rectal mucosa (True) Explanation: Proctitis is a typical finding segmental involvement of the colon and rectum (False) Explanation: Suggests Crohn's disease pseudopolyposis following healing of mucosal damage (True) Explanation: Due to oedema and hyperplasia inflammation extending from the mucosa to the serosa (False) Explanation: Affects mucosa and submucosa only

By A. H.

MCQs VIA WEB 2005 enterocutaneous and enteroenteric fistulae (False) Explanation: Suggest Crohn's disease Question 34. Ulcerative colitis (UC) differs from Crohn's colitis in that UC occurs at any age (False) Explanation: Both have a peak incidence at the age of about 20 years cessation of smoking is likely to reduce activity of Crohn's disease (True) Explanation: Smoking exacerbates Crohn's disease but not ulcerative colitis toxic dilatation only occurs in ulcerative colitis (False) Explanation: Also occurs in severe Crohn's colitis there is no association with aphthous mouth ulcers in UC (unlike Crohn's disease) (False) Explanation: Occur in both there is no involvement of the small bowel in UC (True) Question 35. Recognised complications of ulcerative colitis include pyoderma gangrenosum (True) Explanation: Also occurs in Crohn's disease and rheumatoid arthritis pericholangitis (True) Explanation: Suggested by abnormal liver function tests amyloidosis (True) Explanation: Induced by many chronic inflammatory diseases colonic carcinoma (True) Explanation: Long-standing disease (> 10 years) enteropathic arthritis (True) Question 36. Characteristic features of Crohn's disease include familial association with ulcerative colitis (True) Explanation: And vice versa onset after the age of 70 years (False) Explanation: Early adult life most commonly disease confined to the terminal ileum and colon (False) Explanation: Affects any part of the alimentary tract predisposition to biliary and renal calculi (True) Explanation: Bile acid malabsorption and hyperoxaluria giant cell granulomata on histopathology (True) Explanation: Crohn's granulomata are non-caseating unlike those of tuberculosis Explanation: Large joints especially, or spondyloarthritis Question 37. The typical clinical features of ileal Crohn's disease include association with tobacco consumption (True) Explanation: In contrast to ulcerative colitis presentation with bloody diarrhoea (False) Explanation: Usually pain rather than diarrhoea unless there is rectal involvement also presentation with subacute intestinal obstruction (True) Explanation: With episodes of colicky pain segmental involvement of the colon and rectum (True) Explanation: In contrast to ulcerative colitis inflammatory changes confined to the mucosa on histopathology (False) Explanation: Inflammation is transmural Question 38. The typical features of the irritable bowel syndrome include nocturnal diarrhoea and weight loss (False) Explanation: Such symptoms suggest organic pathology onset after the age of 45 years (False) Explanation: Typically affects females aged 16-45 years history of childhood abdominal pain (True) Explanation: Many also have dyspeptic and urinary symptoms

By A. H.

MCQs VIA WEB 2005 right iliac fossa pain and urinary frequency (True) Explanation: Pain may be relieved by defaecation abdominal distension, flatulence and pellety stools (True) Explanation: May be tenesmus, mucus PR and diarrhoea Question 39. The management of the irritable bowel syndrome should include explanation and reassurance after a detailed clinical examination (True) Explanation: Probably the most important therapeutic tools barium enema and barium follow-through examinations in all patients (False) Explanation: Investigations are important in older patients evaluation of social and emotional factors (True) Explanation: Anxiety and/or depression are often associated with refractory symptoms referral for psychiatric assessment and therapy (False) Explanation: Although occasionally psychiatric intervention may be necessary dihydrocodeine for abdominal pain and diarrhoea (False) Explanation: Use loperamide, a safer opioid that does not cross the blood-brain barrier Question 40. Typical features of colonic diverticulosis include predominant involvement of the right hemicolon (False) Explanation: Sigmoid colon is most commonly involved predisposition to the development of colonic carcinoma (False) Explanation: No causative association complications are more common in patients receiving NSAID therapy (True) Explanation: Especially bleeding and perforation reduction in the number of diverticula with a high-fibre diet (False) Explanation: But symptoms may be improved the absence of symptoms in the absence of complications (True) Explanation: Such as acute diverticulitis Question 41. Typical features of colonic diverticulitis include severe rectal bleeding (True) Explanation: Exclusion of malignancy may be necessary chronic iron deficiency anaemia (False) Explanation: But this may be a feature of chronic diverticulosis septicaemia and paralytic ileus (True) Explanation: With or without perforation right iliac fossa pain (False) Explanation: Left iliac fossa or hypogastric pain is typical vesicocolic fistula (True) Explanation: Or enterocolic or colovaginal Question 42. Typical features of pseudomembranous colitis include onset within two weeks of antibiotic therapy (True) Explanation: Occurs from 4 days to 6 weeks post-antibiotics normal appearance of the rectal mucosa (False) Explanation: Usually appears as a non-specific proctitis Clostridium difficile toxin in the stool (True) presentation with abdominal pain and diarrhoea (True) Explanation: And even bloody diarrhoea clinical relapse despite prompt treatment (True) Explanation: Treated with metronidazole or vancomycin Question 43. Familial adenomatous polyposis is inherited as an autosomal recessive trait (False) Explanation: Autosomal dominant with a prevalence of 1 in 14 000 usually clinically apparent before the age of 10 years (False) Explanation: Typically presents in the age group 20-40 years

By A. H.

MCQs VIA WEB 2005 likely to progress to carcinoma before the age of 40 years (True) Explanation: Carcinoma is usually present when symptoms commence associated with gastric and small bowel polyps (True) Explanation: Also with lipomas, epidermoid cysts, osteomas and desmoid tumours best treated with immunosuppressant therapy in patients aged < 20 years (False) Explanation: Immunosuppressives have no role; prophylactic colectomy is warranted Module 18 (Chapter 18) Question 1. Bilirubin is derived exclusively from the breakdown of haemoglobin (False) Explanation: Also from catabolism of other haem-containing proteins (e.g. myoglobin) bound in the unconjugated form to plasma â-globulin (False) Explanation: Bound to albumin conjugated in the microsomes of the hepatocytes (True) Explanation: By enzymes of the smooth endoplasmic reticulum reabsorbed in the small bowel as bilirubin diglucuronide (False) Explanation: Only reabsorbed after metabolism to stercobilinogen normally excreted as stercobilinogen in the faeces and as urobilinogen in the urine (True) Explanation: And as the oxidation products stercobilin and urobilin Question 2. The concentration of conjugated bilirubin in the serum in haemolytic anaemia is typically increased (False) Explanation: Unconjugated hyperbilirubinaemia urine of healthy subjects is typically undetectable (True) Explanation: As almost all bilirubin is unconjugated and albumin-bound serum normally constitutes most of the total serum bilirubin (False) Explanation: Most of the serum bilirubin is unconjugated serum in Gilbert's syndrome is typically increased (False) Explanation: Unconjugated bilirubin is increased urine in viral hepatitis parallels that of urobilinogen (False) Explanation: Urobilinogen is an unreliable indicator of hepatobiliary disease Question 3. The serum alanine aminotransferase (ALT) concentration is derived from a microsomal enzyme specific to hepatocytes (False) Explanation: Neither ALT nor AST is specific to the liver typically more than six times normal in alcoholic hepatitis (False) Explanation: Not usually > three times normal usually normal in both obstructive and haemolytic jaundice (False) Explanation: May be elevated in either likely to rise and fall in parallel with the serum bilirubin in viral hepatitis (False) Explanation: Changes in serum ALT precede changes in the serum bilirubin likely to increase in response to enzyme-inducing drug therapy (False) Explanation: Only the gamma-glutamyl transferase levels increase Question 4. The serum alkaline phosphatase concentration is derived from the liver, bone, small bowel and placenta (True) Explanation: Therefore not specific to liver disease typically increased to more than six times normal in viral hepatitis (False) Explanation: Not usually > 2.5 times normal derived mainly from hepatic sinusoidal and canalicular membranes (True) Explanation: Excess synthesis in cholestasis of particular prognostic value in chronic liver disease (False) Explanation: No prognostic value increased more in extrahepatic than in intrahepatic cholestasis (False) Explanation: No site-specific pattern

By A. H.

MCQs VIA WEB 2005 Question 5. In the investigation of suspected liver disease ultrasonography reliably distinguishes solid from cystic masses (True) ultrasonography reliably excludes liver disease (False) Explanation: May appear normal in disease normal liver function values exclude significant liver disease (False) Explanation: May be normal in 10-15% of patients with cirrhosis the mortality rate of percutaneous liver biopsy is about 5% (False) Explanation: Approximately 0.05% ascitic protein concentrations > 25 g/l are compatible with a diagnosis of carcinomatosis (True) Explanation: And tuberculosis and hepatic vein obstruction; protein concentration < 30 g/l = transudate Question 6. Characteristic features of Gilbert's syndrome include an autosomal recessive mode of inheritance (False) Explanation: Typically autosomal dominant decreased hepatic glucuronyl transferase activity (True) Explanation: Causing failure of bilirubin conjugation unconjugated hyperbilirubinaemia < 100 µmol/l (True) Explanation: And no abnormality of other liver function tests serum bilirubin concentration increased by fasting (True) Explanation: Sometimes used as a diagnostic test increased serum bile acid concentrations (False) Explanation: Unconjugated hyperbilirubinaemia is the sole abnormality Question 7. Characteristic features of cholestatic jaundice include dark green stools (False) Explanation: Typically pale stools-steatorrhoea dark brown urine (True) Explanation: Due to conjugated bilirubinuria unconjugated hyperbilirubinaemia (False) Explanation: Conjugated hyperbilirubinaemia serum alkaline phosphatase concentration > 2.5 times normal (True) Explanation: Diagnostic feature increased serum bile acid concentrations (True) Question 8. Typical causes of extrahepatic cholestatic jaundice include sclerosing cholangitis (False) Explanation: Intrahepatic primary biliary cirrhosis (False) Explanation: Intrahepatic cystic fibrosis (True) Explanation: Common bile duct obstruction from chronic pancreatitis alcoholic cirrhosis (False) Explanation: Intrahepatic non-alcoholic steatohepatitis (False) Explanation: Rarely causes jaundice Question 9. The following features suggest extrahepatic cholestasis rather than viral hepatitis a palpable gallbladder (True) Explanation: E.g. pancreatic carcinoma right hypochondrial tenderness (False) Explanation: Also common in acute hepatitis serum alkaline phosphatase concentration > 2.5 times normal (True) pruritus and rigors (True) Explanation: Suggests obstruction with cholangitis peripheral blood polymorph leucocytosis (True) Explanation: Sometimes relative lymphocytosis in viral hepatitis

By A. H.

MCQs VIA WEB 2005 Question 10. The typical causes of macrovesicular steatosis include alcohol misuse (True) Explanation: Often asymptomatic pregnancy (False) Explanation: Microvesicular steatosis Reye's syndrome (False) Explanation: Microvesicular steatosis severe malnutrition (True) Explanation: Steatohepatitis (macrovesicular steatosis with hepatocyte necrosis) can be serious diabetes mellitus (True) Explanation: Common and benign Question 11. The typical features of type A viral hepatitis (HAV) include picornavirus infection spread by the faecal-oral route (True) an incubation period of 3 months (False) Explanation: 2-4 weeks a greater risk of acute liver failure in the young than in the old (False) Explanation: But children are more frequently infected right hypochondrial pain and tenderness (True) Explanation: Non-specific findings of acute hepatitis progression to cirrhosis if cholestasis is prolonged (False) Explanation: Chronic hepatitis does not occur Question 12. The following statements about type A viral hepatitis are true persistent viraemia produces the post-hepatitis syndrome (False) Explanation: Viraemia is only transient in hepatitis A relapsing hepatitis usually indicates a poorer prognosis (False) Explanation: Spontaneous recovery is the typical outcome the virus is not usually transmitted via infected blood (True) Explanation: But a recognised rarity drug-induced acute hepatitis produces similar liver histology (True) Explanation: Serological investigations should help distinguish travellers given immune serum globulin are protected for 3 months (True) Explanation: Some will have natural endogenous protection Question 13. Circulating hepatitis B surface antigen (HBsAg) is detectable during the prodrome of acute type B hepatitis (True) Explanation: A reliable marker of hepatitis B infection a DNA viral particle transmissible in all body fluids (True) Explanation: A DNA hepadna virus likely to persist in about 50% of adults following acute type B hepatitis (False) Explanation: Chronic carriage occurs in 5-10% of adults invariably present in a patient with jaundice attributable to type B hepatitis infection (False) Explanation: Alternative serological evidence of infection should be sought commoner in asymptomatic subjects in the Western rather than the Eastern hemisphere (False) Explanation: Carriage rates are highest in the Middle East and Far East Question 14. The typical features of type B viral hepatitis (HBV) include an incubation period of 1 month (False) Explanation: Average incubation 3 months history of exposure to unsafe sex or drug misuse (True) Explanation: Or other exposure to blood or blood products prodromal illness with polyrtharalgia (True) Explanation: May cause serum sickness hepatitic illness more severe than with type A virus (True) Explanation: Hepatitis A is usually a mild illness

By A. H.

MCQs VIA WEB 2005 absence of progression to chronic hepatitis (False) Explanation: And hepatic cirrhosis also occurs Question 15. In hepatitis C (HCV) a chronic carriage rate of > 50% is the rule (True) Explanation: With varying degrees of severity the infecting agent is an RNA flavivirus (True) the disease does not progress to chronic hepatitis (False) Explanation: Hepatitis C may progress to chronic disease most patients experience the symptoms of acute hepatitis (False) Explanation: Most patients are asymptomatic; incubation period is 2-26 weeks the virus is responsible for 90% of all post-transfusion hepatitis (True) Explanation: Although serological screening methods have greatly reduced this Question 16. The typical features of acute (fulminant) hepatic failure include onset within 8 weeks of the initial illness (True) Explanation: Without evidence of pre-existing liver disease hepatosplenomegaly and ascites (False) Explanation: Suggest chronic liver disease encephalopathy and fetor hepaticus (True) Explanation: With confusion and asterixis (liver flap) nausea, vomiting and renal failure (True) Explanation: Renal failure is an ominous development cerebral oedema without papilloedema (True) Explanation: Occurs late, if at all Question 17. Typical liver function values in acute hepatic failure include hypoalbuminaemia (False) Explanation: Serum albumin has a long half-life hypoglycaemia (True) Explanation: Impaired hepatic gluconeogenesis prolonged prothrombin time (True) Explanation: Useful in determining prognosis serum alkaline phosphatase > 6 times normal (False) Explanation: Typically not so elevated, unlike the serum transaminases peripheral blood lymphocytosis (False) Explanation: May be a polymorphonuclear leucocytosis Question 18. The clinical features of autoimmune hepatitis include an association with autoimmune thyroiditis (True) Explanation: Type I autoimmune liver disease is associated with Graves' disease and Hashimoto's thyroiditis acute onset simulating viral hepatitis in 25% of patients (True) Explanation: Occurs in 25% of patients but symptoms and signs then persist arthralgia, fever and amenorrhoea (True) Explanation: And fatigue, anorexia and jaundice spider telangiectasia and hepatosplenomegaly (True) Explanation: And other signs of chronic liver disease Cushingoid facies, hirsutism and acne (True) Explanation: Altered steroid hormone metabolism Question 19. The typical features of hepatic cirrhosis include a small shrunken liver (True) Explanation: Liver size reduces as disease progresses painful splenomegaly (False) Explanation: Painless splenomegaly due to portal hypertension peripheral blood macrocytosis (True) Explanation: Particularly in alcoholic liver disease

By A. H.

MCQs VIA WEB 2005 parotid gland enlargement (True) Explanation: Particularly in alcoholic cirrhosis central cyanosis (True) Explanation: Hepatopulmonary syndrome associated with pulmonary telangiectasia Question 20. Typical features of hepatic encephalopathy include disordered sleep and loss of concentration (True) Explanation: Grade 1 aggressive behaviour and personality change (True) Explanation: Grade 2 yawning and hiccuping (True) Explanation: And asterixis (hepatic flap) drowsiness and disorientation (True) Explanation: Grade 3 confusion progressing to coma (True) Explanation: Grade 4 Question 21. Causes of ascites in the absence of intrahepatic liver disease include congestive cardiac failure (True) Explanation: Also constrictive pericarditis-transudate nephrotic syndrome (True) Explanation: Also protein-losing enteropathy-transudate peritoneal tuberculosis (True) Explanation: Also carcinomatosis-exudate lymphatic obstruction (True) Explanation: Chylous effusion Budd-Chiari syndrome (True) Explanation: Transudate associated with hepatic vein occlusion Question 22. In the management of ascites due to hepatic cirrhosis the dietary sodium intake should be restricted to 140 mmol/day (False) Explanation: Restriction < 40 mmol/day is usually required paracentesis and parenteral albumin replacement improve the survival rate (False) Explanation: A palliative, symptomatic measure with no prognostic value the daily calorie intake should be restricted to 1500 calories (False) Explanation: Calorie restriction is neither required nor desirable diuretic therapy should achieve a daily weight loss of at least 2.5 kg (False) Explanation: Daily weight loss > 1 kg may precipitate renal impairment and/or encephalopathy the protein intake should be at least 40 g/day unless encephalopathy is suspected (True) Explanation: Restriction may be necessary to control encephalopathy Question 23. Causes of portal hypertension include alcoholic cirrhosis (True) Explanation: Intrahepatic parenchymal myeloproliferative disease (True) Explanation: Intrahepatic pre-sinusoidal hepatic schistosomiasis (True) Explanation: Intrahepatic pre-sinusoidal; also sarcoidosis abdominal trauma (True) Explanation: Extrahepatic pre-sinusoidal (portal vein thrombosis) hepatic vein obstruction (Budd-Chiari syndrome) (True) Explanation: Extrahepatic post-sinusoidal Question 24. Complications of portal hypertension include variceal haemorrhage (True) Explanation: Oesophageal, gastric, stomal and rectal varices congestive gastropathy (True)

By A. H.

MCQs VIA WEB 2005 Explanation: Associated with hypergastrinaemia hepatorenal failure (True) Explanation: Associated with reduced renal blood flow hepatic encephalopathy (True) ascites (True) Explanation: And hypersplenism Question 25. In the management of acute variceal bleeding due to hepatic cirrhosis the priority is to restore normovolaemia (True) Explanation: Untreated, shock dramatically reduces liver blood flow and liver function pharmacological therapy is more effective than variceal banding or sclerotherapy (False) Explanation: Local measures stop bleeding in 80% of patients somatostatin (octreotide) and vasopressin both reduce portal venous pressure (True) Explanation: Constrict splanchnic arterioles; glyceryl trinitrate is given to reduce vasoconstriction balloon tamponade is best undertaken after endoscopic confirmation of bleeding varices (True) Explanation: Unless the patient is exsanguinating; 20% of patients are bleeding from non-variceal causes transjugular intrahepatic portosystemic stent shunting (TIPSS) is contraindicated in hepatic failure (False) Explanation: TIPSS is used when local measures fail and has replaced emergency shunt surgery Question 26. Prevention of recurrent variceal bleeding is achievable using somatostatin (octreotide) therapy (False) Explanation: Somatostatin may be useful in acute bleeds TIPSS (True) Explanation: Also used in acute variceal bleeding â-adrenoceptor antagonist (â-blocker) treatment (True) Explanation: â-blockers reduce portal pressure variceal banding (True) Explanation: Better than sclerotherapy in the elective situation sclerotherapy (True) Explanation: Easier than banding in the emergency situation Question 27. In primary biliary cirrhosis middle-aged males are affected predominantly (False) Explanation: Middle-aged females pruritus is invariably accompanied by jaundice (False) Explanation: May precede jaundice by months or years osteomalacia and osteoporosis both occur as the disease progresses (True) Explanation: Vitamin D malabsorption and hepatic osteodystrophy rigors and abdominal pain are presenting features (False) Explanation: Suggests obstruction of large bile duct smooth muscle antibodies are present in high titres in the serum (False) Explanation: High titres of antimitochondrial antibody Question 28. The typical features of primary haemochromatosis include association with an autosomal dominant pattern of inheritance (False) Explanation: Inherited as an autosomal recessive gene located on chromosome 6 male predominance (True) Explanation: 90% are males; females may be protected by menstruation and pregnancy hepatic cirrhosis and diabetes mellitus (True) Explanation: 'Bronzed diabetes' congestive cardiomyopathy (True) Explanation: May be a congestive cardiomyopathy grey skin pigmentation due to ferritin deposition (False) Explanation: Melanin not iron deposition Question 29. The typical features of Wilson's disease include haemolytic anaemia (True)

By A. H.

MCQs VIA WEB 2005 Explanation: Sometimes accompanying an acute hepatitis in children acute hepatitis and chronic hepatitis (True) Explanation: Or acute hepatic failure or cirrhosis parkinsonian syndrome and hepatic cirrhosis (True) Explanation: A variety of extrapyramidal syndromes may be seen Kayser-Fleischer rings (True) Explanation: Kayser-Fleischer rings are an important diagnostic clue renal tubular acidosis (True) Explanation: Copper is deposited in the liver and kidneys Question 30. The typical features of alcoholic liver disease include microvesicular steatosis (False) Explanation: Macrovesicular steatosis is the earliest stage when abstinence will achieve a good prognosis acute hepatitis and chronic hepatitis (True) Explanation: 33% mortality if liver dysfunction is severe hepatic cirrhosis (True) Explanation: 50% 5-year survival after the initial presentation if abstinent cholestatic jaundice (True) Explanation: Often associated with tender hepatomegaly and abdominal pain alcohol intake > 30 g/day for > 5 years (True) Explanation: Usually associated with at least 50 g/day for at least 10 years Question 31. The typical features of hepatocellular carcinoma include fever, weight loss and abdominal pain (True) Explanation: Abdominal pain and a cirrhotic liver suggest hepatoma ascites and intra-abdominal bleeding (True) Explanation: Tumours are vascular and spread locally arterial bruit over the liver (True) Explanation: There may also be a hepatic rub rising serum á-fetoprotein titre (True) Explanation: Rises in 90% of cases surgically resectable disease in 50% of patients (False) Explanation: Only 10% are suitable for surgery Question 32. Pyogenic liver abscess is a recognised complication of ascending cholangitis (True) Explanation: Secondary to biliary obstruction Crohn's disease (True) Explanation: Secondary to portal pyaemia pancreatitis (True) Explanation: Acute pancreatitis septicaemia (True) Explanation: Infection via hepatic artery subphrenic abscess (True) Explanation: Direct local spread Question 33. The typical features of pyogenic liver abscess include obstructive jaundice and pruritus (False) Explanation: Jaundice is usually mild and not often obstructive tender hepatomegaly without splenomegaly (True) Explanation: Splenomegaly suggests coexistent pathology pleuritic pain and pleural effusion (True) Explanation: May be right shoulder tip pain multiple abscesses, especially in ascending cholangitis (True) Explanation: Single lesions are more common in the right liver Escherichia coli, anaerobes and streptococci present in pus (True) Explanation: Multiple organisms in one-third of cases

By A. H.

MCQs VIA WEB 2005 Question 34. Gallstones are a recognised complication of obesity (True) Explanation: Increased hepatic cholesterol secretion pregnancy (True) Explanation: Increased hepatic cholesterol secretion and impaired gallbladder motility chronic haemolytic anaemia (True) Explanation: Pigment stones terminal ileal disease (True) Explanation: Pigment stones rapid weight loss (True) Explanation: Increased hepatic cholesterol secretion Question 35. The typical clinical features of acute cholecystitis include jaundice, nausea and vomiting (False) Explanation: Jaundice occurs in less than 20% even in the absence of stones (Mirizzi's syndrome) colicky abdominal pain in spasms lasting about 5 minutes (False) Explanation: Pain is typically continuous for up to 6 hours right hypochondrial tenderness worse on inspiration (True) Explanation: Murphy's sign air in the biliary tree on plain radiograph (False) Explanation: May follow passage of a gallstone into intestine or biliary surgery peripheral blood leucocytosis (True) Explanation: May be absent in the elderly Question 36. The post-cholecystectomy syndrome is characteristically associated with patients with previous acalculous cholecystitis (True) Explanation: Less common in patients with previous typical biliary colic and gallstones females with a history of abdominal pain > 5 years in duration (True) Explanation: Associated with the irritable bowel syndrome and functional dyspepsia retained stones in the common bile duct (True) Explanation: Hence the need to investigate this possibility dysfunction of the sphincter of Oddi (False) Explanation: This abnormality may not be causal and may in fact result from cholecystectomy early postoperative complications (True) Explanation: Suggest the possibility of a biliary stricture Module 19 (Chapter 19) Question 1. Peripheral blood lymphocytosis would be an expected finding in brucellosis (True) Explanation: Often with neutropenia pneumococcal pneumonia (False) Explanation: Polymorphonuclear leucocytosis measles and rubella (True) Explanation: Non-specific feature of many viral infections Hodgkin's disease (False) Explanation: Non-Hodgkin's lymphoma chronic lymphatic leukaemia (True) Explanation: Predominantly small lymphocytes

Question 2. Peripheral blood neutrophil leucocytosis would be an expected finding in connective tissue disease (True) Explanation: Or may be neutropenia in systemic lupus erythematosus corticosteroid therapy (True) Explanation: And lithium therapy pregnancy (True) Explanation: Variable, increases at delivery

By A. H.

MCQs VIA WEB 2005 whooping cough (False) Explanation: Typically lymphocytosis mesenteric infarction (True) Explanation: And myocardial infarction Question 3. Platelets have a circulation lifespan of 10 hours in healthy subjects (False) Explanation: 10-day lifespan are produced and regulated under the control of thrombopoietins (True) Explanation: By the megakaryocytes contain small nuclear remnants called Howell-Jolly bodies (False) Explanation: Found in red blood cells decrease in number in response to aspirin therapy (False) Explanation: May increase release 5-hydroxytryptamine (5-HT, serotonin) and von Willebrand factor (vWF) (True) Explanation: 5-HT (delta granules), and vWF and fibrinogen (alpha granules) Question 4. The following statements about red blood cell morphology are true hypochromia is pathognomonic of iron deficiency (False) Explanation: Seen in other disorders of haemoglobin synthesis (e.g. thalassaemia) polychromasia indicates active production of new red blood cells (True) Explanation: Residual ribosomal material is stained faintly poikilocytosis is invariably associated with anisocytosis (True) Explanation: Sign of dyserythropoiesis punctate basophilia is a typical feature of beta-thalassaemia (True) Explanation: And lead poisoning target cells are associated with hyposplenism and liver disease (True) Explanation: And haemoglobinopathies Question 5. Peripheral blood findings in dietary iron deficiency include microcytosis (True) Explanation: Microcytosis is the first sign ovalocytosis (True) Explanation: Sometimes poikilocytosis mean corpuscular haemoglobin concentration < 50% of normal (False) Explanation: Only in severe anaemia; hypochromia is due to microcytosis Howell-Jolly bodies (False) Explanation: Suggests hyposplenism thrombocytosis (True) Explanation: Thrombocytosis occurs even in the absence of bleeding Question 6. In the treatment of iron deficiency anaemia with iron folic acid should also be given if the anaemia is severe (False) Explanation: Only if coexistent deficiency demonstrated treatment is stopped as soon as haemoglobin normalises (False) Explanation: Continue for 3 months to replenish stores haemoglobin should rise by 1 g/l every 7-10 days (False) Explanation: 10 g/l every 10 days unless there is malabsorption, bleeding or poor compliance maximal reticulocyte count usually develops within 1-2 days (False) Explanation: Peak reticulocyte count at 7-10 days parenteral iron is usually more effective than oral iron (False) Explanation: Oral iron is usually effective Question 7. Hypochromic microcytic anaemia is a recognised finding in haemolytic anaemia (False) Explanation: Macrocytic with polychromasia myelodysplastic syndrome (True)

By A. H.

MCQs VIA WEB 2005 Explanation: Typically a dimorphic red cell population hypothyroidism (False) Explanation: Typically macrocytic beta-thalasaemia (True) Explanation: And other thalassaemias rheumatoid arthritis (True) Explanation: Or a normochromic normocytic picture Question 8. Normocytic normochromic anaemia is an expected feature of alcoholic liver disease (False) Explanation: Typically macrocytic chronic renal failure (True) Explanation: Erythropoietin deficiency rheumatoid arthritis (True) Explanation: Typically macrocytic kwashiorkor (True) Explanation: Protein-energy malnutrition strict vegetarianism (False) Explanation: Anaemia is rare in modest reductions of dietary vitamin B12 intake Question 9. Macrocytic anaemia is a typical finding in folic acid deficiency (True) Explanation: With megaloblastic marrow haemolytic anaemia (True) Explanation: With polychromasia alcohol misuse (True) Explanation: With or without cirrhosis primary sideroblastic anaemia (False) Explanation: Dimorphic, with microcytic population myelodysplastic syndrome (True) Explanation: But variable red cell morphology Question 10. Typical haematological findings in megaloblastic anaemia include pancytopenia and oval macrocytosis (True) Explanation: Commonly due to vitamin B12 deficiency neutrophil leucocyte hypersegmentation (True) Explanation: Shift to the right in the nuclear segmentation count (Arneth count) anisocytosis and poikilocytosis (True) Explanation: And red cell fragmentation reticulocytosis and polychromasia (False) Explanation: Features of bleeding or haemolysis excess urinary urobilinogen and bilirubinuria (False) Explanation: Bilirubinuria is not a feature of any anaemia Question 11. Folate and vitamin B12 deficiency both typically produce subacute combined degeneration of the spinal cord (False) Explanation: Feature of vitamin B12 deficiency only intermittent glossitis and diarrhoea (True) Explanation: Glossitis less common in folate deficiency mild jaundice and splenomegaly (True) Explanation: Mild haemolysis peripheral neuropathy (True) marked weight loss (True) Explanation: Partially dependent on underlying cause Question 12. Characteristic features of Addisonian pernicious anaemia include onset before the age of 20 years (False)

By A. H.

MCQs VIA WEB 2005 Explanation: Typically 45-65 years gastric parietal cell and intrinsic factor antibodies in the serum (True) Explanation: Found in 90% and < 50% respectively increased serum bilirubin and lactate dehydrogenase concentrations (True) Explanation: Mild haemolysis occurs four-fold increase in the risk of developing gastric carcinoma (True) Explanation: Associated gastric atrophy Schilling test usually reverts to normal with intrinsic factor (True) Explanation: Failure to correct suggests terminal ileal disease Question 13. Causes of folic acid deficiency include vegetarian diet (False) Explanation: Caused by inadequate vegetable intake gluten enteropathy (True) Explanation: Characteristic finding pregnancy (True) Explanation: Increased requirements haemolytic anaemia (True) Explanation: Increased requirements antibiotic therapy (False) Explanation: Methotrexate and phenytoin may cause folate deficiency Question 14. Characteristic features of primary aplastic anaemia include peak incidence in the elderly (False) Explanation: Peaks about 30 years of age normocytic normochromic anaemia with thrombocytosis (False) Explanation: Thrombocytopenia bone marrow trephine is required to confirm the diagnosis (True) Explanation: Diagnosis cannot be made on peripheral blood film alone splenomegaly indicating extramedullary erythropoiesis (False) Explanation: Splenomegaly occurs in under 10% of cases pancytopenia (True) Explanation: Typical Question 15. Typical features suggesting intravascular haemolysis include bilirubinuria and haemoglobinuria (False) Explanation: Bilirubin is unconjugated therefore not found in urine methaemalbuminaemia and haemosiderinuria (True) Explanation: The latter always indicating intravascular haemolysis increased serum haptoglobin concentration (False) Explanation: Decreased serum haptoglobin increased plasma haemoglobin concentration (True) Explanation: Most is bound to serum haptoglobin splenomegaly (True) Explanation: Often with reticulocytosis Question 16. Laboratory features suggesting haemolytic anaemia include increased serum lactate dehydrogenase (LDH) concentration (True) Explanation: Red cells are rich in LDH conjugated hyperbilirubinaemia and bilirubinuria (False) Explanation: Unconjugated hyperbilirubinaemia and excess urobilinogen in the urine peripheral blood neutrophil leucocytosis (True) Explanation: Also red cell abnormalities (e.g. spherocytes) peripheral blood polychromasia and macrocytosis (True) Explanation: Reflects reticulocytosis bone marrow erythroid hyperplasia (True) Explanation: With megaloblastic change if folate deficiency is also present

By A. H.

MCQs VIA WEB 2005 Question 17. Typical features of hereditary spherocytosis include splenomegaly (True) Explanation: Also pigment gallstones intravascular haemolysis (False) Explanation: Red blood cell destruction occurs in the spleen decreased red blood cell osmotic fragility (False) Explanation: Osmotic fragility is increased transient aplastic anaemia (True) Explanation: Often in association with parvovirus infection deficiency of red cell spectrin (True) Explanation: Red blood cell membrane protein Question 18. The typical clinical features of sickle-cell anaemia include haemolytic and aplastic crises (True) Explanation: Often precipitated by viral infection neonatal spherocytic haemolytic anaemia (False) Explanation: Not until HbF levels fall after the age of 3 months pulmonary, splenic and mesenteric infarcts (True) Explanation: Causing pleuritic pain and also renal infarcts splenomegaly with hypersplenism (False) Explanation: Splenic atrophy and functional hyposplenism bone necrosis and osteomyelitis (True) Explanation: Painful bone infarcts Question 19 In patients with sickle-cell disease, acute painful crises are likely to be precipitated by high altitude (True) Explanation: Decreased PaO2 pregnancy (True) Explanation: May present as pseudo-toxaemia syndrome dehydration (True) Explanation: Rehydration is an essential component of therapy systemic infection (True) Explanation: Treat promptly to prevent sickle-cell crises hypoxia (True) Question 20. The typical features of the beta-thalassaemias include macrocytic anaemia (False) Explanation: Typically hypochromic microcytic anaemia hepatosplenomegaly (True) Explanation: In the 'major' (homozygous) form pigment gallstones (True) Explanation: Pigment gallstones can be associated with chronic haemolysis neonatal haemolytic anaemia (False) Explanation: Not until HbF synthesis declines bone infarcts (False) Explanation: Unlike sickle cell disease Question 21. The typical features of autoimmune haemolytic anaemia include peripheral blood spherocytosis and splenomegaly (True) Explanation: Characteristic haemoglobinuria and haemosiderinuria (True) Explanation: Suggesting intravascular haemolysis increased serum haptoglobin concentration (False) Explanation: Decreased serum haptoglobin concentration positive Coombs test (True) Explanation: Warm usually IgG, cold usually IgM association with lymphoproliferative disease (True)

By A. H.

MCQs VIA WEB 2005 Explanation: Chronic lymphatic leukaemia, lymphoma and also systemic lupus erythematosus Question 22. The typical features of polycythaemia rubra vera include peak prevalence aged > 40 years (True) splenomegaly, leucocytosis and thrombocytosis (True) Explanation: And elevated red cell mass headaches, pruritus and peptic ulcer dyspepsia (True) Explanation: But may be asymptomatic decreased leucocyte alkaline phosphatase score (False) Explanation: A feature of chronic myeloid leukaemia increased blood viscosity associated with vascular disease (True) Explanation: E.g. increased risk of stroke Question 23. Acute lymphoblastic leukaemia (ALL) has a peak prevalence in patients aged 20-30 years (False) Explanation: Peaks in childhood typically produces cytoplasmic Auer rods in blast cells (False) Explanation: Acute myeloblastic leukaemia (AML) has a median survival of 30 months with chemotherapy (True) Explanation: AML has a 40% 5-year survival with chemotherapy is the most common of all acute leukaemias (False) Explanation: AML is four times more common than ALL is a typical complication of multiple myeloma (False) Explanation: May complicate myelofibrosis Question 24. Clinical features of chronic myeloid leukaemia (CML) include painful splenomegaly (True) Explanation: Splenomegaly in 90% of cases gout and arthralgia (True) Explanation: Hyperuricaemia is often asymptomatic generalised lymphadenopathy (False) Explanation: Atypical feature tendency to bleeding and bruising (True) Explanation: Variable platelet dysfunction median survival of 15 years with chemotherapy (False) Explanation: Median survival 5 years Question 25. The typical laboratory findings in chronic myeloid leukaemia include leucoerythroblastic anaemia and thrombocytosis (True) Explanation: Platelet count falls after blast transformation peripheral blood neutrophilia, eosinophilia and basophilia (True) chromosomal translocation q-22/q+9 (True) Explanation: Philadelphia chromosome increased neutrophil leucocyte alkaline phosphatase (LAP) score (False) Explanation: Usually decreased LAP score transformation to acute leukaemia (True) Explanation: Transformation results to either ALL (30%) or acute myeloid leukaemia (AML) (70%) Question 26. Typical features of chronic lymphocytic leukaemia include onset in younger patients than in chronic myeloid leukaemia (False) Explanation: Peak age 65 years development of autoimmune haemolytic anaemia (True) Explanation: Typically warm antibody presentation with massive hepatosplenomegaly (False) Explanation: Mild organomegaly only lymphadenopathy associated with recurrent infections (True) Explanation: Bacterial more than viral

By A. H.

MCQs VIA WEB 2005 median survival of 15 years following chemotherapy (False) Explanation: Overall median survival 6 years Question 27. The typical laboratory features in chronic lymphocytic leukaemia include hyperuricaemia and thrombocytosis (False) Explanation: Mild thrombocytopenia with urate usually normal hypogammaglobulinaemia (True) Explanation: Associated with a paraproteinaemia in 5% peripheral blood lymphocytosis in the absence of lymphoblasts (True) Explanation: Total WCC typically 50-200 × 109/l positive Coombs test (True) Explanation: May be associated with haemolysis transformation to acute leukaemia (False) Explanation: Transformation is rare Question 28. Allogeneic bone marrow transplantation is particularly useful in the treatment of multiple myeloma (True) Explanation: Also useful in acute myelofibrosis severe aplastic anaemia (True) alpha-thalassaemia (True) Explanation: All severe thalassaemias severe combined immunodeficiency disorder (True) chronic lymphocytic leukaemia (False) Explanation: But useful in most other acute and chronic leukaemias Question 29. Complications of allogeneic bone marrow transplantation include acute graft-versus-host disease (True) Explanation: Usually occurs 2-3 weeks after the graft and is associated with infection severe infection (True) Explanation: A major problem, especially with viruses and atypical microorganisms infertility (True) Explanation: Important given the age of many of the patients pneumonitis (True) malignant disease during long-term follow-up (True)

Question 30. The presence of lymphadenopathy and splenomegaly would be expected findings in multiple myeloma (False) Explanation: Neither is characteristic chronic lymphocytic leukaemia (True) Explanation: Mild splenomegaly, generalised lymphadenopathy chronic myeloid leukaemia (False) Explanation: Moderate to massive splenomegaly, no lymphadenopathy infectious mononucleosis (True) Explanation: Usually both mild myelofibrosis (False) Explanation: Splenomegaly without lymphadenopathy Question 31. Recognised clinical features of multiple myeloma include peak incidence between the ages of 30 and 50 years (False) Explanation: Peak prevalence in males aged 60-70 years secondary amyloidosis (True) Explanation: Amyloidosis occurs in 10% of cases median survival > 10 years with chemotherapy (False) Explanation: Median survival of 40 months recurrent infections and pancytopenia (True) Explanation: Reduction of normal plasma cells causes immunodeficiency

By A. H.

MCQs VIA WEB 2005 increased serum calcium, urate and blood urea (True) Explanation: All of which may be asymptomatic Question 32. In differentiating multiple myeloma from a benign monoclonal gammopathy, the following findings would favour the diagnosis of multiple myeloma monoclonal gammopathy with normal serum immunoglobulin levels (False) Explanation: Myeloma produces suppression of the other serum immunoglobulins bone marrow plasmacytosis of > 20% (True) Explanation: A diagnostic prerequisite bilateral carpal tunnel syndrome (True) Explanation: Amyloidosis also causes a restrictive cardiomyopathy Bence Jones proteinuria (True) Explanation: But the serum paraprotein may be undetectable multiple osteolytic lesions on radiograph (True) Explanation: Malignant infiltration is typically associated with a normal isotope bone scan Question 33. The clinical features of Hodgkin's disease include painless cervical lymphadenopathy (True) Explanation: Usually painless anaemia due to bone marrow involvement (False) Explanation: Unlike non-Hodgkin's lymphoma impaired T-cell function in the absence of lymphopenia (True) Explanation: Lymphopenia suggests poor prognosis fever and weight loss (True) Explanation: Stage B median survival > 10 years (True) Explanation: Dependent on staging at presentation Question 34. Typical characteristics of non-Hodgkin's lymphoma include low-grade lymphomas rapidly produce symptoms due to high cell proliferation rates (False) Explanation: Indolent and often asymptomatic course with low cell proliferation rates bone marrow and splenic involvement are present from the onset (True) Explanation: Typically extranodal at diagnosis isolated involvement of gastric mucosa associated with Helicobacter pylori infection (True) Explanation: MALToma may be cured by H. pylori eradication the majority are T-cell rather than B-cell in origin (False) Explanation: 70% are B-cell tumours better prognosis in high-grade rather than low-grade lymphomas (True) Explanation: Prognosis is also stage- and age-dependent Question 35. Recognised causes of thrombocytopenia include megaloblastic anaemia (True) Explanation: Often with leucopenia acquired immunodeficiency syndrome (True) Explanation: Primary, or secondary to superimposed infections disseminated intravascular coagulation (True) Explanation: Increased peripheral consumption of platelets von Willebrand's disease (False) Explanation: The platelet count is normal aspirin therapy (True) Explanation: Also many commonly used drugs including heparin and â-blockers Question 36. Typical features of idiopathic thrombocytopenic purpura include IgG-mediated thrombocytopenia (True) Explanation: Can therefore be transmitted transplacentally peak prevalence in patients aged > 60 years old (False) Explanation: Usually the young and commoner in females

By A. H.

MCQs VIA WEB 2005 prolongation of the bleeding time (True) Explanation: Other clotting tests normal splenomegaly (False) Explanation: Suggests other causes of thrombocytopenia prompt response to corticosteroid therapy (True) Explanation: Particularly in children Question 37. The prothrombin time is typically prolonged in disorders of the intrinsic pathway (False) Explanation: The extrinsic pathway factor X deficiency (True) Explanation: The Stuart-Prower factor factor VII deficiency (True) Explanation: First factor in extrinsic pathway factor V deficiency (True) Explanation: Also affects the activated partial thromboplastin time factor XII deficiency (False) Explanation: Disorder of the intrinsic pathway Question 38. The activated partial thromboplastin time (APTT) is typically prolonged in disorders of the extrinsic pathway (False) Explanation: The intrinsic pathway factor VII deficiency (False) Explanation: Detected by prothrombin time factor VIII or X deficiency (True) Explanation: Factor X also influences prothrombin time factor XIII deficiency (False) Explanation: Specific assay to measure factor IX, XI or XII deficiency (True) Explanation: Initial factors in the intrinsic system Question 39. Disseminated intravascular coagulation is a complication of amniotic fluid embolism (True) Explanation: Initiated by thromboplastin incompatible blood transfusion (True) Explanation: An unusual complication hypovolaemic and anaphylactic shock (True) Explanation: Endothelial injury septicaemic shock (True) Explanation: Exogenous endotoxins carcinomatosis (True) Explanation: Commonly bronchial carcinoma Question 40. The bleeding time is characteristically prolonged in ascorbic acid deficiency (False) Explanation: Bleeding time is normal but petechial haemorrhages may occur thrombocytopenia (True) Explanation: Irrespective of its cause haemophilia (False) Explanation: No vessel wall or platelet defect warfarin therapy (False) von Willebrand's disease (True) Explanation: Secondary decrease in factor VIII level with a qualitative platelet defect Question 41. The following statements about severe haemophilia A are true the disorder is inherited in an X-linked recessive mode (True) Explanation: Prenatal diagnosis is possible

By A. H.

MCQs VIA WEB 2005 recurrent haemarthroses and haematuria are typical (True) Explanation: Usually not apparent until the age of 6 months activated partial thromboplastin time and prothrombin time are both prolonged (False) Explanation: Only the activated partial thromboplastin time is prolonged factor VIII has a biological half-life of about 12 days (False) Explanation: Half-life is 12 hours desmopressin therapy increases factor VIII concentrations (True) Explanation: Desmopressin (DDAVP) therapy is useful to limit exposure to blood products Question 42. The following statements about von Willebrand's disease are true the disorder is inherited in an X-linked recessive mode (False) Explanation: Autosomal dominant-gene locus on chromosome 12 it is characterised by a prolonged bleeding time (True) Explanation: And secondary reduction in factor VIII levels the von Willebrand factor (vWF) is synthesised by both platelets and endothelial cells (True) vWF is a carrier protein which is bound to factor VIII (True) deficiency of vWF is best treated by desmopressin (True) Explanation: Desmopressin (DDAVP) therapy increases vWF concentrations Question 43. Thrombophilia with a predisposition to recurrent venous thromboses is associated with the antiphospholipid antibody syndrome (True) Explanation: May present with recurrent spontaneous abortion antithrombin deficiency (True) Explanation: Decreased inactivation of factors IIa, VIIa, IXa, Xa, XIa, causing heparin resistance factor V Leiden (True) Explanation: Prolonged factor V activation; factor II Leiden increases plasma prothrombin levels polycythaemia rubra vera (True) Explanation: And chronic myeloid leukaemia-both are associated with thrombocytosis protein C deficiency (True) Explanation: And protein S deficiency-reduced inactivation of factors Va and VIIIa Question 44. Indications for warfarin anticoagulation include venous thromboembolism (True) Explanation: Maintain the prothrombin ratio in the range 2.0-4.0 arterial embolism (True) Explanation: Less effective in non-embolic peripheral vascular disease myocardial infarction (False) Explanation: Unless associated with mural thrombus atrial fibrillation (True) Explanation: Reduces the risk of arterial embolism mechanical prosthetic heart valves (True) Explanation: Reduces the risk of embolic clots and possibly endocarditis Question 45. The hazards of blood transfusion include urticaria (True) Explanation: Allergic reaction cardiac failure (True) Explanation: Volume overload-in patients with previous CCF, give prophylactic diuretic therapy development of Rhesus antibodies in a Rhesus-negative patient (True) Explanation: Particularly important in women of child-bearing age fever (True) Explanation: Allergic reaction to one or more of the constituents of the transfusion acute intravascular haemolysis (True) Explanation: Major ABO incompatibility is the likeliest cause Question 46. Clinical features suggesting an acute transfusion reaction include onset within an hour of starting the transfusion (True)

By A. H.

MCQs VIA WEB 2005 Explanation: Delayed haemolytic transfusion reaction occurs 5-7 days after the transfusion rigors and fever (True) Explanation: Stop the transfusion immediately chest and back pain (True) sudden loss of consciousness (False) Explanation: Unlikely in the absence of other premonitory changes development of hypotension and shock (True) Explanation: May be problematic in anaesthetised patients Module 20 (Chapter 20) Question 1. The following diseases are associated with antinuclear and/or rheumatoid factor antibodies infective endocarditis (True) Explanation: Chronic infections (e.g. tuberculosis, leishmaniasis and schistosomiasis) autoimmune thyroiditis (True) Explanation: Also found in myasthenia gravis Sjögren's syndrome (True) Explanation: And systemic lupus erythematosus, dermatomyositis and progressive systemic sclerosis fibrosing alveolitis (True) Explanation: And autoimmune hepatitis and sarcoidosis ankylosing spondylitis (False) Explanation: And, by definition, all the seronegative spondyloarthritides Question 2. The biochemical features listed below characterise the following metabolic bone disorders increased serum calcium, serum phosphate and serum alkaline phosphatase-osteoporosis (False) Explanation: All three are normal in osteoporosis normal serum calcium and serum phosphate but increased serum alkaline phosphatase-Paget's disease (True) Explanation: Occasionally the serum calcium may be elevated if immobilisation is prolonged normal serum calcium and serum alkaline phosphatase but decreased serum phosphate-osteomalacia (False) Explanation: All three may be normal (see E) decreased serum calcium, serum phosphate and serum alkaline phosphatase-metastatic bone disease (False) Explanation: Increased calcium, normal or low phosphate, and high serum alkaline phosphatase decreased serum calcium and serum phosphate but increased serum alkaline phosphatase-osteomalacia (True) Explanation: But all three may be normal Question 3. Presentation with acute monoarthritis suggests the possibility of crystal arthritis (True) Explanation: Gout and pseudogout trauma (True) Explanation: Trauma usually obvious bacterial infection (True) rheumatoid arthritis (False) Explanation: Usually polyarticular in onset enteropathic arthritis (True) Explanation: Reactive arthritis following enterically or sexually acquired infection Question 4. The following statements about infective arthritis are true the onset is typically insidious (False) Explanation: Onset usually acute, but less so in the elderly or the immunocompromised pre-existing arthritis is a recognised predisposing factor (True) Explanation: Also occurs after trauma or surgery small peripheral joints are involved more often than larger joints (False) Explanation: Large joints are most frequently affected Haemophilus influenzae is the commonest causative organism in adults (False) Explanation: H. influenzae is the main cause in children, streptococci and staphylococci in adults joint aspiration should be avoided given the risk of septicaemia (False) Explanation: Early joint aspiration is vital if the diagnosis is not to be delayed

By A. H.

MCQs VIA WEB 2005 Question 5. The following features of backache suggest mechanical or radicular pain rather than inflammatory pain radiation of pain down the back of one leg to the ankle (True) Explanation: Suggests lumbar nerve root compression an elevated C-reactive protein (CRP) (False) Explanation: Suggests an active inflammatory pathology localised tenderness over the greater sciatic notch (True) Explanation: Suggests lumbar nerve root compression gradual mode of onset in an elderly patient (False) Explanation: Suggests significant pathology even if there are no physical signs back pain and stiffness exacerbated by resting (False) Explanation: Suggests inflammatory disease Question 6. The typical findings in fibromyalgia include elevation of the ESR (False) Explanation: A high ESR suggests another diagnosis symptoms of fatigue and an irritable bowel (True) Explanation: Typical of most psychosomatic disorders coexistent anxiety and depression (True) rapid, spontaneous resolution (False) Explanation: Often very chronic musculoskeletal pain without local tenderness (False) Explanation: Multiple tender points are characteristic Question 7. Shoulder pain is a recognised feature of myocardial ischaemia (True) Explanation: Either alone or associated with central chest pain supraspinatus tendonitis (True) Explanation: With characteristic painful arc on shoulder abduction bronchial carcinoma (True) Explanation: Suggests extra-pleural spread or bony metastases pneumococcal pneumonia (True) Explanation: Classically due to diaphragmatic irritation secondary to pleurisy cervical spondylosis (True) Explanation: Due to cervical nerve root compression Question 8. In a patient with neck pain aggravation by sneezing suggests cervical disc prolapse (True) Explanation: Disc prolapse may also produce upper or lower limb neurological signs radiation to the occiput suggests disease affecting the upper cervical vertebrae (True) Explanation: Common in tension headache associated bilateral arm paraesthesiae suggest angina pectoris as the most likely diagnosis (False) Explanation: Suggest cervical radiculopathy and otherwise normal joints, rheumatoid arthritis is excluded as a possible diagnosis (False) Explanation: Rheumatoid arthritis typically involves atlantoaxial articulations Question 9. The clinical features of primary (nodal) osteoarthrosis include joint pain aggravated by rest and relieved by activity (False) Explanation: More suggestive of an inflammatory arthritis such as rheumatoid arthritis proximal interphalangeal and metacarpal-phalangeal joint involvement (False) Explanation: Typically distal interphalangeal joint involvement involvement of the hip, knee and spinal apophyseal joints (True) a strong family history of Heberden's nodes (True) microfractures of subchondral bone (True) Question 10. Causes of secondary osteoarthritis include acromegaly (True)

By A. H.

MCQs VIA WEB 2005 septic arthritis (True) Explanation: And any joint previously traumatised haemochromatosis (True) Explanation: Also chondrocalcinosis and Wilson's disease Perthes' disease (True) Explanation: And most hip dysplasias Ehlers-Danlos syndrome (True) Explanation: Also other causes of hypermobility Question 11. Criteria for the diagnosis of rheumatoid arthritis include morning stiffness lasting more than 1 hour (True) Explanation: American Rheumatism Association criteria (1998) arthritis in both hip joints (False) Explanation: Arthritis affecting three or more joint areas the presence of rheumatoid nodules (True) Explanation: Pathognomonic symmetrical polyarthritis (True) Explanation: Diagnosis of RA requires four or more of the criteria radiological changes (True) Explanation: In significant titres Question 12. Common extra-articular manifestations of rheumatological disorders include episcleritis and keratoconjunctivitis sicca in rheumatoid arthritis (True) erythema nodosum in enteropathic arthritis (True) enthesitis in ankylosing spondylitis (True) Explanation: And Reiter's disease alopecia in systemic lupus erythematosus (True) Explanation: Also photosensitive skin rashes retinitis pigmentosa in psoriatic arthritis (False) Question 13. Typical features of active rheumatoid arthritis include fever and weight loss (True) Explanation: These also occur with minimal joint symptoms, making diagnosis difficult macrocytic anaemia (False) Explanation: Anaemia is classically normochromic and normocytic anterior uveitis (False) Explanation: Anterior uveitis is specifically associated with the seronegative spondyloarthritides thrombocytopenia (False) Explanation: Modest elevation in platelet count is common generalised lymphadenopathy (True) Explanation: Most obvious in nodes draining actively inflamed joints Question 14. The typical pattern of synovial disease in rheumatoid arthritis includes early involvement of the sacroiliac joints (False) Explanation: More suggestive of a seronegative spondyloarthritis such as ankylosing spondylitis symmetrical peripheral joint involvement (True) Explanation: Characteristic pattern of onset spindling of the fingers and broadening of the forefeet (True) Explanation: Involvement of the proximal interphalangeal and metatarsophalangeal joints respectively distal interphalangeal joint involvement of fingers and toes (False) Explanation: More suggestive of osteoarthrosis or psoriatic arthritis atlantoaxial joint involvement (True) Explanation: Often not obvious clinically but can produce cord compression Question 15. The following statements about rheumatoid arthritis are true joint pain and stiffness are typically aggravated by rest (True) Explanation: Early morning stiffness is a characteristic feature of all inflammatory arthritides

By A. H.

MCQs VIA WEB 2005 the rheumatoid factor test is positive in about 70% of patients (True) Explanation: May be absent at disease onset and is not specific to rheumatoid arthritis joint involvement is additive rather than flitting (True) Explanation: The usual pattern; in palindromic arthritis flitting episodes are typical associated scleromalacia typically produces painful red eyes (False) Explanation: Scleromalacia is a painless wasting of the sclera unlike the rarer scleritis sicca syndrome suggests the presence of an alternative diagnosis (False) Explanation: Common in rheumatoid arthritis Question 16. The clinical features of Felty's syndrome include peak prevalence in the age group 20-30 years (False) Explanation: Peak prevalence in the age group 50-70 years previous long-standing rheumatoid arthritis (True) negative rheumatoid factor test (False) Explanation: Positive rheumatoid factor test lymphadenopathy and splenomegaly (True) Explanation: Characteristic recurrent infections and leg ulcers (True) Explanation: Characteristic Question 17. In the treatment of rheumatoid arthritis bed rest should be avoided because of bony ankylosis (False) Explanation: Bed rest is of great value and without risk of bony ankylosis splinting of the affected joints reduces pain and swelling (True) Explanation: Reduces joint pain and may reduce contractures associated anaemia responds promptly to oral iron therapy (False) Explanation: Not usually iron-deficient and reflects disease activity systemic corticosteroids are contraindicated (False) Explanation: Low-dose steroids may lessen disease progression with only a small risk of side-effects non-steroidal anti-inflammatory drugs retard disease progression (False) Explanation: Not disease-modifying drugs, unlike gold, penicillamine and immunosuppressants Question 18. Disease-modifying antirheumatic drugs (DMARD) in rheumatoid arthritis include sulfasalazine (True) Explanation: 50% of patients respond in 3-6 months naproxen (False) Explanation: None of the NSAIDs are DMARDs D-penicillamine (True) Explanation: Benefit may not be apparent for 3 months sodium aurothiomalate (True) Explanation: Adverse effects are common (e.g. proteinuria and marrow suppression) azathioprine (True) Explanation: Reserved for life-threatening or unresponsive disease Question 19. A poorer prognosis in rheumatoid arthritis is associated with insidious onset of rheumatoid arthritis (True) Explanation: An explosive onset confers a relatively better prognosis high titres of rheumatoid factor early in the course of the disease (True) Explanation: Especially within 12 months of onset early development of subcutaneous nodules and erosive arthritis (True) Explanation: Indicates seropositive disease extra-articular manifestations (True) onset with palindromic rheumatism (False) Explanation: The presence of periods of remission is a favourable sign Question 20. Typical features of seronegative spondyloarthritis include asymmetrical oligoarthritis (True)

By A. H.

MCQs VIA WEB 2005 Explanation: Axial joints are involved initially; only 10% of cases present with a peripheral arthritis involvement of cartilaginous joints (True) Explanation: E.g. the sacroiliac joints; involvement is rare in seropositive arthritides enthesitis of tendinous insertions (True) Explanation: Achilles tendonitis scleritis and episcleritis (False) Explanation: Typical ocular problem is acute anterior uveitis mitral valve disease (False) Explanation: An aortitis usually causing aortic regurgitation Question 21. Features associated with ankylosing spondylitis include peak onset in the second and third decades (True) subcutaneous nodules (False) Explanation: Nodules suggest seropositive arthritis, especially rheumatoid arthritis HLA-B27 in at least 90% of affected patients (True) Explanation: Identical twins homozygous for HLA-B27 may, however, be discordant for the disease faecal carriage of specific Klebsiella species (True) Explanation: Klebsiella carry an antigen similar to HLA-B27, suggesting a possible aetiology family history of psoriatic arthritis and Reiter's syndrome (True) Explanation: Familial aggregation of overlapping seronegative spondyloarthritides Question 22. Features suggesting ankylosing spondylitis include early morning low back pain radiating to the buttocks (True) Explanation: Due to sacroiliitis and sometimes mistaken for lumbar disc disease persistence of lumbar lordosis on spinal flexion (True) Explanation: Lumbar lordosis may be lost in advanced disease chest pain aggravated by breathing (True) Explanation: Due to involvement of the costovertebral joints 'squaring' of the lumbar vertebrae on radiograph (True) Explanation: Leading to the 'bamboo' spine appearance erosions of the symphysis pubis on radiograph (True) Explanation: Involvement of cartilaginous joints is a hallmark of the disease Question 23. In the treatment of ankylosing spondylitis systemic corticosteroid therapy is contraindicated (False) Explanation: Can be invaluable in acute iritis prolonged bed rest accelerates functional recovery (False) Explanation: In contrast to rheumatoid arthritis, the patient with ankylosing spondylitis stiffens with bed rest spinal radiotherapy modifies the course of the disease (False) Explanation: Only to improve symptoms spinal deformity is minimised with physiotherapy (True) Explanation: Education regarding appropriate back exercises is vital hip joint involvement augurs a poorer prognosis (True) Explanation: As does extra-articular disease Question 24. The typical features of reactive arthritis include the development of anterior uveitis more often than conjunctivitis (False) Explanation: Conjunctivitis is the classical ocular manifestation non-specific urethritis and prostatitis (True) Explanation: Cause dysuria, frequency and suprapubic discomfort symmetrical small joint polyarthritis (False) Explanation: Arthritis is asymmetrical, involving large or small joints onset 1-3 weeks following bacterial dysentery (True) Explanation: Similar delay following sexually acquired infections keratoderma blenorrhagica and nail dystrophy (True) Explanation: Similar to psoriatic skin and nail disease

By A. H.

MCQs VIA WEB 2005 Question 25. In Reiter's disease a peripheral blood monocytosis is commonly found (False) Explanation: Polymorphonuclear leucocytosis is typical in the acute phase sacroiliitis and spondylitis develop in most patients (False) Explanation: Occur in only 15% of patients Salmonella or Shigella species can be cultured from joint aspirates (False) Explanation: Organisms cause the preceding dysenteric illness calcaneal spurs are not apparent radiologically (False) Explanation: Appear on radiograph as a periostitis arthritis resolves within 3-6 months of onset (False) Explanation: 10% of patients have chronic active arthritis 20 years after onset Question 26. Psoriatic arthritis is usually preceded by the development of psoriasis (True) Explanation: Occasionally there is no evidence of skin disease at onset likely to develop in 25% of patients with psoriasis (False) Explanation: Occurs in around 7% of patients commoner in patients with psoriatic nail changes (True) Explanation: Such as pitting and onycholysis associated with a poorer prognosis than rheumatoid arthritis (False) Explanation: Except for patients with arthritis mutilans likely to respond to hydroxychloroquine (False) Explanation: Should be avoided due to precipitation of an exfoliative dermatitis Question 27. Recognised patterns of psoriatic arthritis include asymmetrical oligoarthritis of the fingers and toes (True) Explanation: Occurs in 40% of patients distal interphalangeal joint involvement with nail dystrophy (True) Explanation: Occurs in 15% of patients sacroiliitis and spondylitis (True) Explanation: Develops in 15% of patients-may be indistinguishable from ankylosing spondylitis rheumatoid-like symmetrical small joint arthritis (True) Explanation: Occurs in 25% of patients arthritis mutilans with telescoping of the digits (True) Explanation: Occurs in 5% of patients Question 28. Diseases associated with seronegative spondyloarthritis include Sjögren's syndrome (False) Explanation: Either as a primary disorder or in association with some connective tissue diseases Whipple's disease (True) Explanation: Rare condition coeliac disease (False) Explanation: An association between coeliac disease and HLA-B8, DR17 and OQ2 but not HLA-B27 ulcerative colitis (True) Explanation: Arthritis may precede evidence of ulcerative colitis or Crohn's disease Behçet's disease (True) Explanation: Suggested by orogenital ulceration and iritis (more common in Japan) Question 29. Factors predisposing to hyperuricaemia and gout include hypothyroidism (True) Explanation: Diminished renal excretion of uric acid severe exfoliative psoriasis (True) Explanation: Increased purine turnover chronic renal failure (True) Explanation: Diminished renal excretion of uric acid polycythaemia rubra vera (True)

By A. H.

MCQs VIA WEB 2005 Explanation: Increased purine turnover therapy with loop diuretic agents (True) Explanation: Diminished renal excretion of uric acid Question 30. The clinical features of gout include precipitation of an acute attack by allopurinol (True) Explanation: Enzyme induction induces an acute attack cellulitis, tenosynovitis and bursitis (True) Explanation: Non-articular signs may predominate the abrupt onset of severe joint pain and tenderness (True) Explanation: Onset may be explosively sudden serum urate levels fall during an acute attack (False) Explanation: Serum urate is usually elevated but may be normal loin pain and haematuria (True) Explanation: Urate urolithiasis Question 31. In the treatment of gout NSAID therapy increases urinary urate excretion (False) Explanation: Uricosuric drugs include probenecid, sulfinpyrazone and the NSAID azapropazone salicylates control symptoms and accelerate resolution of the acute attack (False) Explanation: Aspirin may worsen an acute attack by reducing renal urate excretion allopurinol inhibits xanthine oxidase and hence urate production (True) tophi should resolve with control of hyperuricaemia (True) allopurinol or probenecid should be given within 24 hours of onset of the acute attack (False) Explanation: Delay hypouricaemic therapy unless concomitant colchicine therapy is given Question 32. In pyrophosphate arthropathy calcium pyrophosphate dihydrate crystals are deposited in the synovial cells (False) Explanation: Crystals are deposited in articular cartilage then shed into the joint space haemochromatosis is a recognised predisposing factor (True) the clinical appearances are similar to acute gout (True) Explanation: Hence 'pseudogout' the findings on synovial aspiration are indistinguishable from acute gout (False) Explanation: Characteristic appearances of calcium pyrophosphate dihydrate (CPPD) crystals under polarising light microscopy intra-articular corticosteroid injections are contraindicated (False) Explanation: Such injections are often highly effective Question 33. Osteoporosis is usually associated with normal serum calcium, phosphate and alkaline phosphatase (True) Explanation: Serum alkaline phosphatase may rise if fractures occur more likely to occur if menopause is early (True) Explanation: Accelerated bone loss occurs with oestrogen withdrawal commonly asymptomatic (True) Explanation: Pain only occurs after fracture a typical complication of untreated Addison's disease (False) Explanation: Occurs in states of corticosteroid excess more common in patients with chronic high alcohol intake (True) Explanation: Also associated with cigarette smoking Question 34. Risk factors for osteoporosis include gluten enteropathy (True) Explanation: All causes of malabsorption including liver disease rheumatoid arthritis (True) Explanation: And ankylosing spondylitis hyperparathyroidism (True) Explanation: Multifactorial

By A. H.

MCQs VIA WEB 2005 anorexia nervosa (True) Explanation: Multifactorial hypogonadism (True) Explanation: Improved by androgen replacement therapy Question 35. Therapies useful in preventing recurrent vertebral fractures in osteoporosis include regular exercise (True) Explanation: Excessive exercise may be associated with low body weight and osteoporosis oral phosphate supplementation (False) Explanation: Unless the patient is hypophosphataemic from severe malnutrition etidronate (True) Explanation: Bisphosphonate therapy is the most effective and best evaluated vitamin D and calcium supplementation (True) Explanation: But this is less effective than bisphosphonate therapy corticosteroid (False) Explanation: Causes osteoporosis; androgen and oestrogen therapy are both effective Question 36. In osteomalacia the finding of a proximal myopathy suggests an alternative diagnosis (False) Explanation: Characteristic; patients may have difficulty in standing up or in climbing stairs bone involvement is characteristically painless (False) Explanation: Pain may be generalised and severe Chvostek's sign indicates that the underlying diagnosis may be hyperparathyroidism (False) Explanation: Hypocalcaemia increases neuromuscular excitability (latent tetany) due to renal disease, 25-hydroxycholecalciferol therapy is recommended (False) Explanation: Give 1-á-hydroxycholecalciferol; renal 1-á-hydroxylation is impaired pseudofractures on radiograph are pathognomonic (True) Explanation: Looser's zones are translucent bands seen on radiograph Question 37. Typical features of Paget's disease of bone include onset before the age of 40 years (False) Explanation: Onset usually over the age of 60 years increased serum alkaline phosphatase and urinary hydroxyproline excretion (True) Explanation: Increased bone turnover and osteoblast activity presentation with severe bone pain, especially in elderly patients (False) Explanation: Insidious asymptomatic progression; with nerve root and spinal cord compression delayed healing of fractures (False) Explanation: Fractures occur more commonly but usually heal normally risk of development of osteogenic sarcoma (True) Explanation: Rare complication suggested by bony expansion and localised pain Question 38. In a male patient with prostate cancer and widespread metastatic bone disease osteolytic deposits are characteristic (False) Explanation: Prostatic secondaries are typically osteosclerotic the plasma parathyroid hormone (PTH) concentration is typically elevated (False) Explanation: Serum PTH is usually normal even when the serum calcium is high bone pain is invariably present (False) Explanation: Asymptomatic disease may be detected coincidentally on radiograph the alkaline phosphatase is only elevated if pathological fracture occurs (False) Explanation: Serum alkaline phosphatase is frequently elevated due to osteoblast activation cyproterone acetate retards progress of the disease (True) Explanation: Androgen deprivation therapy is of proven value in prostatic cancer Question 39. Typical features of systemic lupus erythematosus (SLE) include a higher prevalence in Caucasian than in African women (False) Explanation: Afro-Caribbean females are particularly susceptible onset usually in the fourth and fifth decades (False)

By A. H.

MCQs VIA WEB 2005 Explanation: Most commonly in the second and third decades impaired function of suppressor T lymphocytes (True) Explanation: Associated with polyclonal B lymphocyte activation increased prevalence in women compared with men (True) Explanation: Genetic factors appear to be of importance in aetiology presentation with Raynaud's phenomenon in young men rather than young women (True) Explanation: And in women aged > 30 years Question 40. Characteristic clinical features of SLE include Raynaud's phenomenon (True) Explanation: Not, however, specific to SLE alopecia (True) Explanation: Occurs in at least 50% of patients an erythematous photosensitive facial rash (True) Explanation: Characteristic absence of renal complications (False) Explanation: Renal involvement is not infrequent and heralds a poor prognosis neuropsychiatric symptoms (True) Explanation: Especially depression and organic psychosis Question 41. In the management of systemic lupus erythematosus, the following are of proven value NSAIDs for renal involvement (False) Explanation: NSAIDs may worsen renal function corticosteroid therapy for cerebral involvement (True) Explanation: High doses are often used initially, then reduced to as low a dose as possible on remission of disease plasmapheresis for immune complex disease (True) Explanation: Especially when combined with immunosuppressant drugs hydroxychloroquine for skin and joint involvement (True) Explanation: Beware retinal complications long-term corticosteroid therapy during periods of remission to prevent relapse (False) Explanation: Little evidence to suggest that this improves the long-term prognosis Question 42. Recognised features of primary Sjögren's syndrome include an increased incidence of lymphoma (True) dryness of the eyes, mouth and vagina (True) reduced lacrimal secretion rate (True) Explanation: Demonstrable with the Shirmer test more males affected than females (False) Explanation: More females than males a positive IgM rheumatoid factor in over 80% of patients (True) Explanation: Not diagnostic of primary Sjögren's (sicca) syndrome Question 43. The clinical features of progressive systemic sclerosis include presentation with Raynaud's phenomenon (True) Explanation: Raynaud's may precede other features by years reflux oesophagitis and dysphagia (True) Explanation: Gastrointestinal tract is involved in most patients fibrosing alveolitis (True) Explanation: Occurs in the majority of cases ulceration, atrophy and subcutaneous calcification of the fingertips (True) Explanation: 'Sausaging' of the fingers and sclerodactyly are also seen anti-DNA antibodies and decreased serum complement levels (False) Explanation: ANA only in 50%; anti-DNA antibodies are not seen and complement is normal Question 44. In polymyositis a normal serum creatine kinase does not exclude the diagnosis (True) Explanation: Especially common in juvenile myositis

By A. H.

MCQs VIA WEB 2005 antinuclear (DNA) antibodies are characteristically absent (True) Explanation: Similarly in polyarteritis nodosa electromyography is helpful in differentiation from peripheral neuropathy (True) underlying malignancy is usually present if weight loss is marked (False) Explanation: Weight loss may occur in the absence of malignancy an erythematous rash on the knuckles, elbows, knees and face is typical (True) Explanation: Cutaneous features suggest dermatomyositis (Gottron's papules) Question 45. Features of giant cell arteritis include a predominance in females > 60 years of age (True) pain in the jaw during eating (True) Explanation: Due to claudication of the masseters confluent involvement of affected arteries (False) Explanation: Histological involvement is characteristically patchy difficulty in rising from the seated position (False) Explanation: Suggests proximal myopathy weight loss with normochromic anaemia and high ESR (True) Question 46. In polymyalgia rheumatica antinuclear and rheumatoid factor antibodies are present in high titre (False) Explanation: This finding would suggest an alternative diagnosis temporal artery biopsy usually confirms the diagnosis (False) Explanation: Biopsy is positive in < 40% of patients response to oral corticosteroids typically occurs within 7 days (True) Explanation: No such response should prompt a review of the diagnosis corticosteroid therapy should be Question 47. The features of classical polyarteritis nodosa include increased prevalence in males (True) Explanation: Male to female ratio is 2:1 an association with circulating immune complexes containing hepatitis B virus (True) Explanation: HBV markers may only become apparent on follow-up involvement of small arteries and arterioles (False) Explanation: Systemic vasculitis affecting medium-sized arteries multiple peripheral nerve palsies (True) Explanation: Due to arteritis of the vasa nervorum severe hypertension (True) Explanation: Especially in association with renal involvement Module 21 (Chapter 21) Question 1. The following statements about the skin are true the surface area of an adult is approximately 2 m2 (True) Explanation: Comprising the epidermis, dermis and subcutis layers the predominant cell of the dermis is the fibroblast (True) keratinocytes comprise 10% of the epidermal cell mass (False) Explanation: They comprise 95% of epidermal cells Langerhans cells synthesise vitamin D in the epidermis (False) Explanation: These are modified macrophages; keratinocytes synthesise vitamin D eccrine sweat glands are innervated by the parasympathetic nervous system (False) Explanation: They are innervated by cholinergic fibres of the sympathetic system

Question 2. In the terminology of skin lesions papules are solid skin elevations > 20 mm in diameter (False) Explanation: Papules < 5 mm in diameter nodules are solid skin masses > 5 mm in diameter (True) Explanation: Larger than papules vesicles are fluid-containing skin elevations > 5 mm in diameter (False)

By A. H.

MCQs VIA WEB 2005 Explanation: Vesicles < 5 mm in diameter petechiae are pinhead-sized macules of blood within the skin (True) Explanation: They are not palpable macules are small raised areas of skin of altered colour (False) Explanation: Macules are flat, with altered skin colour or texture Question 3. Typical features of melanocytic naevi include the following usually present from birth (False) Explanation: Most appear in early childhood development after the age of 40 years (False) Explanation: Should raise suspicion of malignancy junctional naevi are smooth, papillomatous, hairy nodules (False) Explanation: Not hairy and are macular intradermal naevi are circular brown macules < 10 mm in diameter (False) Explanation: They are nodular 30% life-time risk of malignant transformation (False) Explanation: 6% in congenital melanocytic naevi Question 4. Typical features of malignant melanoma include changing appearance of a preceding melanocytic naevus (True) Explanation: 30-50% develop in this way diameter of the lesion > 5 mm (True) Explanation: But smaller lesions may be malignant irregular colour, border and elevation (True) Explanation: Typically asymmetrical personal or family history of melanoma (True) Explanation: Risk is also increased with fair skin and blonde hair painless, expanding, subungual area of pigmentation (True) Explanation: Characteristically painless Question 5. Characteristic features of eczema include epidermal oedema and intra-epidermal vesicles (True) Explanation: Epidermal oedema (spongiosis) and epidermal thickening (acanthosis) delayed hypersensitivity reaction in seborrhoeic eczema (False) Explanation: This is a feature of allergic contact eczema increased serum IgA concentration in discoid eczema (False) Explanation: Serum IgE concentrations are elevated eyelid and scrotal oedema in allergic contact eczema (True) Explanation: The initial eruption occurs at the contact site occurrence in the flexures of the elbows and knees in pompholyx (False) Explanation: Occurs on palms and plantar surfaces of hands and feet Question 6. The following blistering eruptions are typically associated with mucosal involvement dermatitis herpetiformis (False) Explanation: An intensely itchy rash without oral mucosal involvement bullous pemphigoid (False) pemphigus (True) Explanation: Often erosive and with mucosal involvement toxic epidermal necrolysis (True) porphyria cutanea tarda (False) Question 7. The following are recognised causes of leg ulcers leprosy (True) Explanation: Typically painless sickle-cell disease (True) Explanation: And also cryoglobulinaemia diabetes mellitus (True)

By A. H.

MCQs VIA WEB 2005 Explanation: Arterial and neuropathic aetiology pyoderma gangrenosum (True) Explanation: Associated with inflammatory bowel disease syphilis (True) Question 8. The following cause alopecia with scarring tinea capitis (False) alopecia areata (False) discoid lupus erythematosus (True) Explanation: Typically patchy telogen effluvium (False) androgenetic alopecia (False) Explanation: Male-pattern baldness Question 9. With regard to psoriasis a child with one affected parent has a 50% chance of developing the disease (False) Explanation: 15% if there is one affected parent the cellular infiltrate is typically lymphocytic (True) Explanation: Of helper type in the dermis guttate psoriasis may be preceded by â-haemolytic streptococcal infection (True) Explanation: Typically throat infection nail pitting is associated with distal interphalangeal arthropathy (True) Explanation: And onycholysis about 5% of patients develop arthropathy (True) Question 10. Typical features of psoriasis include well-defined erythematous plaques with adherent silvery scales (True) Explanation: Typically on the elbows, knees and lower back epidermal thickening and nucleated horny layer cells (parakeratosis) (True) Explanation: Also a dermal T lymphocyte infiltrate induction of plaques by local trauma (True) Explanation: Including surgical wounds (Köbner phenomenon) an association with HLA Cw6 (True) Explanation: Inheritance is probably polygenic exacerbation by propranolol and lithium carbonate therapy (True) Explanation: Also antimalarial drugs Question 11. The characteristic clinical features of psoriasis include sparing of the skin over the head, face and neck (False) Explanation: The scalp is frequently involved guttate psoriasis usually affects the elderly (False) Explanation: Usually seen in children nail changes with pitting and onycholysis (True) Explanation: Also subungual hyperkeratosis oligoarthritis particularly associated with nail changes occurring in 5% of cases (True) Explanation: Perhaps mimicking rheumatoid arthritis red non-scaly skin areas in the natal cleft and submammary folds (True) Explanation: Axillary folds may be similarly affected Question 12. The typical features of acne vulgaris include involvement of pilosebaceous glands and their ducts (True) Explanation: Ducts may be obstructed distribution over the face and upper torso (True) Explanation: Lesions elsewhere suggest an alternative diagnosis infection with the skin commensal Propionibacterium acnes (True) Explanation: Antibiotics are helpful increased sebum production containing excess free fatty acids (True)

By A. H.

MCQs VIA WEB 2005 Explanation: Largely hormonally mediated open and closed comedones, inflammatory papules, nodules and cysts (True) Explanation: Seborrhoea (greasy skin) is often present also Question 13. Therapies of proven value in acne vulgaris include oral tetracycline or erythromycin therapy (True) Explanation: For a minimum of 3 months topical preparations of benzoyl peroxide and retinoic acid (True) Explanation: Antibacterials such as chlorhexidine may also help oral contraceptive pill (False) Explanation: Unless given with cyproterone acetate cyproterone acetate (True) Explanation: Anti-androgen therapy often in combination with an oestrogen oral isotretinoin (True) Explanation: Reduces sebum secretion; highly teratogenic Question 14. The characteristic features of rosacea include predominantly affects adolescents (False) Explanation: Commonest in middle age increased secretion of sebum (False) Explanation: Sebum secretion is normal facial erythema, telangiectasia, pustules and papules (True) rhinophyma, conjunctivitis and keratitis (True) non-responsive to oral tetracycline therapy (False) Explanation: Repeated courses may be necessary Question 15. Medical conditions that cause pruritus include oral contraceptives and pregnancy (True) hypothyroidism and hyperthyroidism (True) Explanation: Also caused by biliary obstruction lymphoproliferative and myeloproliferative diseases (True) iron deficiency anaemia (True) Explanation: Also caused by chronic renal failure opiate and antidepressant drug therapy (True) Question 16. Skin diseases associated with marked pruritus include cutaneous vasculitis (False) Explanation: The rash is non-pruritic lichen planus (True) Explanation: Usually intensely itchy atopic eczema (True) Explanation: Classically pruritic seborrhoeic keratosis (False) Explanation: Non-pruritic dermatitis herpetiformis (True) Explanation: Associated with coeliac disease Question 17. Skin diseases associated with blistering eruptions include erythema multiforme (True) Explanation: Perhaps with target lesions dermatitis herpetiformis (True) Explanation: Typically on extensor surfaces pemphigoid (True) Explanation: Tense blood-filled lesions pemphigus vulgaris (True) Explanation: Superficial flaccid lesions guttate psoriasis (False)

By A. H.

MCQs VIA WEB 2005 Explanation: Small scaly raised lesions Question 18. Skin diseases associated with photosensitivity include variegate and hepatic porphyrias (True) Explanation: Disordered haem metabolism atopic eczema (True) Explanation: Perhaps progressing to chronic actinic dermatitis drug reactions to phenothiazine, thiazide and tetracycline (True) Explanation: And also to amiodarone and enalapril therapy pyoderma gangrenosum (False) Explanation: Associated with inflammatory bowel disease pityriasis rosea (False) Explanation: Unaffected by sunlight Question 19. Recognised causes of erythema multiforme include herpes simplex infection (True) Explanation: Also orf and other viruses mycoplasmal pneumonia (True) Explanation: Classical sulphonamide therapy (True) Explanation: Also penicillins and barbiturates systemic lupus erythematosus (True) Explanation: And other connective tissue disorders bronchogenic carcinoma (True) Explanation: And especially post-radiotherapy Question 20. Recognised causes of erythema nodosum include sarcoidosis (True) Explanation: Also brucellosis â-haemolytic streptococcal infection (True) Explanation: Also mycoplasmal and chlamydial infections inflammatory bowel disease (True) Explanation: Also leukaemias and Hodgkin's disease tuberculosis (True) Explanation: Also leprosy contraceptive drug therapy (True) Explanation: Erythema nodosum can also be caused by some other drugs, e.g. iodides and sulphonamides Question 21. The typical features of basal cell carcinoma include the following predominantly affects the elderly (True) Explanation: Rare in young adults metastatic spread to the lungs if untreated (False) Explanation: Spread by local invasion occurrence in areas exposed to light or X-irradiation (True) Explanation: Typically on the face or head papule with surface telangiectasia or ulcerated nodule (True) Explanation: With a rolled, pearly edge unresponsive to radiotherapy (False) Explanation: Radiosensitive but surgery is preferred Question 22. The typical features of squamous cell carcinoma include occurrence in areas exposed to light or X-irradiation (True) Explanation: Typically in Caucasians living in equatorial regions association with chronic immunosuppressant drug therapy (True) Explanation: E.g. following organ transplantation preceded by leucoplakia on the lips, mouth or genitalia (True) Explanation: Or actinic keratosis on the skin

By A. H.

MCQs VIA WEB 2005 metastatic spread to the liver and lungs (False) Explanation: Haematogenous dissemination is rare unresponsive to radiotherapy (False) Explanation: Radiosensitive but surgery is preferred Module 22 (Chapter 22) Question 1. The predominant segmental innervation of the following tendon reflexes is biceps jerk-C5 (True) supinator jerk-C6 (True) Explanation: Same as the biceps jerk triceps jerk-C7 (True) Explanation: Finger flexion jerk-C8 knee jerk-L4 (True) ankle jerk-S1 (True)

Question 2. A right homonymous hemianopia would be an expected finding in disorders of the left optic tract (True) Explanation: The optic tract runs between optic chiasm and lateral geniculate body left optic radiation (True) Explanation: Upper fibre damage causes lower field defect optic chiasm (False) Explanation: Midline lesions cause bitemporal hemianopia right lateral geniculate body (False) Explanation: Left lateral geniculate body left optic nerve (False) Explanation: Left monocular visual loss Question 3. Features suggesting a 3rd cranial nerve palsy include paralysis of abduction (False) Explanation: Suggests 6th cranial nerve palsy absence of facial sweating (False) Explanation: Occurs in Horner's syndrome complete ptosis (True) Explanation: Paralysis of levator palpebrae superioris pupillary dilatation (True) Explanation: Impaired parasympathetic flow absence of the accommodation reflex (True) Explanation: And direct light response impaired Question 4. Paralysis of the 4th cranial nerve produces weakness of the inferior oblique muscle (False) Explanation: Superior oblique pupillary dilatation (False) Explanation: No pupillary change impaired downward gaze in adduction (True) Explanation: May be difficult to detect clinically elevation and abduction of the eye (True) Explanation: Head may tilt towards normal side nystagmus more marked in the abducted eye (False) Explanation: Suggests internuclear ophthalmoplegia Question 5. Paralysis of the 6th cranial nerve produces impaired adduction of the eye (False) Explanation: Impaired abduction produces enophthalmos (False) Explanation: May be a feature of Horner's syndrome is a characteristic feature of Wernicke's encephalopathy (True)

By A. H.

MCQs VIA WEB 2005 Explanation: Usually bilateral, perhaps other ocular nerves also involved results from disease of the upper pons (True) Explanation: Infarction, haemorrhage or demyelination typically is a recognised feature of posterior fossa tumour (True) Explanation: May be 'false localising sign' in raised intracranial pressure Question 6. Drooping of the upper eyelid results from a lesion of the levator palpebrae superioris (True) Explanation: Partial or complete ptosis 3rd cranial nerve (True) Explanation: With pupillary dilatation cervical sympathetic outflow (True) Explanation: With pupillary constriction 7th cranial nerve (False) Explanation: Orbicularis oculi may be affected abducens nucleus (False) Explanation: No ptosis, just a lateral rectus palsy Question 7. Absence of pupillary constriction in either eye on shining a light into the right pupil suggests bilateral Argyll Robertson pupils (True) Explanation: Accommodation preserved bilateral Holmes-Adie pupils (True) Explanation: Defect is probably in the ciliary ganglia right optic nerve lesion (True) Explanation: An afferent defect right oculomotor nerve lesion (False) Explanation: Reaction in right eye only is impaired bilateral Horner's syndrome (True) Explanation: Both pupils may be small but response preserved Question 8. Features of an intracranial lower motor neuron lesion of the facial nerve include inability to wrinkle the forehead (True) Explanation: Frontalis weakness increased lacrimation on the affected side (False) Explanation: Decreased due to involvement of nervus intermedius upward deviation of the eye on attempted eyelid closure (True) Explanation: Bell's sign deafness due to loss of the nerve to the stapedius muscle (False) Explanation: Produces hyperacusis loss of taste over the anterior two-thirds of the tongue (True) Explanation: Involvement of the chorda tympani Question 9. Characteristic features of pseudobulbar palsy include dysarthria (True) Explanation: With dysphonia dysphagia (True) Explanation: Often with aspiration emotional lability (True) Explanation: Particularly in cerebrovascular disease wasting and fasciculation of the tongue (False) Explanation: Suggest lower motor neuron lesion, 12th nerve absence of the jaw jerk (False) Explanation: Jaw jerk is typically brisk Question 10. The following statements about bladder innervation are correct sacral cord lesions usually produce urinary retention (True) Explanation: Parasympathetic innervation impaired

By A. H.

MCQs VIA WEB 2005 thoracic cord lesions produce urinary urge incontinence (True) Explanation: And incomplete bladder emptying pelvic nerve parasympathetic stimulation causes bladder emptying (True) Explanation: Internal sphincter relaxation and detrusor contraction pudendal nerve lesions produce automatic bladder emptying (False) Explanation: Feature of spinal cord lesions the L1-L2 segment sympathetic outflow mediates bladder relaxation (True) Explanation: And internal sphincter contraction Question 11. The following statements about the Glasgow coma scale are correct the best response to an arousal stimulus should be measured (True) Explanation: Test at least twice appropriate motor responses to verbal commands = score 6 (True) Explanation: No response to pain = 1 spontaneous eye opening = score 4 (True) Explanation: No eye opening = 1 verbal responses with normal speech and orientation = score 5 (True) Explanation: No speech = 1 the minimum total score = 3 (True) Explanation: Maximum score = 15 Question 12. The diagnosis of brain death is supported by pin-point pupils (False) Explanation: Dilated and unreactive to light absent corneal reflexes (True) Explanation: A brain-stem reflex absent vestibulo-ocular responses to caloric testing (True) Explanation: 20 ml ice-cold water into each ear in turn absence of spontaneous respiration (True) Explanation: With PaCO2 > 6.7 kPa preservation of the cough and gag reflexes (False) Explanation: All brain-stem reflexes absent Question 13. Typical features of prefrontal lobe lesions include positive grasp reflex (True) Explanation: And other 'primitive' reflexes astereognosis (False) Explanation: Suggests a parietal lobe lesion sensory dysphasia (False) Explanation: Posterior temporo-parietal lesion (Wernicke's area) olfactory hallucinations (False) Explanation: Temporal lobe sign social disinhibition (True) Explanation: Perhaps with antisocial behaviour Question 14. Typical features of posterior parietal lobe lesions include lower homonymous quadrantanopia (False) Explanation: Contralateral to lesion constructional apraxia (False) Explanation: Non-dominant hemisphere sensory inattention (False) Explanation: Perhaps with sensory neglect motor dysphasia (True) Explanation: Broca's area in the inferior frontal lobe agnosia and acalculia (False) Explanation: Gerstmann's syndrome of the dominant angular gyral region

By A. H.

MCQs VIA WEB 2005 Question 15. In the evaluation of a patient with headache thunderclap headache is invariably associated with subarachnoid haemorrhage (False) Explanation: Only associated in 1 in 8 patients patients with viral meningitis invariably display meningism (False) Explanation: Meningism less common than in bacterial infection the presence of concurrent focal limb weakness excludes migraine (False) Explanation: Migrainous hemiparesis is well recognised improvement with simple analgesia suggests tension headache (False) Explanation: Tension headaches are typically poorly responsive headache on waking suggests raised intracranial pressure (True) Explanation: As does morning vomiting Question 16. Migrainous neuralgia (cluster headache) is more common in females than in males (False) Explanation: Male to female ratio is 5:1 the commonest form of migraine (False) Explanation: 10-50 times less common associated with Horner's syndrome in some patients (True) Explanation: And unilateral lacrimation likely to be cured by prophylactic propranolol treatment (False) Explanation: Prophylaxis may not be helpful likely to respond well to sumatriptan therapy (True) Question 17. In the evaluation of a patient with true vertigo short-lived symptoms favour a labyrinthine cause (True) Explanation: Persistent vertigo is more often central the presence of nystagmus excludes viral labyrinthitis (False) Explanation: Often present although transient associated paroxysmal tinnitus suggests Ménière's disease (True) Explanation: Exclude acoustic neuroma positional vertigo fatigues rapidly when due to central cause (False) Explanation: Tends to persist temporal lobe epilepsy should be considered (True) Explanation: But a rare cause Question 18. Features suggesting vasovagal faint rather than epilepsy in a patient with a blackout include an olfactory aura (False) Explanation: But many patients are aware that something is about to happen confusion following the event (False) headache following the event (False) Explanation: Also absence of injury or tongue-biting memory loss surrounding the event (False) tongue-biting (False) Explanation: Also pallor rather than central cyanosis suggests fainting Question 19. In the analysis of gait circumduction of a leg suggests pyramidal weakness (True) Explanation: Often with dragging of the affected foot a high-stepping gait suggests foot drop (True) Explanation: Perhaps with slapping steps inability to walk heel-to-toe suggests cerebellar disease (True) Explanation: Classically of the vermis difficulty negotiating doorways suggests parkinsonism (True) Explanation: Associated with festination a waddling gait suggests proximal muscle weakness (True) Explanation: Usually myopathic in nature

By A. H.

MCQs VIA WEB 2005 Question 20. Jerking nystagmus that changes in direction with the direction of gaze is compatible with cerebellar hemisphere disease (True) Explanation: Maximal on gaze towards lesion if cerebellar disease is unilateral indicative of a brain-stem disorder (True) Explanation: May be more marked in the abducting eye (ataxic nystagmus) compatible with a vestibular nerve lesion (False) Explanation: Typically present only when looking away from side of lesion typically accompanied by vertigo and tinnitus (False) Explanation: Suggests vestibulocochlear disease likely to continue following closure of the eyes (True) Explanation: Demonstrable using electronystagmography Question 21. The characteristic features of trigeminal neuralgia include pain lasting several hours at a time (False) Explanation: Lancinating paroxysms lasting a few seconds pain precipitated by touching the face and/or chewing (True) Explanation: 'Trigger areas' may exist absence of the corneal reflex (False) Explanation: No abnormal signs predominance in young females (False) Explanation: Occurs in elderly subjects response to anticonvulsants (True) Explanation: E.g. carbamazepine Question 22. The typical features of Ménière's disease include sudden onset of vertigo, nausea and vomiting (True) Explanation: May be disabling progressive sensorineural deafness and tinnitus (True) Explanation: Usually unilateral jerking nystagmus and ataxic gait (True) Explanation: Typically during attacks nystagmus usually persists between attacks (False) Explanation: Suggests benign positional vertigo restoration of hearing following effective treatment (False) Explanation: May delay progression but cannot restore auditory loss Question 23. Typical causes of vertigo include cardiac arrhythmia (False) Explanation: Postural instability and syncopal symptoms acoustic neuroma (True) Explanation: Or other pathology of the 8th nerve vestibular neuronitis (True) Explanation: Usually associated with vertebral artery ischaemia gentamicin drug therapy (True) Explanation: And other ototoxic drugs otitis media (True) Explanation: With secondary labyrinthine inflammation Question 24. Typical features of generalised epilepsy include loss of consciousness accompanied by symmetrical EEG discharge (True) Explanation: May follow focal EEG abnormality and symptoms-partial seizures invariable presence of an aura (False) Explanation: Often absent lesion demonstrable on CT of the brain (False) Explanation: Usually no obvious abnormality induction by photic stimulation (True) Explanation: TV or computer games may induce fits

By A. H.

MCQs VIA WEB 2005 induction by hyperventilation (True) Explanation: Often used during the recording of an EEG Question 25. The clinical features of tonic clonic seizures include prodromal phase lasting hours or days (True) Explanation: With vague irritability or lethargy onset with an audible cry due to the aura (False) Explanation: Audible cry may occur at the onset of the tonic phase sustained spasm of all muscles lasting 30 seconds (True) Explanation: Tonic phase interrupted jerking movements lasting 1-5 minutes (True) Explanation: Clonic phase flaccid post-ictal state with bilateral extensor plantars (True) Explanation: Variable duration Question 26. The typical features of absence (petit mal) seizures include loss of consciousness lasting up to 10 seconds (True) Explanation: Sometimes with loss of posture onset around age 25-30 years (False) Explanation: Typically in childhood synchronous three per second spike and wave activity on EEG (True) Explanation: May be detected inter-ictally later development of tonic clonic seizures in 40% of patients (True) Explanation: May not occur until adulthood sleepiness lasting several hours post-ictally (False) Explanation: Rapid recovery although may occur very frequently Question 27. The following statements about epilepsy are correct treatment should be started following a single witnessed seizure (False) Explanation: Await evidence of recurrent seizures 25% of patients will have a further seizure within 1 year of a first seizure (False) Explanation: 70%, mostly in first 2 months trigger factors for epilepsy include sleep deprivation and physical exhaustion (True) Explanation: Also febrile illnesses and metabolic disturbances the lifetime risk of a single seizure is 20% (False) Explanation: 5% sharp waves on EEG are highly specific for epilepsy (True) Explanation: Only one in 1000 are false positives Question 28. A patient with seizures in the UK can drive a private car following a single seizure after 1 year free of recurrence (True) hold a heavy goods vehicle licence if all seizures occurred before the age of 5 years (True) Explanation: Providing no potentially epileptogenic brain lesion identified drive a private car during the withdrawal of anticonvulsant therapy (False) Explanation: Should stop driving for 6 months after their withdrawal drive a heavy goods vehicle only if seizure-free for 5 years (False) Explanation: 10 years drive a private car if seizures have only occurred during sleep in the previous 3 years (True) Question 29. The following statements about anticonvulsants are correct plasma level monitoring is particularly useful in sodium valproate therapy (False) Explanation: Phenytoin and carbamazepine primidone is likely to cause sideroblastic anaemia (False) Explanation: Megaloblastic anaemia clonazepam is the first-line treatment of absence seizures (False) Explanation: Ethosuximide sodium valproate is the first-line treatment in primary generalised tonic clonic seizures (True)

By A. H.

MCQs VIA WEB 2005 carbamazepine is a recognised cause of hyponatraemia (True) Explanation: Particularly in older patients Question 30. Features suggesting epilepsy rather than a simple faint as the cause of blackouts include impairment of vision heralding the attack (False) Explanation: Suggests syncopal episode tongue-biting during the attack (True) Explanation: Not specific, especially in the elderly eye-witness account of sustained jerking movements during the attack (True) Explanation: Some jerking movements are common in simple faints attacks aborted by lying supine (False) Explanation: Suggests vasovagal syncope attacks confined to the sleeping hours (True) Explanation: May occur in blackouts due to bradycardias Question 31. Clinical features of raised intracranial pressure include tachycardia and hypotension (False) Explanation: Bradycardia and hypertension dizziness and lightheadedness (True) Explanation: And vomiting headache aggravated by bending and straining (True) Explanation: And coughing behavioural and personality changes (True) Explanation: And impairment of conscious level 6th or 3rd cranial nerve palsies (True) Explanation: 'False localising signs' Question 32. The following statements about primary brain tumours are correct meningiomas are the most common type in the middle-aged (True) Explanation: 20% of all cerebral tumours gliomas are the most common type in childhood (False) Explanation: 40% of all cerebral tumours most childhood brain tumours arise within the posterior fossa (True) Explanation: They are usually cerebellar tumours presentation with adult-onset partial seizures is typical (True) Explanation: Indication for CT acoustic neuromas usually present in the 6th and 7th decades (False) Explanation: Fourth and fifth decades Question 33. Typical causes of transient cerebral ischaemic attacks include carotid artery stenosis (True) Explanation: Usually contralateral motor, sensory, speech disturbance atrial fibrillation (True) Explanation: Bilateral events may occur hypotension (True) Explanation: Associated with standing intracerebellar haemorrhage (False) Explanation: Fixed deficit stroke intracerebral tumour (False) Explanation: Slowly progressive typically Question 34. Clinical features suggesting lacunar stroke include homonymous hemianopia (False) Explanation: The optic pathway is only affected by larger lesions motor or sensory dysphasia (False) Explanation: Suggests cortical damage facial weakness and arm monoparesis (True)

By A. H.

MCQs VIA WEB 2005 Explanation: Internal capsule lacuna isolated hemiparesis or hemianaesthesia (True) Explanation: Internal capsule lacuna history of hypertension or diabetes mellitus (True) Explanation: Account for > 80% of lacunar strokes Question 35. The following statements about stroke are correct 65% of completed strokes are due to cerebral infarction (False) Explanation: 85% most strokes are complete in < 6 hours (True) Explanation: Minority 'stutter' over a longer period 20% of cerebral infarcts are secondary to cardiogenic embolism (True) Explanation: Another 20% are lacunar infarcts following an ischaemic stroke, aspirin reduces the risk of death or further stroke by 25% (True) Explanation: 75-150 mg daily 20% of patients with carotid territory symptoms have a major (> 70%) stenosis (True) Explanation: Carotid endarterectomy may then be beneficial Question 36. Clinical features suggesting intracerebral haemorrhage include abrupt onset of severe headache followed by coma (True) Explanation: Headache is not specific to haemorrhage 3rd cranial nerve palsy (True) Explanation: In midbrain haemorrhage retinal haemorrhages and/or papilloedema (True) Explanation: With subhyaloid retinal haemorrhage vomiting and neck stiffness (True) Explanation: Raised ICP tinnitus, deafness and vertigo (False) Explanation: Suggest peripheral 8th nerve lesion Question 37. Intracerebral abscess is a typical complication of infective endocarditis (True) Explanation: Often streptococcal in origin bronchiectasis (True) Explanation: Usually staphylococcal in origin frontal sinusitis (True) Explanation: Typically affects the frontal lobe otitis media (True) Explanation: Cerebellar or temporal head injury (True) Explanation: Typically staphylococcal in origin Question 38. The typical features of an intracerebral abscess include high fever, weight loss and peripheral blood leucocytosis (False) Explanation: Usually there is no suggestion of infection epilepsy persisting after successful treatment of the abscess (True) Explanation: Prophylactic anticonvulsants should be considered bradycardia and papilloedema (True) Explanation: Raised intracranial pressure headache, vomiting and confusion (True) Explanation: With focal hemispheric signs positive blood and CSF cultures (False) Explanation: Lumbar puncture may be hazardous Question 39. The typical features of adult tuberculous meningitis include headache and vomiting (True) Explanation: And general malaise

By A. H.

MCQs VIA WEB 2005 fever associated with neck stiffness (True) Explanation: Fever often low-grade cranial nerve palsies associated with coma (True) Explanation: Cranial nerve lesions in 25% of cases miliary tuberculosis is often present (True) Explanation: Usual source of infection CSF cell count > 400 neutrophil leucocytes per ml (False) Explanation: Lymphocytic meningitis Question 40. In the treatment of adult pyogenic meningitis penicillin therapy should be given intrathecally initially (False) Explanation: Intrathecal penicillin is both unnecessary and dangerous chloramphenicol therapy should be considered for penicillin-allergic patients (True) Explanation: Covers meningococci, pneumococci and Haemophilus antibiotic therapy should not be given before CSF analysis has been undertaken (False) Explanation: Start therapy if the diagnosis is likely, given the mortality and morbidity parenteral fluid therapy should be instituted immediately (True) Explanation: Septicaemic shock often complicates the disease the onset of a purpuric rash suggests drug allergy is likely (False) Explanation: Suggests meningococcaemia Question 41. Recognised causes of viral meningitis include herpes simplex (True) Explanation: Sometimes with encephalitis poliomyelitis (True) Explanation: With subsequent anterior horn cell infection arenavirus (True) Explanation: Lymphocytic choriomeningitis Coxsackie viruses (True) Explanation: Common cause in UK mumps virus (True) Explanation: Usually self-limiting Question 42. Typical features of adult viral encephalitis include acute onset of headache and fever (True) Explanation: Usually no prodrome partial epilepsy and coma rapidly ensue (True) Explanation: Occasionally a mild impairment of consciousness decreased CSF glucose concentration (False) Explanation: Suggests pyogenic infection temporal lobe EEG abnormalities are pathognomonic of herpes simplex infection (False) Explanation: Other viruses may cause this meningism (True) Explanation: In 75% of patients Question 43. The typical features of multiple sclerosis include invariable progression with relapses and remission (False) Explanation: Only 25% of cases have a chronically progressive course onset often occurs before the age of puberty (False) Explanation: Rare in childhood choreoathetosis and parkinsonism (False) Explanation: No extrapyramidal features urinary urgency, frequency and incontinence (True) Explanation: In spinal involvement epilepsy, dysphasia or hemiplegia (False) Explanation: Epilepsy and hemiplegia are unusual

By A. H.

MCQs VIA WEB 2005 Question 44. Useful investigations in diagnosing multiple sclerosis include visual and somatosensory evoked potentials (True) Explanation: Can detect clinically silent lesions in 75% of patients magnetic resonance brain scanning (True) Explanation: MRI more sensitive than CT CSF analysis for oligoclonal IgG bands (True) Explanation: Occurs in 70-90% of patients between attacks electroencephalography (False) Explanation: Non-specific abnormalities electromyography (False) Explanation: Test of lower motor neuronal disease Question 45. The typical features of idiopathic parkinsonism include hypokinesia (True) Explanation: Impaired fine finger movements early-onset dementia (False) Explanation: Cognitive impairment develops in about 30% of patients as the disease progresses intention tremor (False) Explanation: Resting tremor 'leadpipe' rigidity (True) Explanation: Also 'cogwheel' rigidity if a tremor is prominent normal eye movements (True) Question 46. Clinical findings consistent with the diagnosis of idiopathic Parkinson's disease include unilateral onset of the disorder (True) Explanation: Typically arm tremor emotional lability (False) Explanation: Suggests underlying cerebrovascular disease oculogyric crises (False) Explanation: Suggests drug-induced extrapyramidal disease extensor plantar responses (False) Explanation: Suggests multisystems atrophy (MSA) impaired voluntary eye movements (False) Explanation: Impairment of conjugate eye movements suggests progressive supranuclear palsy Question 47. In the management of Parkinson's disease anticholinergic therapy is the best first-line therapy for hypokinesis (False) Explanation: Principally useful for tremor levodopa should be introduced as soon as the diagnosis is made (False) Explanation: Early introduction means earlier waning of effect hypersalivation invariably indicates overuse of levodopa (False) Explanation: May be a sign of undertreatment causing hypokinesis dopamine receptor agonists, unlike anticholinergics, do not cause confusion (False) Explanation: Neuropsychiatric problems occur with both types of therapy dyskinesia is a frequent dose-limiting side-effect of levodopa (True) Explanation: Sustained-release preparations sometimes help Question 48. The characteristic features of Huntington's disease include autosomal recessive inheritance (False) Explanation: Autosomal dominant transmission clinical onset before the age of puberty (False) Explanation: Onset in middle-aged subjects progress of dementia arrested with tetrabenazine therapy (False) Explanation: May help chorea choreiform movements of the face and arms particularly (True) Explanation: But become generalised cardiomyopathic changes on echocardiography (False)

By A. H.

MCQs VIA WEB 2005 Explanation: Suggests Friedreich's ataxia Question 49. The clinical features of motor neuron disease (MND) include insidious onset in elderly males (True) Explanation: Prevalence of 4 per 100 000 progressive distal muscular atrophy (True) Explanation: Typically with absent reflexes progressive bulbar palsy (True) Explanation: Particularly tongue fasciculation upper motor neuron signs in the lower limbs (True) Explanation: Or in the upper limbs lower motor neuron signs in the upper limbs (True) Explanation: Or in the lower limbs Question 50. The differential diagnosis in MND includes syringomyelia (True) Explanation: But no sensory signs in MND diabetic amyotrophy (True) Explanation: Look for evidence of diabetes mellitus cervical myelopathy (True) Explanation: Treatment may limit progression paraneoplastic syndrome (True) Explanation: Protean manifestations of a number of tumours meningovascular syphilis (True) Explanation: Check syphilis serology Question 51. Typical features of cervical radiculopathy include pathognomonic radiograph abnormalities of the cervical spine (False) Explanation: Changes are usually degenerative and non-specific radicular pain in the arm and shoulder (True) Explanation: Follows the distribution of nerve root(s) painful limitation of movements of the cervical spine (True) Explanation: Only if due to disc prolapse or destructive pathology C5-C7 sensory and/or motor loss in the upper limb (True) Explanation: C5-C7 involvement with appropriate reflex loss neurosurgical intervention is often required (False) Explanation: Conservative management is usually adequate Question 52. The following statements about spinal cord compression are correct metastatic disease is a more common cause than primary tumour (True) Explanation: Usually extradural deposits the CSF protein concentration is likely to be normal (False) Explanation: Typically elevated with xanthochromia (Froin's syndrome) local spinal pain and tenderness usually precede motor weakness (True) Explanation: Pain may follow nerve root distribution urinary urgency is commonly the presenting feature (False) Explanation: A late feature myelography is the best and most appropriate investigation (True) Explanation: MRI is now invaluable Question 53. Recognised causes of paraplegia include intracranial parasagittal meningioma (True) Explanation: Important to remember if spinal investigations are normal vitamin B12 deficiency (True) Explanation: Rare in UK in this severity tuberculosis of the thoracic spine (True) Explanation: Associated with vertebral collapse (Pott's disease)

By A. H.

MCQs VIA WEB 2005 anterior spinal artery thrombosis (True) Explanation: Sudden onset typically spinal neurofibromas and gliomas (True) Explanation: Intradural pathology accounts for 20% of cases of cord compression Question 54. The typical features of syringomyelia include slow insidious progression of the disease (True) Explanation: Onset in third or fourth decade dissociate sensory loss with normal touch and position sense (True) Explanation: Leading to trophic ulceration loss of one or more upper limb tendon reflexes is invariable (True) Explanation: Damage to anterior horn cells wasting of the small muscles of the hands (True) Explanation: A common early feature hyperreflexia of the lower limbs and extensor plantar responses (True) Explanation: Pyramidal tract damage Question 55. Recognised features of neurofibromatosis include autosomal dominant inheritance (True) Explanation: Central and peripheral forms occur café-au-lait spots (True) Explanation: And axillary skin freckling association with multiple endocrine neoplasias (True) Explanation: E.g. phaeochromocytoma intraspinal and intracranial neuromas and meningiomas (True) Explanation: At almost any site nerve deafness (True) Explanation: Acoustic neuroma Question 56. The following statements about dementia are correct 20% of the population aged over 80 years suffer a dementing illness (True) Explanation: Most commonly Alzheimer's disease inheritance of the apolipoprotein å4 allele is associated with multi-infarct dementia (False) Explanation: Risk of Alzheimer's increased four-fold cerebral acetylcholinesterase inhibitors arrest progression of the disease (True) Explanation: Particularly in Alzheimer's Alzheimer's disease is characterised by the presence of neurofibrillary tangles (True) Explanation: And amyloid-rich plaques associated parkinsonism suggests possible Lewy body disease (True) Explanation: Patients often made worse by levodopa therapy Question 57. Recognised causes of a generalised polyneuropathy include bronchial carcinoma (True) Explanation: Typically sensory rheumatoid arthritis (True) Explanation: And systemic lupus erythematosus; also cause mononeuritis multiplex vitamin B12 deficiency and folate deficiency (True) Explanation: Also vitamin B1, B2, B6, A and E deficiency amiodarone therapy (True) Explanation: And numerous drugs diabetes mellitus (True) Explanation: And myxoedema Question 58. Clinical features typical of the following polyneuropathies include predominantly motor loss-lead poisoning (True) Explanation: Look for haematological clues predominantly sensory loss-post-inflammatory polyneuropathy (False)

By A. H.

MCQs VIA WEB 2005 Explanation: Motor weakness predominates painful sensory impairment-alcohol misuse (True) Explanation: Also autonomic neuropathy with local sympathetic neural dysfunction sparing of the cranial nerves-sarcoidosis (False) Explanation: The 7th nerve especially is commonly involved in neurosarcoid prominent postural hypotension-diabetes mellitus (True) Explanation: Suggests autonomic involvement Question 59. The typical features of Guillain-Barré polyneuropathy include onset within 4 weeks of an acute infective illness (True) Explanation: 1-4 weeks, usually after viral infection peripheral paraesthesiae (True) Explanation: Paraesthesiae spread proximally ascending flaccid paralysis with areflexia (True) Explanation: Muscle wasting is usually absent sparing of the respiratory and facial nerves (False) Explanation: Cranial nerves involved in 30-40% normal CSF protein concentration and cell count (False) Explanation: CSF protein is elevated, cell count is normal Question 60. Typical causes of proximal myopathy include hypothyroidism (True) Explanation: And also hyperthyroidism; both resolve with treatment type 1 diabetes mellitus (False) Explanation: Causes a variety of different peripheral nerve disorders Cushing's syndrome (True) Explanation: And also acromegaly pernicious anaemia (False) Explanation: Causes a peripheral neuropathy and spinal cord degeneration chronic alcohol misuse (True) Explanation: Often with a peripheral neuropathy Question 61. Acute confusion in the elderly is likely to be the result of an adverse drug reaction (True) Explanation: E.g. opiates, levodopa hypothermia (True) Explanation: Check core temperature with a low-reading thermometer bronchopneumonia (True) Explanation: Consider the possibility of meningitis myocardial infarction (True) Explanation: More often asymptomatic in the elderly cerebral infarction (True) Explanation: CT to exclude subdural haematoma or tumour Question 62. Recurrent dizziness in the elderly is likely to be the result of an adverse drug reaction (True) Explanation: Especially if associated with postural hypotension postural hypotension (True) Explanation: Absence of attacks when lying in bed is suggestive Ménière's disease (True) Explanation: Rare in the absence of hearing loss vertebrobasilar insufficiency (True) Explanation: Common and may be reproduced by head movements sick sinus syndrome (True) Explanation: Dizziness is more likely to occur with bradycardias than tachycardias Downloaded By Ahmed Hakim

By A. H.

View more...

Comments

Copyright ©2017 KUPDF Inc.
SUPPORT KUPDF